Sie sind auf Seite 1von 120

WKk9 BHUSHAN,MI3

Uni~rrsity at I;aliin~nia, San 1:rancisco. Class 01 19111 Series Erlilor; Dia~nosticRadiultqihl

WSHAL PALL, MSBS


G t n e r n m c n r Merlical ~ ~ ~ (:hanrliparI~. l l ~ ~ c India. , Claw of Illqfi Seric-r Erlitor. C. r ~Texas, f Ihlveston. Rrsirirr~ti r ~ l r l t e n ~ a Metliciine l Pr Prex entive Merlicir~e

TAO LE, MD
I rnivrrqitv nf I :,ltiror~l~il, Sari Francism, Clasq nl 10516

HOANG NGUYEN, MD, MBA

Blackwell
Science

COMf FXRUXORS
Sandra Mun Cnirrrsitv of Texa.; M r dicnl Brarlrh, Clns:. of 2002

Beth Ann Fair. MD E<~\trll'ivgi~lia t~ Metlical Scl>c>ol, Rruidml i ~ Erncrgr11r.v r hleclicinr


Kristen Lem M@II, MD Cn~trrsi~v (IF F;a~~sah Schr)ol of Wrrlirinr. Rrsidrnt in Kxliology

Mae Sheikh-AL, h . I D C'II irmrruity OF Dnma\r115. Svria, C l a s ~ of 1119t)


Shalin Patel, MD k?f.I;ra~v Mrrlical Crhntrr.Nor~l~\r.rslcnl L~II~VW".;~N, Re~idrnt in Intrnlal JlrtFicirlcJose M. Fierro. MD

I,n Snllc I ~ n i ~ ~ ~Vexico i-si~~,

F A C U L V ?EvTEM!Efl
Warren Levinson.

MD. PHD

I'roles.sor r > T Micrnt,Enlop' a n d Imm~molngv. LUC:SFSchool o f Metlicir~r

Q21109~ Rlack~vell Scicnce. Inc.


Editorial Offices: Cnmmcrlr~ I'lace. 550 Main Rrl.cct. M n l d c ~ ~ , Macs;~chuact~\ 11'11 48. LISA Oane): Mvacl. flxfni-d OX? IIEl.. E11gI:ind

25John Slrt~t.1, I.uncl(m WCI N ?BS. Erighncl IiAI, 23 A i ~ ~ sl'l:ice. l i ~ I<# b i ~ ~ l ~ ~I? r~ l i %(irti+t\(l
34 Utiivci.r;itvSirtci, Carliui~. \7ctrlria 3055.

Attslr~Ii?~
Other Editorial Offices:

.4ccqiiisitio1~s: l.nur;~lL)cYottt~g Devrlopment: :\mv N~~rthrnck PI-nrlrrrtinn:I ornn Hind and Shawn Girsberger %ktrutfac'l lirirq: Lis;~ FI;liia~an \{i~rkrringShii:i~rr: hlern Mu1c;rhy Cover c l r s i ~ Ilv l L~>slitHai1nc.u J n ~ r r i o drsign r lw Sharvil Cinbeqcr T?w w ~ I )r T c i c c h 1300 b\ Prinrrcl and b n ~ ~ nhv r l C:apital Cih. Prrss Blackwelt's Unde-und MimbioIogy 1,3e lhliN l &A7?-04517-7
Clinical Vignettes:

Rlackwell Wisscnl;cl~nli+\C~ l ~ Criil>l3. g K~~urff~rrtendninm 57. l(lT07 1Ecl lit>.C;cr-m,lnv Rlackwell Srirnce ItK. MC; KrxIrnrnacl~o f<u~lrl~nq, 7-1 0 Ko~tt:~ii~~~irlitx NiIiil~iil).>\!~i, t?I~~ir~hi~ TnLyrl 10.1. ,Jxp;rn The RIackwell Scicncc Iogc is a tl-ade mnl-k of I o \ : j Scttc C'nivc~ s11y P i ~ c u A . Elarkwell Srienrr BPLtrkwrITScie~lce Ltd., I-v~istrjrri at. the LhitecI Crjrnplnv. "12 l S State ;\vcnuc. .krlcs, 1uw.1 Kiiil~grlo~i~ T~trlt. M:irks RcgisLiv 501ll4HsOn. 1'SA

Iliatribiicors:
Tlrs A mmmr Rlackwrll Pithli~hinfi c / o AIDC ELI. D r ~ s 2f1 50 Wintei- Spnn t=int. h'i Ifkto~i, 1,T1154!I7)4)W20 (Telt.phr)ne clrrlrrs: RlIO-21 A9522: Tax 01-rlcr,:Htl?-Xf*l-76!2li) A,ti,<trnli<r 13f;tc~kwcll Scirnce Pk1: 1 .td. 5-1 llnivcrsitv 81rect Larl t o ~ ii'irinrid . 30.52 tTelepIinr~c nl dci,s: 0:<-93.17-0300: rax nrders: 1 IS-!14.IY-.'%lll t;) C l r r t s i t i p Thu .4mprjrr7v ov~rl ,-lv~rt~r~liu Rlacklvelt Scicncc. Lid. c / o Marston RorjL Srn4rt.q. I .ttl.

Library ol' Congress Cataloging-in-PtblicalionData


Bhusllan. G'iku. BIackwr:Fl'x 11 r i d ~ ~ r ~d ~i nical o ~ r vigrwt~es. ~~d Mic~-nl~iulr~gy Ji\i~~h VFku ~ ~ r I31t11~11at1. , - 3rd cd. p. : cnr. - (Llnderxround clinical vigr~citcs) Rm, eed. ol- h,licrobiuIom / Vk;ts 'Dllasllan ... l e r al.]. 2nd erl, r lCj!)!b. ISRY Ofi32-0-0451'i-7 ( n l k . paper) 1. Mcclic:tl rr~icr-clhir~h)~~ -C L Wsll~dies. 2.Phpirianr - L,icensrs- Llni~ctl Srari-s Examinatiuns - Srrlrly guides. [DNLM:1. h,licrohinlop - Casr Rt:poi-t. 2.Mirr-nl~inlop - I'rohlcnls mid E X C I . ~ ~ ~ - S . l J < 18.2' B.57513 2002j I. Title: Micrnhiolnq. 11. Tiilc: l t ~ ~ c i t r g cIini~-itl ~~(~i v~ i g~ ~~ ~c ~i l ks. Mir-rt>hinlnp.IT!. Microhiologv. 11'. Tillc. V. Scrics. QR,16 .B,16L 2n02
ti1 li'.O~'O71~(1c21

P,(>. b x ?{;!I
Allir~grlrm 0 * 0 l l 0x1-I -I\?J End;uirt (l'clrplio~~r orders: 4:i-C1123~AT,.550fl; i i lxrrlers: ~ 4.U) I ?RTAti5555 1

All r i ~ l i t s rcsc~vcd.No part nf t h i s honk ninv I,r rt'pn~Ci~c:rrI in anv hrnm { > r h1f nnv clcclronir ur
inecliani~tfrne;trls, i i ~ c l ~ t dinfomiatinn in~ st~>txxc and ret~ierral systems, rsitllout pcr,l~li%siun ill writing frcxn the publisher. exr:rpt ihy a rrvicwrr whn rnny ~IIOIP brief passages i l l n I-ccierv.

Acknowiedgments Preface to the 3rd Edition How to Use This Book Abbreviations
Cardiology

xiii xvi Mi

Acute Bacterial Endocaditis

'@m
s

Dermatology

ENT/QphthalmoIogy

Gastroenterology

Myocarditis-Viral Pencarditis-Acute Pxosthetic Valve Endocarditis Subacute Bacterial Endocarditis Cellulitis Erysipelas Erysipeloid Erythema Infectiosurn Impetigo Mo~luscurnContagiosum Fitysiasis Versicalor Roseola Infanturn Scalded Skin Syndrome Tinea Corporis (Ringworm) Urticaria Acute Conjunctivitis Acute Sinusitis Allergic Rhinitis (Hay Fever) Common Cold (Viral Respiratory Infection) Herpes Zoster Ophthalrnicus BSV Keratitis Otitis Externa Otitis Media r Pharyngitis-Adenovirus Phavgitis-Streptococcal Fitz-Hugh-Curtis Syndrome Gastroenteritis-Staphyiucoccu~ aureus Hepatitis A Hepatitis B-Acute Hepatitis C-Chronic Active f
b
A

<
$

'

Hookworm
Necrotizing Enterocolitis Neutropenic En'cerocolitis Peptic Ulcer Disease (H.pylon') Pinworm Infection Rotavirus Diarrhea in Infants
I

Geneh'cs Hernatology/Oncology

Infe&'ous Disease

Salmonella Food Poisoning Spontaneous Bacterial Peritonitis Travelers Diarrhea Vibrio parohaemolyticus Food Poisoning Vibrio vulnpcus Food Poisoning Whipple's Disease Yersinia Enterocolitis Chgdiak-Higashi Syndrome Anemia-Aplastic Crisis (Parvovirus B 19) Anemia-Diphyllobothn'um laturn Graft-Versus-Host Disease Hemolytic-Uremic Syndrome (HUS) Actinomycosis Acute Bronchiolitis Acute Rheumatic Fever African Trypanosomiasis AIDS-Related Complex (ARC) Amebic Colitis Amebic Liver Abscess Amebic Meningoencephalitis

Anthrax Aspergillosis Aspergillosis-Allergic Bronchopulmonary Aspiration Pneumonia with Lung Abscess Atypical Mycobacterial Infection BadSus cereus Food Poisoning B arto nello sis BIastomycosis
Botulism Brucellosis Campylobacter Enteritis Candidiasis Cat-Scratch Disease C hagas' Disease Chlamydia Pneumonia
Chiamydio tr~chomatis

Cholera CMV-Congenital CMV Pneurnonitis CMV Retinitis Coccidioidomycosis Colorado Tick Fever Croup Cryptosporidiosis

Diphtheria Echinococcosis Ehrlichiosis Endemic Typhus Epidemic Typhus Epiglottitis Gas Gangrene-Traumatic Giardiasis Gonococcal Ophthalmia Neonatorurn Gonorrhea Granuloma Inguinale H, influenzae i n a COFD Patient Hantavirus Pulmonary Syndrome Hemorrhagic Fever-Crimean-Congo Hemorrhagic Fever-Dengue Hemorrhagic Fever-Ebola Virus Hemorrhagic Fever-Renal Syndrome Berpangina Herpes Genitalis Herpes Zoster (Shingles)

Tl~roz~ghou he t pl-nc1uctio1.1 or this hook, rvc have Iiacl llle s u p port trf Inany friendc al~rl tollcagl~cs. Spccinl th;lnkr: to nitr S I I ~ port ream it~clurling An11 G n p n . Andsea F u l l o ~ vAnastasia ~, A~xlcl-sou. Srisl~ti Gi~pta. Mona Pxll, JonatIi;tn Tiir.;ch ;tild Chirag , h i i n . Poi. prior coiitrib~tlir,nc w.e thiit~k I:inilni L e N ~ l v e n , TAT-trrl MatIi111;Ales GI-i~nm, Sonin Snntos and E l i ~ a h e t h
Sandu1-5.

I V r tlaw mioyrd working with a ~\rc~~-I~E-cI:zss intcr~iatior~al ptib li~hiilg p-ni~p at B l i ~ c k ~ ~ Scicncc, rll iilcli~tling 1,;3~ir;1 11)cYo~1ng, ,%ny Nuithr.uck, Lisa Flanagarl, Sha1z.11Gil-uhprqrr, I,orn;l Hind ancl C:OI-doll Tihbitts, For krlp with srct~ring image5 for the

er~tire series MY a l s o thank I,ee Mat-tin. I<r.isrol~lrcr~joncs. Tina R r i ~ i / / i ;it~d Peter Anrlel~otia1 the L'nivrmitv oFAlaharri;t, ihe .-2r1nrd For-ccs Inslitutc of PathoIugv, ancl rnany ol'our frllnw Clack~vcll Scicncc ~ i ~ t 1 i o ~ ' i .

For

~ l l l ~ l lting lli CCII~~IIIC L%. 'I~ cr~r~ecrions. erliung,

prooCrcacli~rg.

anti ; ~ ~ r i s t a n c arrnsc c all o l t h r ~ ~ i g n c tti11c.s t c in all eclilions. we

collrctivclv thailk:
3'a~;t Adi~n~c>~icli. I~irol~ ,~ I !i ~ is a n r l ~hi? t : iuclen. I Icnrv E. A I ~ I I I . I .vn lnan 15acnIor. Natalie Bal-tenc1-a.Dean Bar-tholon~ew,

,I111iati IIV

Ilehn.;hiufi R r h e r ; ~S , ~ i i nRllatia, i~ Sanjav E3ii1clm. Dnve Brinton. l3row11. .4lt=xa1-trler. Rrc~wnic.Tamara Callal~an, David C:anc-5. Br-nn Cascy ,+aron C:al~ghcv,Hcbrrt Clrrn, Jonathan {:hcngb Arnolcl C h c ~ u ~,417101~1 g, I:hi 11. Siiiiioti E:l~it)~ea. \hen Cho. S:I~II 11e1 (:ii~inq,G r r t c l ~ e C ~~iJ I I > I It, I b1I:~dimir Coric. Cl~ristt~plwr COF~I-eve. Ronald Cki~ran.JG~rvLirlR. Cuiixlingha~i~. A. Sean DaIiuv. R;~rilaDantl:irnodi, Sunir B ~ F R!';111 . ;\rni;lt~~fn L):~ve. John D;ivirl, F.min;lr~~~cl d r 1;i C:r.tt/, Roljert 13eMellrt. h'a\-neet Dhillon. Shal-inila Iliqsatlai k c . Ilavid D o n ~ o n . hdolf E tchrgi~fi~).; Neil E~lscbio, Priscilln ' 1 .Flasc. Drwirl Frc-nz. Ibistin C;;i~uii~r, Yol~annc.; C;t.l~tec.g/,ial~l~er, Atiil Grlli, l i > l ~ Gctn-gc, v L.51. I;ot nrirc~, Par111L:c>val, Alex G r i ~ n n Ilajecv . Gupta, X1.lmad I-lali~n, Si~e Hall, lj;~virl1-Iaswlbachc~:Tarnra Hci1ner.1, Michelle T-liglrv, D a ~ H l o i ~ Eric. , Ja-lckrr~n. Tim .JilcL~on, S ~ ~ n r lJayamman, nr 1'c.i-'\'i-lone.:\arcI~nn-Jorhi. Kajlii Ii.,lutl;i. Lciviaz Lrpadi. Seth 1L1rl~. --liar011S. I<c.s.;cIh v i n ~S4n;i , Khan, .Z11dre1vI'in-wri Kn. Fmntis Long. Pal11Ficmil/ke \$'al-~.et~ S. li~xrkov, Benjamin H.S. Lari. /I11 n I .,3t:awe. Cc)nn i e Lee, Scnr t Lcp. Cuillcrnmo Lrhrn:~r~ri. I<ci.i~~ Leirng, I'aul Levctt. ll'a~-rci~ Lrvi.ln~orl, Eric Ley, Ken I-in.

Pavel Lobanov,J. Mark Marlrlox, :barn Mardian. Samir Mehta, Ciil Melrmetl, Joe hlcssina. Rolwrt Mt rwa. Michael Muq>h\.:Vivrk N a n d k a ~ n iS , iv;~ Karayman, Cailrell Nguyen, l,i~ili Nguvrrl, Dcanna Nr,l>lr7a, Cixig Nodurft, Ccorgc h'oirmi, Dar-it1 T. Cllii~da. Adam t.Palance. Paiil Parnl~lii.us.~Jirlha. Park, Snnny Patel. Ricardo Pietrohon. Riva 1 . Raltl, :bshita Kanclr~ia. Karhan Redrlv. Beatriu Reig. Marilou Rryes,Jc-l.emv Richrnon. Tai Rne. liick Roller, Rajiv Rov, T)iegc>R u i ~ ,lZnrEiony , Russdl. Sanjny SahPl, Urmirn;~l;lSarkal;*[ohi~ Srl-iilling. r ~ a b e l l Schrnitt. I3;ircil Schnl~macher. Sonal Shall, Fnrli Abu Shahin. Mae SheikllAli, Edie Shcn,Justin Smith, John Stulak. Lillian S ~ ~ . J u l i e Sundar.;nn, Kza Suri, Seth Sweetset.. A t ~ l o r ~ Talayrro. io klcrira Tan, Mal-k Tanaka, E ~ i Evlol; c Jcus 'Shr )rnpso~l, IncIi Trehan, Ra\imond Tht-riel; Okafo L'chcnna, Eric Llyg~~arlco. Rich:% Var~iia,Jol~n M h ~ e s Alan , Wmg, Emricc I4'ang, A ~ ~ r M'ei\s. ly Arnv IZ'iIliams, R l i n r ~ Ya'ang, Hany %sky, :r"lshraF Zarnan ancl David Zipf.
For g e t ~ ~ m l ~ s co~ztrib~rting lly image^ 10 the eiltii-c Unrlrymu,nrl C,'!irrtrnl T r i p ~ t t Step r 1 wries. wc culIrctivclv thank the stitfff' a1 Rlackwell Scicncr in 0xfoi.d. Bo'itorl, ancl Berlin as well ns:
,4xl'orrl,J. 1Z/l~rlirj17t. Osncy hlearl: Elnc k1~11 Scie~ice Ltd, 1996. F i g ~ w s 2 . 1 42.15. , 2.1Ci. 2.27, 2.28. 2.31. 2.35. ?..?(i, 2.38, 2-43, 2.ti5a, P.(i.'lb, 2 . 6 5 ~ 2.10Sb, . 2.P0.511.,7.201>. 3.2 t, 3.27, H.L'7b, 8.7'713. 8.7Tc, 10.81b, 1O.Stia. 12.283. 1l.(i, l l . l t i , 14.50.

Banni~tei. R, R e ~ g N, GilIespie S. Itt[i.r/in~rsDZIPCI yo3 2,rYdiiir)ji. Ocncy Mead: nlackwell Scicncc CrrE, 2000. Fi~tii-cs 2.8, 3.4. 5.28, 18.10, U ' 5 . 3 2 . M'5.6.

14cr-gD. Arlun~rr-prl f:linr'rc~lSktlls crnd P h y i i r nl D i f l , p f i ~ i v . 13lackwell Science Ltd., 1!)90. F i g ~ ~ r 7.10, e s 7.12. 7.13, 7.2. 7 . 3 , 7.7, 7.X, T.!),6.1. X.2, 8.4, 8.5, 9.2, 30.2, 11.3, J 5.5, 12.6.
I:~r.;chieriA, I-Iennes5yLrI'J,G r ~ e n l ~ a l g Rhf. h Rowlev Ur\, Gl;we PA. Ulintm/ Snr-yos~. Osiley Mcad: Bl;lck~t~eJI Science 1,trl. 1996. Figurcs lS.l!), 18.22, 1X.33.

* Gil lrspie SH, Bamfol-d li 3 I~dicol ~ V l i r m h i o k(mnd ) ~ ~ It?fkc/ion n / n


Glrintr. Osnev Mead: R~;Ic~Ivc-II Science Ltd. 'lU00. Figt1rr.s 20, 23.

* Ginshrrg I . . I - t r / l r ~ N~ ot~s on .VP~I?Y~~I!;?, 7'hE:rl,lton. (Ssncv M e a d :


Blackwcll S c i r n u ~ T,{rl. 1999. F i ~ i z r r \12.3. 18.9, 18.:511. Elliott T, I-Iasrinv XI, Des.;ell>ergcrU. I , ~ r t l ~ r ~ . V oot) t r \~l/Ic,cIirtrl l\lir.roi)iri!rp, P" Erlriin71. Oqnw Mead: I'llackwell Science Ltd, 1!)97. Figllrps 2, 5. 7 , 8, 9. l l . 12, 14, 1.5. I f . 17. 19, 20. 25. 26, 27, 20. 30. 34. 35,52.

M c h h ,a, Hoffhranrl , % I ' Ilr~(~mnto/o n/ , qrr M ( ~ ~ v CP h.i - v Mead: BlackwclI Scirucu Ltrl. 20110. Figurt-<22.1. '2.2, 22.3.
Please l e ~ 114 k 1 1 0 ~if - your nai-He liaq llepr~mirserl nr mi~rpellerl

ancl wt. I ~ ~ Ijc I I happy to I-liakcthe r~pdatc in the ncst rdition.

PREFACE $84 THE 3!?D WnlTlOY


We were ve1-y pleared wit11 thc u\,c.rwl~elmingly positi\,c stuclent feedback Cnr thr: 2nd etlition oi'otu- I'nrlqrnunrl (Ctiniral T'r,p~tft-r svrics. M'ell over 100,000 copies o f the U C V books are in print atid havr heen usecl b y students a11 owl- the world.
Ch-er thc last twn years w e have accumulaiecl and incorporated over a thousand "r~pdates" and imprr~vcnlen~s ~r~ggesrcrl by yotl, OUT readers, inrl~tding:

* manv additions of

specific hoarcls and warcts tesralde content

deletions oC red~indar~ t aiid ov~rliippinq cases

* r-eordering ancl reorganizarioil c>fall cases in hrrth series


a nt-tv rna';ler index by casc name
ilt

each Atlas

cnrrrctiur~of a few l x t l l i t l errors

diagnosis and treat~netlt updates


addi lion of 3-20 new cilses in evc~v book

and the addition of clinical exam photog~:~phs wilhin l J C l L A nntorny

Ar~d rrio\t important ol all. thc third e d i ~ i o n sets nowr include two I ~ n n d n e w COLOR ATLAS supplements. o n e for each CIinicaI V i s t ~ e le ~ el-ieq.
'I'lie 1 '(:If-Ensir S r i m r . ~ Culor Allr,,~ (Stdl I ) includcs oxmer. 250 color plateu. clividecl into gross patl~olo~gy, rnicrr)sro~~ic patholng-v Il~istology). hcrn:~roloq, and mirr(~hiolagy (smears).

* The I'l:1'-Clinic nl Srr'mrr &lor rltlnt


L'.'PV.

( S l r Y j 2) has over 125 color

plates. including parieni i~nages, rlermatoEop, ;lnrl liinrlus-

Eacla atlas iinago is dercl-ipiively captioned and lirihrd to i r cor~ re~poncling Step I case, S ~ e p 2 case. ancl/or Step 2 MiniCasc.

How Atlas Links Work:


Step 1 Book todes are: :I = Anataslnv I\S = F3thlt:wi<1~il SI.~(-IICC, M:= t%if~t~I~r.rnistry lil - > l i ~ - r ( > h i c ~ l ~ 1'1~1. . > pt, \I!?= \7ic-1-1>hioft >gr. \,>>I.TI

Step 2 Book Codes are: ER = Ernelgrncr, h4c.drclrit. I M I = lnternal Mrrlicirrt*. b 1 . I


I M 2 = ItttprnaE Mrrlicirjc. \'nI. I T

PI I'?

= P,~1lla~~~~l~-~ii~ll~~~. \'i.ll. 1 = r":~tl~trphvsicilo~y, L11. 11 = 1 ' ; 1 t ~ l ! > [ > ~ l ~ ~ ~ < >!!(>I. ~t)~~ 111 .

N 1. I 1 = Y e i ~ r n l n k ~ O R = OR11;l'f PED = Pctl~atrlcf SUR = Srlrqcr e P.W= P\xcliiat~ r

M-P3-032A

Indicates Type of Image: I I

/ -

Il ~ . i ~ i . ~ i n l o ~ ?

1 1 = M~rirrl~iolrrm

ER-035A, ER-035B

Indicates U C Y l or UCVZ Series

la(; = (;rnw I > ~ t I ~ < > ? t y q ~ In4! = \Iirrosrnpir l ' ; ~ r I ~ ~ l o ~

I f t h e Ch\e n u m b e r (052, 03.5. etc.) is noLfoIlo~ucd h v a letlet; then tl1cr.r is only o n c irnagp. O t l i e r w i s ~ A. B. C. D iilclicatc
(11) I 0

4 ilnilge~.

Bolcl Faced Links: In circler to give voll access 10 the larpeqt rrrmiber of irnag~s possikle, !tme have chosen to crow link rEle Stcp I and 2 ~cl-ius.
IF ~ l l e liiik is hrhd-facer1 this indicate5 that the link is dircct (i.e.. Step 1 Case with the Basic S c i m c r Str-p 1 Atlas link).

If thc link is not holcl-filcrcl r l i i h indicates that ilir link is inrlir c ~ (Srep l I care with Clinical Srience Step 2 Atlas link or vice \*el-\a ).
' I k have. also implcmuntcd a Krw s i r t i c 1 1 1 1 2 lr l ~ n n g e s upon your
reclt lesi:

* ~ C IC I U T I - C I I ~ant1 fl~ture erli ion o f otir p o p ~ ~ l a !:in! l . rlirl Jbr 1j1t) 1 'S;kli-l< ,Vr// J applet to^^ k h n ~ ~ / ~ ~ c G r a ~ v - H ainld I) Fir<{ Air! [or rht, I:F1Z.lI2fi: :'.Vtrjj2 (Xpl>lrtorl Pc T,ange/MrCrat\vLHi11) hook will he linked ro the cortpsponrling t J W caw.

.bd li~lks a4 rhev frequentlv I>c-c.o~~it~ cmt of rliil e , as the I;irrl (1 id hooks are revi~ed yearly
Fl'c clirninatc-d UCV t First

Thc I:r)l{br- ,41l;lric also y>rciall!' dc~igner! li>rq~lizzingwptions arc descriptive ancl do 11131 ~ i v away e thr caw Ilarlrc diirc tly.
11'~ hap^ the l~pdatecl. L!Clr scrim rvill rcmain n uniqur and welt ilitegri~lr-cl st\~dv r r m l illat pr.o\iclt-s rwrnpact clinicril cnl-l-e1ations I ~ bacic I wirnce inIi>l.rnatinn. T h ~ are y cle<iyird to bc, easy and Ftul (cr~rnparati~ely) to read. and h c l p f ~ for ~ l hot11 lirrn.;ing
esarnq mntl tl~e ~~i~rcl<.

iYc invile \.orlr correction^ ;tnrl s t ~ ~ g e s t i o!'or t ~ stlre Court11 editior) ol-ili~re hrroks. For- rIie f i l - ~ rstlhmi<rintl oreach r;lctual crxi.errinr~ or. rlelvvig~~etre that is selcctecl T o r inclusinn in [he li>urth crli rion. will rccci~c a pcrct>nalackno~\*lrtlge~nen~ in [he revisctl hook. TI' volr rtlbrnit ovei. 20 l i i ~ h q i l a l irr ~)r~.ertions. ~. nclrlitir~ri\or ~ I P W~ ' i g ~ e t we t r will ~ illco rot1l;irler inviting you to become a "Contrihuior" an the book of your choice. [ f v c l u arc I ntel-rst~d i t l t-lcrnlni I I a ~ potential "C:ot~trih~~to~-" or "Ai~thor" on a rurul-e LIC:\' hook. a l working with o u r team in develc~ping

a(ldi1ii)i~al booku. ~ ~ l r aalsn s e e-innil 11s yotii- C3:Z:/re%1ime.


'In('?

prrl'er thai ?nu submit rnrrectinns or c r ~ g g r s t i o via ~~s eIectrr~n ic ttaail t u UCVtearn@yahoo.com. PI<-asei n r l ~ l d r "l-ntlrrgr-ntuirFVigliertes" xs the ~tll>ject oT~wiir mcwagr. If y ) r i do iiot havu itcceh? to r-mail. use ~ h ibllowirig c mailing ildtl1TW:

Rlac.kwcll P~il)lishing, Xun: UC\' \4;lldvn, \ , I ; \ 021 48, USA.

1 ' t l i i c ) r q .

530 blain Stree~.

TI1i~ ~ e r i waq e ~ nrigi 11allv tleveloped to addrcss rhc increasing numher of cli~~ical vipvttf: qucctinns on medical exarr~itla~ions. incl~lrlirrg the LTSMLE Step 1 and Step 2. I t is alvu designed to supplemmt anrl complerr~cnt ~lic popul;~iFir.\/ Aid /or lht. IJ(Yl\.II,l<; .?;,I>/) I (Appleion & I,at~g-~.,/MrCraw Hill) and F i r ~Aid f jifr I ~ I P ? $ : l f l S!q) , l ~ 2 (Appkton & 1 ,a~~gc/\frGrmv Hill).
Each UCl' 1 hook u s e s a ~ei-ies of appsosim:!tel\ I I)O "supraprototypical" cases as a T V ; ? ~ to condense testable Facts and a5sociations. Tlic clin icul vignette< in this writ=?are designed to incorpotn~e as many testable facts as possihle illto a c o h r s i ~ ~ c iinrF mrmorahlc cliniril pic ttkre. T h e vigtlette~ represent compo5i1e3 drawn frnnl general ararl specialtl; tcx~books. rrfcrencc I~ooks, tl~ousnnds of USMLF, ~tvl e queslions anrl the pursr~nalcxpcrin~cc of die atllllol-c arlrl re\-iewers. Al~hoitghpi1c11 rase rendq to present all tilr sigrls. symptoms, atzrl diaqnosur ljnrlings Tor a particular illness, patients generaIIy will not present with such a "complete" pictrim either clinicaIly or on a medical examination. C a s c ~ arc not rnearlr to simlllatc a potcntid 1-ral patient or a11 erarn vigiietle, :\I1 t h e boldfaced "buzzwords" are for learning purposes and are not rrrcerwrily especreri m hr Counrl i r t any onc paticnt with the disease.
1Sefi I I ~ini~s I of srl~creil important ~rt-~ns arc placccl within the

\8igiictresi n ( n 1 . 1 1 I. (/\P.;) i n parentheses. Otl-rcrpkrenthctical remarks often rcfcr to the patl~uphvsiolo~p or rrirchat~ivn of clis~ase. The Li)l-n~at SEIC)LIICI alw help stucleilts lcarn to presenl CXWF s~lrcitlctly CILII-~H r~ ~ l a "br~llct" l prcscntatir>il$on clinical rotntions. T h r cascs are Incant to w t v e a'i a cot~rlenserl review, tint F I ; a pl-iinars r e r ~ r r n c cThc . information pr-o~ided in I hiq hook has bucn prrpa-rrd wit11 a g r a t deal OF r h n ~ ~ ga hit d carrfill r.ese;~r-ch. T h i 5 lmok vhouid not. however, lsc considered as yntlt- wte musce nT inCormntion . Corrections. suggcsticms and cuhmi~sion~ or new caw5 arc cnco~~ragr=d and will he acknowlcclgcd and i ~ r c o q ~ o r ; l tvheii ~ r d :~pprnpriate in Cut~~re cclitions.

5-ASA ,4R(;s
, I 1 B % B AC t<

ACT1 I !ZDI I
r\FP

A1

AIDS :2LL ALT rLML m ,4 r tk DS

XSD
IZSO

AST

11v
RE
RP

I3UK C4D CALLA CRC CHF CIi


GLL

CMI, CMI/ CNS

coPn
CPK CSF CT m , 4 T A XR DlC: DIP DKi\
DM

DTRs

n n

5-atninosalicvlic acid arterial hloucl @wes aririalnvcin/blec>mycin /vincristine/dacarl~azinc angiorcnsin-coi~vertir~g enzyme adreiiacr>r-ticotropic I ~ o ~ r r ~ o n e a ~ ~ ~ i d i ~ hormone ~retic alpha Fetal pi.otrin iiort i c i n~utlicicncv acquired irnrnunod~ficiencv sydrorn~ acute Iynpl~ncptic lrulremia alanine transaminast. ;IC LIIP n~yelogenoux leukemia an~intlclear antibody adult respir-atnn, dish-css syidrnme alrial septa1 defcct anti-sl r e p t o l y i 0 asparlale tranmminasr ;11.1ctio\~~nous barium cnerna blood pressure l~lonrl urea nitrogen r n r n n a r y arten disrase comlrlon acute Ivrnphcrhl;istic Icukelnia antigen complete blood c o u i ~ l congrstive heart f ~ l i l ~ l r ~ creatine Knaw chroilir Ivmphoc~~ic leliliemia cllronic rnyelogcrln~~s leukemia cytrwnegalori~~ls rpntral nen-ous System chronic ohstnictive p1~1rnona1-y disease crui~tirie phnsphokinnsc cel-ehrnspinal f l l ~ i r l comp~tled t nmography LL'I ekro\-a-asrr~lar accidt.11t chest x-ray dissenrinalrd inrl-avrlscular c o a g ~I luon ? ' diqral intei-phalangeal diabetic krtnacidmis diabutcq rnellit~~s deep rendon rcflexus deep rrenoIls ihm~tllmsis

Epstt'iri-R;zr r \. ir-11% c.lrctrr)c;~rtliclfir;~~>Iiy t-choc.i1r(liog~11111~ r-iection i h c t i o u ccopI1~~~~~jiau1rr,rlnode1~o~1-r~~~~


clrc trr>m.i.agrap?~y

t=ncloscopicsctroptle cholilnginpa11rrexto~rap11y r.n*hroc!.tc uttlimcnl;~ticmrdtc li~rcuclt-spir.;~~or-v vol~ime fine necclle aspiration flunl-rsrcnt trcponc~n;~l antil~ntly al~sr)rl,tic)t~ lorcud vital crlpilci~y glurnrn11;)rfillfittio~i~ x t ~ ~l-owth Ilnl.t?lOll~ ga~troin tcs tinal granuloc~,lc macrophagc d:~lc)nt' \rit~~~~!atit~g
L1ctol.

(;L:
H!\IT

hcc; I-IEENT

Mn'
HI-*\

I-IPI HR HRIC: 11s

rn/cc
IDDM
TK I(:F 1\4

.PT
I<L' R I ,1)H LES I,Im 1.P LIT I, I 7 1 I,\T(==< MC:FIC: k t( 1 ; bIl.:N

gcnirclt~riiian hepati~is 1 1 vii.11~ hurn,rri chnriclnir guii;irloti-c ~phiii I I I X I ~ , cvcs, ears, rlclrr. ;?rid thr-r);tt htimarl i r n r n ~ t n n r l ~ f i r i ei,il-llr i~c~~ hzuna~l l e ~kr~rvte i antigcn 11i.itu~-r. r , f ' ~ r - c ~ 'illr~es\ ~~'iit 11ea1.tl.FI(F hurnan rnhies i ~ n m llnc qlr>h~rliu tlei-cclita1-v sphrrucytori~ irlen~if car ion :~iirE r11ic.l' rr)inplaint ii1.;11lii1drpc-irrlcntrliahc-trs nirlri 1119 iirunl~nogloht~lin i11~~ilir1-1ike gt-t~wil~ ~~CTOI. int~ami~~cular jugkil;~r \vnnli.i pr-essurr kirl~~~y~/t~i.~~tei./!~l:~c!c!~~. I;~cr;tte cleh\,di.n~enaw lowcr rsoph:igral sphincter liver- F ~ t r l r ~i t-i ~ Ii r~l i t q 111n117:11. piii1cllli.e left ~ v l ~ t r i c n ~ l a r lefi ~,ei~ti-icuIar hvpel-~rnl,h\
electrol\2e<

iriean rorpusr,t~l,~r hrinnglohiii rnncctirratint1 mean rtrrp~tsrtllar vol~~rnc rnliltiplr eiirlncl-inr ntv>pla\i;t

MHC \,f 1 MOPP


\I K

NHT, NIDDM NPO NS,IIIJ PA PIP PRS


PE:

PFTs Phl l PlvIN


PT

F I ' ( :A
PTI 1 PTr PU D
1 m:

RPR

RR
RS RV
RVH

%BIT SZ4DH SLE

SrI)
TIT5

1 Pji TST I TIRC

'r-I PS
TPO
TSH TTP UX LTI:I

L's

r n o ~ ~ o r l n n gamniopathv al of'~~nclerer~niuecl ~ign~lic~~~~ce ~nqior histocompatihilitv cumplcs ~nvoc;~l-dial infarction niccFilr~retl~;~i~~inc /\iiiciistine (Oncovc>tin )/ pr~clca1.1~17.i rtc/predniwn~ ma~rsetirrewtiance (imaging) t~nti-Horlgld n's Iynphorna non-inslzlindepenrlcl~t diahctes mellituq t~iI p r r os (notliing 1 1 1 rnoltth) nr?nu~er.c~idal a11ti-i rllla~~ltn:~tr~l-v dnig posrernan terinr prnsimal intel-phalangeal pcriphur;~lblood srncar pl~yir:~ ~X ]; I I I I puln~onan: r~~nctim terts i point nf tnaxitnal i n t e n s i c pol!mcn-p hr~nuclear lcukocvtc prc>tl~rumbin time perrlllaneolis 1sxn5l1nnilialangioplasnv parath!~o irl lionnone partial thromhopl;~stintimc prptic lllcer diseasc retl hlonrl cell 1.ilpid pIilsS~lil r(-i1~11 r~spitalon' rate R~erl-Srcrnhcrg (crll) right ~ ~ n t r i c r ~ l a r right vrti 1riri1l;lr h ~ ~ ~ ~ e r ~ t t - ~ ~ p f i v 51nall howel rollow-r h r o ~ ~ g h ~yndrol-r~c oC inappl-rjpl-iatc secretion c~f .4DH ,systc.nlic l ~ ~ p r=rythcmatr~sus us sexually t I-anvni tird tli.;eal;e thvroirl fi111c tion tests tissrle pl;tsminnge:r!~ actimtlrc~t. ali!~nirl-5tim1r lating hol-nir me rntal il-rjn-binding capaci t\ rr;lnsjt~gr~lar. iri~ralirpalicpol-~usvrrernir sli~tnl t h ~ ~ o pcrr>sicIasc irl zhvi-oirl-$tiin til;l~i~ lm ~g -rnr~i~c t hronrhotir ~ f ~ r o n l h o c v t o1~urpur.a ~~~~~ic ~iriualvcis 2 1 1 ' P P " GI ~iltmso~~ncl

1 . S L-1.
MTSC:

MTIY

SR

\'~n~rra Disease l Research L a b o r a l r u ~ vital sig.11~ veu t ~ i c u l a r tacl~vcardia rz~lritch l o r ~cclE l .2'Tr1lff-l'nr~kin5nn-M71i te (.wndrome) x-riiv

A 25-war-old W Q u g abuser prercnts wi111 a high fever with c-hill.i, malaiw, a prnriuctivc c o ~ i g hhr.mopvc;is, , ant1 right-siclet1 pleuritic chest pain. IIc also rcpvrts r r l r ~ l t i l ~skin l e in~t=ctions at i i ~ j c c t i o ~~ i l~ es.
VS: fevei: PI<: stigmata of intravenorrs drug abuse ;+t multiple injection sites: ski13 i n f c ~ t i u n \tfi~.niiihofed ; peripheral vcius; splenomegdy and pulsatile hepatornegaly; ejection systolic m~umur, incrr-asing cvirlt inrpiz-alinn. heard in cric~t.jpicl ai-en.
tabs
r l
i3

>

E <
G1

CM:: n<)rtnocht-onlic,normoc!.tic anemia. U:2: ~nirroscopic


hcmat~~~ Blood i a . c-1111t ~ yicldq r ~ Sfaph?lococnrs
alr reus.

Imaging

Echo: prcsenc~' o f vegetations on tricuspid valve and tricuspid incompetence. CXR: consolidntion.

Treatment

High-dose in~avenous penicillinase-resistantpenicillin it1 cumbination wirli a n arninoglycoside. IC the. isolalrrl i,~~,rrfrrs strnin is methieillin resistant, vancomycin is thc clrug c ~ tchoice. '
In rlt-~lg addicts, thchtsicuspid valve is t h e ~ i t or e inrection mor-r CrpqucntEv (55%) than lhe aortic valve (:35!':,1 or thc rilitral txl~e (30%): thesr fji~cliilgc cnntraqt 1n:lrkerllv wit11 tllc ~ t r i t cva t ' rightsidrd irivoli~ementin cases of i~lfcctivc. under-;trdifiu that are n o t assoriatccl with d i - ~ abuse. ~g Shpbio(orr7r \ ~trrr,.r~c i c responsil~lr For thr rnditju~-iry II~C~SPY C. ~ r t a i oljiani~rns l~ haw a prcrlilrctiail for 13a1-ii~ular. valves i n cases af acldict-aqsocialrtl en~lorarditiq; for r.xainple, et~rerococci. uthcr slrchptocc~ccal sppries. and 11011-alhican.; CnnrlirI<ru r ~ a i ~ i s pl-ertn~nitlantly rn~ affrct tlw v;llves 111 111r IeR ~ i r t e of the heart, urhilu S . rr~~rr~ir infertc valves o n hot11 the right and the Teft \ide of lhe heart. P~nrdonlonns org;~riisrr~s arc- ausrlcia~ed t v i h hivrntrirular arlrl rn~~ltiplc-valve intkcl inn in nrlclict.;. Corr~pl i c a ion ~ of endocarditis irlclutte congrs~ive heart fail~rrr. r ~ ~ p t ~ vaIve ~red cusp. inyucardial infai-ction. and mvncardial abrcesu.

Discussion

Atlas Link

ACUTE BACTERIAL ENDOCARDITIS

ID/CC

h 25-year-old male complains of irlcrcasing shortness o f breath and ankle edema t hat. have pray-rssively w o r s ~ n ~nver c l the past

2 weeks.
HPI tie also conlplainc or fatigl~c, palpitarions, and low-gmclc fevcr. EIis q\mptc)irlsfollawecl a severe URI. Ho drr~ieq any hiutrll-v of j o i n t pain or skin rash (vs. 1.he11rnaric I'ever).

PE

,PT t,!rvatcd; pitting pcdal cclc-ma;fine inspira~or.?, crepitalions


hu;~rclat ho th lung hasrs: rr~ildhepa~osplenomepaly.

Labs

AS0 ti tci-s 1 1 c t ~ clc*vated. CBC: Iy~ipl h~cylo'ii~ ECG: , fil-stdrgree hlock. ESR clc~.;~tc-d; incra:~.;erttiters of anti bodies to coxsackievirus clcmonstraicd i l l surtiln.

Imaging

CXR: cardiomegaiy anrl pulmonary edema. Echo: dilated cardiomyopathv with low ejection fraction.
Dilil~eclI J ~ X I - with t foci ~ lcpic;irdial. ' rnyocartlial , a11d cnr1ucardi;ll petechial I1emc)rrlmges.

Gross Pathology

Micro Pathology

Endorn\~ocarcliaI biopsy rrr.ealq d i m e infiltration by mononuclear cells. preclominan~ly Iy~~~ph(~cv incal l e q ; fil~roric.
Manage congestive heart hilurc. and i~rrhythmias: cardiac transplant in intr;icti~l>jr c;i.ieh.

Treatment

Discussion

Coxsackie B is 1 1 1 0 4 ~ ortcn implicatccl in ilral myocarditis. Icion~il-alcittlsrs of r n y ~ ~ i ~ r d iilclude iti\ 1~;lcreri;l .;I ~ c h as Ijrwr~lio hlrrgdorw (Lynledi~ieare), parasites s11chas Tr~ltana~ornn rncri (Cliajias' discasc) . h \ ~ ~ - ~ - ~ e ntv s ireaction ti\i (sysletl~ir lup~~q c.r\~thcmatnsus.r l i - ~ ~rcaczion), g radiation. ancI wrcoidosis; tnay also be idiopathic (gian I re11 ~nvncarcliti~) .
[-I-I-u M-MI-002

Atlas Link

MYOCARDITIS-VIRAL

male r o m p l a i i ~ s (ITfever, nonproductive cotrgh, :~nd chest pain.

,"I,?-~'P;~I--cII(I

He <l;ltru 1l~;tt the chrcl pain rleveloprrl after he had a scvcr' ccll(l lor 1 twek. He del;cril>es the pain :lr severe, mslung, and constant ovet. I ~ ;P ~t~ierio C lI-I ~ S Iand atlrlc that it worsens with inspiratian xt~tl is rclieved by sitting up and hcrlding Ci3rwarrl.

r,

> 73
0

rub (sin;tolir-arid di:w~rrlicco~nporietjly fdlow~d hy a third c r ~ m p ~ ~ n irl i l iIi; t t ~ ~ l i a ~ taswriated nl~ wii h a ~ r i a cnntraction): l elevatecf JW: il-iappropriate increase in with inspiration (IC~'5hzr \ L ~ I . ' \S T ( : % \ : plllriu~ ~ T J ~ ~ X may ~ alsu I S bu srrn.
friction

2 <

JVP

Labs

R!!~>~c'I-LIIL'~v rlt'vit tctl 1riii1~i111 ~ii~aqe': il I I I ~ ,1)H : elevated ESR; serum CPK-MI3 normal. C t\C:: n r u ~ r o p h i l i c Ic~tkoc~~tosis. ECG: dirfi~se ST-segment elevation (vu. 1-nl.r mi-dial i 11 Lirul ion) ; P R - s e p e n t depression. E~EIo: pericardial effusion. CXR: ;lpp;treilr cardinmcgaly ( r t ~ l r tn plT~rsinn).
In Iong-:-~larldi~ig c ~ ~ wprric u . :~r.diz~~ii r t l , ~ hrroiiir lihmtic. st;~r~.rcl, anrl calci ficrl.

Imaging

Cross Pathology

Micro Pathology

Treatment

Discussion

Acu t r pcricardi tis i h co1nnlu11Iviriinpa tllic. b t ) ~ c i~l l ik c l i o l ~ s c ; ~ u ~ir~cl~iclc cs toxsnckievin~ A and FZ, tuberculosis,.iti~phvloc-nrcal o r pt~e~lmncnccal iul'ection. ainebi,asi~, or actinoillvcoris: noninfccticuts c ; u l ~ incluclc s clrroi~ic. r t - ~ i ; failitre, ~l
(51 r~ernic luptl~ el-v~hematr~~us, srlr.i.c>rlerma, anrl rhe~umatoirl arrhriti~). ncopl;~srns.nlr.oca1-dial inliirction, anrl tr;ulma. L o n g - t c m ~r.qllrl,iriricl~tclr cl~l-o~iio coi~~lriciiv pt-i=icar.di~i\. r

coFlagen-vasculardiseaqe

Atlas Link

PERICARDITIS-ACUTE

I DJCC

1 1 Ci4vr;lr-nlrl iiialr p r ~ s ~ n\+'it t s11 r ~ p i d l v prog-re~~ive dyspnea

and fever.
HPI

I-Ie t-laq a l ~ i s t n r v of nrthopnca ancl paro~~smal noc tunla1 dvupuea imtl i11u1 report.; pink, f r o ~ l i v rp~~t~t (~ tr ri~ u r ) p - r v s ~ . ; ) . One moil th agn he un del-went a biopmsthetic valve replacemenl Tor caIcilir ar)rtic vtei~osiq.H r i s r l r l t hvpel-tensiw arid has nevrar hacl overt rardizc faillzt-r in t h c pxst.

PE

YS:frver; I~vpclter~aion. PE: Iril,~rcral1,:lsal iilspil-atorv crackles heard: rardiar ausrultation s ~ ~ ~ g e ~rfaortic t i \ , e incompetence (early rliastolic rnllrrnllr ~ P ~ I . c Iradiating clown left sterna! edgc).
CRC:: nor+mutl-lr-omic. tloiq~nr~c!'tic ; u ~ e r r ~ Three i;~. cni~n~cutive hloocl cul~ur-es vielcl coagula~e-negative Sflaphplococc~~r e i d e r m i d i s : st l-jlirl Crn111rl t o he mcthiciUin resistant.

Labs

Imaging

CSR (PA view) : suggesri~u of pulmonary edema. Eclio: cor~firrns


pre~-~tc of -r prosthetic aortic valve dehiscence leaclir~g to incornpeter~ce xntl poor left ventricular runction.

Treatment

High-rlr)sc paren tcral ; ~ ~ ~ t i I ~ i ~ ~ t i c s - v a i ~ ((11-115 c ~ ~ iof' ~~ rh ~n ~c ic i~ n fur n~rtlririllin-r-esistm S. mt~rw.<), genramicin, anrl oral riL~rnpicin: sur-+ul replacernni I of (lamaged p r n s t h e ~ i c valve: p ~ o ~ l i y l iir ~c :mrihiot r ir~ (amoxicillin) Tor patients receiving nrsl /rkrr tal tl.eaIrnpt1 ts rm prpvcln t iransient bactercnlia. Prns~hcric \ alvc rnclncarrli~fs i.:suhtlivirletl in l o rwo raregclries: early prosthetic m l r e endocarrZitis (EPIT), which becemcs clinically nlaniIcct rvithin 60 din-s aftcr r:zlve rc-pli~cclmrrlt (most comiiionly' cau~crl by S/n/rltyl~rrornia r/~i(l~rmidi\, rr>FIo.rvedl)y p.nrn-r~egative Ixtcilli arid <,'rmrlirirr). arlrl l ; ~ i r pros1 hetic valve cnrlncardiris (LPLT). which is m;~nircstrdclirlici~lly more than liO rl:%1;~ a f i w ~ I V Prep8ac~tnenr(mosr rommonlv c a u ~ e r IJV l

Discussion

PROSTHETIC VALVE ENDOCARDITIS

ID/CC

A 25-ycal--old fcmalu complaiils uf low-pyadc fever aiid rnvalgia ol' 3 weeks' rturation.
Sl~c has a historv of rheumatic heart disease (RI-ID). One month agn, she ~iiic!el.went a dental extraction a~icl did not take 111e ail t ihioticv tha i were prescribed for her.
r )

HPI

PE

ITS: fccvel-. PE: pallor: small peripheral he~norrhages wit11 slight nodrilar character (,J,%NEWAY I.F:SIOYS) ; small, tender nod~lles on finger and toe pads (OSLER'S KODES); suhunpral lineal- streaks ( ~ I . I T ~ E I FIF:MORWI,KES); < pctcchiaI hcinorrhages on ct)njunctira. oral mucosii, and upper extrenritius; mild splenomcgaly: apical diastolic murmur on cardiovascular exarn; Sundur exan1 ~ h n w s oval retinal hemorrhages (ROTII'SSPOTS).
CBC;/Pl<S: normocvtic, normochromic anemia. LrA microscopic hcmatr~ria. Gramdl of penicillin-sensitivp-setS t r e p t o c n c m . ~ itiridans or\ five of six blood cultures. Er hn: ve~etations along atrial surfice of'mitral valve. F,rnbulism from vegetative growths on ~ x l v e s mav emholize peripherally left-qirlerl) or. 1 0 r he Iru15 (righl-sided).
Bactcria form 11irl11sof inrectinn in previouslv scari.erl or damaged valves: bacteria cli~idc unirnpeclcd once infection takes llold with further dcpnsitinn of fibrin and plalelets; peripheral y r ~ p t o nS ~U s C as ~ OSICT'S norles are believed to result Frmn depovitiorr of' immune cornplcxes.
W P-Tactamasc-resistant penicillin and gen ta~nicin; bacteriostatic treatments inefrective.

75

G1

Labs

Imaging

Gross Pathology

Micro Pathology

Treatment

Discussion

S . r~irirlnrtrif; ihe morr colnnlun cause of subac~~te infective endocarditis. while S/uj+)h~/i~ri~rru~ azrvms i~ thc most common cillisc of acute bacterial cndocarditis. PI-oplrylartican tihiolics shorilrl he given to all RHD patients hcfnrr, any dental procedure. TIIPrliseaw contirlties lo be associaled with a high rnortalitv rate.

"

SUBACUTE BACTERIAL ENDOCARDITIS

ID/CC

h .5l-ycar-c~lrl k m a l e L\,IICI underwent :I left mastectomy with axillary E y p h node dissection a veal- ago 111-euenls with pain toqcthrr wirh rapidly spl-ci~ding redness iuid swelling ni' Ihe left arm.

HPL

One vedr

as<), she wxs C I ~ ~ ~ I I O F P and CI carcinoma or die left breast.

O P C I . ~ I C I on ~

For StiIKr 1

PE

Lvft forearm swtdlun. indur;~trcl.pirik. and ~riar kedlx tender; c~~r.~-l\ing temprrat111-e r*;lisrrl:~n;~r.xEt~u and brn.tler* c ~ s fk i n I t ~ i o n ill tlc-tinct1 ar~cl not rlev,~ted( ~ u .rrvuiprlal;).
Necrll~ aspiration from :~rl\-dnci~lg 11,order o i '11ie Ierion. tulirn c;tainerl a i d r ~ i l t t ~ r cisoIxtrd d, P-hemolytic p u p A st rrp t ococcli?.
Pcnitilliuasc-1-rsis~ari t pr*nicilliit (i~;tfi.illin/r ~xaciIlin )
I:rlluliris if an acurc sprmding infcclion of 11zc rkirl I hat pl-~rlnminantl~ arfccts rlr.c.pr-~st~l>clr l a i ~ t c I~ i ~~ rIP. i; ~ ~ Group A streptococci and Staphylococcus auwtrs are t h r most common r-I ir,logic agents in a r l ~ ~ lH~IPP~O/J/ZJIII ~\: + I I I / J I ~ ~ Y L inkctior1 P i~ conltrlon i n cl~ilclrcrl.Patirnts with cht4unicveiiclu\ s~asis anr9 hmphtrlcnia uf ariy cause (I!mpl~oms. til:~r-iaris, poqr-rcgionill Ivmpl~ rlotlc tliswctim~, I-acliatinn ~herapvlarc prcrlisposud: r t ~ r e i ~ t lr ve . citrret~ saphrno~~.wrcin ~ clonu~-site ccll~ilitis was fi)l~nd tn he attiihutilblc to grulrp A, C:. r n - I: %tt-eptococri.

Labs

Treatment Discussion

CELLU L I T I S

ID/CC

A lti-yrar-c3lrl teenager prcsrnlts to t1lc. o ~ ~ t p a t i e clinic nt lt-ith a painful facial rash arlcI fever.
Oilc wccL ago, he p;lrient went on 3 calnpinq trip nnrl ~c.i-itrchc~rl lzis f i l c ~ on 40111~l o ~ v - l ~ ~ i11-uc n q I~ri~nchrs. Thcrr i.: no mrdical hisrol-v nf'clialnotr~. c;lrlcer. or r~lller chronic cou<liriorlu.

HPI

PE

1 ' 5 : fever (3C1.OqC) : lacliycartlin ( H f< 1 T O ) : BP 13ormal. PF: : erythematous, warm, plaque-like rash extending ;lcros\ cheek.: i ~ n rfkce l hilatcrally wi 111 sharp, distinct borders and facial swelling.
C:I<C:lerlkoqtosis will1 neutrophilia. ESK elevated.
Antibiotics with sulficier~r coverage fol- penicilli11;t~e-pr-orl~lcing S ~ W ~ ! I ! D C O Citlid C I I SS t a p h p l ~ ~ ~ ~ spp. t r c ~(ics. ~ .c,t k p l i ; ~ l r x;i ~ ~ ) analgesic.;/antipyetics:clevatt. t11r ; { i f ' r c t ~ ~ part l to rcr1uc.e wt-l l i ng.

Labs
Treatment

u
m
73

3 o g tl

<

Discussion

Erysipelas is an acutr i~itlarnrn;-lt iui~ of rhc wpurticial l:wel-5 or ihc corl~~r-ctixr li~~llt" o[' i the &in. ~ ~ s n a l011 l y 11ie ra(-e. alnlost ;ilw';lycca~isrrl 11y infection ~ v[I] i C;t.r)~1l2 .I S f w ptucoccris wliich is part nl' ~ ~ n r l n biic1cri;jl al ckin Ilora. Ri\k Ficl(~rs i ~ l c l t ~ any dr hrcaks in tllc &in or.lymphedema.

Atlas Link

lITm-1 MC-'136

ERYSIPELAS

IO/CC

X SO-~rar-old slaughterhouse worker prc.ie~it% wit11 a painful red swellkg of the index finger of his r i ~ thlland.

HPI

The swelling clcvelnped 4 d a y at rrt-t ~ was r injrmd with while slaughtering a pig.

knife

PE

Wll-defined , cxquisirelv iender, s l i ~ tlv h elera terl violaworts Iesion seen on rig-ht index finger; n o wppirration nt pled: right rpitmclllcar ant1 righr asillai-v lymphatlenopaihv notrrl.
Biupcy from udgc of lesion ?icIds E ~ s i p e l o t h r i x nttttsiofiathiae, a thin. pleornorplric, ~ ~ o n s p o ~ - u l a rrlicroaerophilic tin~, gramprlsitive rc~cl. Penicillin G or ripr-oflosaciiz in penicillin-allergic pa tien t
~ .

Labs

Treatment

Discussion

Erysipeloid refers to localized cellulitis, ~~sually ol' (he lingers : ~ n dI ~ n ( l scn~isetl , by E ~ s i p e l n t r hi x r f t ~ r s i o p a t h i a i ~~ :~fectio in i~ h11tnan5 i q n511allv the result or contact with infected animats or their products (often fish). Orpnisms gain entry via cuts ancI

ID/Ct

,4 10-year-oldmale cornpiains of a spreaclii-tgskin rash ancl painful svrlling of bo th wrists. Thc paticnl's mother slates that the msh l~egan with erythema of the cheeks ( " S L A P P E M : H ~ : F ~ . ~ P P T - ~ R ~ N ( : E " ) suhsequrntly p r o ~ r ~ s ~to r ci ln v ~ l v c the trunk m d limhs.
Erythematous lacy/rcticular skin rash involving fncr, trunk, and limbs; bilatpral .m~vllingand painr111restriction of movement a[ both wrist joints.

HPI

PE

Labs

Serology drtectg presence of specific IgM antibody to parvovirlls; A80 titer (to r ~ ~ 0112 l p a c u rheumatic ~ fever) n.nrm;~l; rheumatnid factor (to rule nllt rheumatoid arthritis) ncjiativc.
Self-lirniti11gcli~ease.

z
XI

3
P

Treatment
Discussion

L1

12 small (20- to PGnm), single-simnded DNA virus, parvovims BI9 causes erythema i n f r c t i o ~ ~ u (fifth n discasc.) in schoolchildr~ri, aplastic crises in persons wit11 ~inrle?-lyitlg Iieiiiol~lic disol-rlers ( e . ~ .sickle . cell anemia), chronic a m n i a i r l immunocoinpromised h o ~ t s ar~hl-aIgia/al-thritis , in ilnrrnal indivicluaIs. and fetal loss in pregnant women.
r _ l .MC-299

Atlas Link

*
-

ERYTHEMA INFECTIOSUM
- -

HPI

He t ~ l a s Iii~toi-v orin~ci.rriitrer~t Ii-nrr-al-;irle Tever., r i . r r l l ~ e ~ "nose ~r picking," nnrI pul-ulcnt rlischargc' froln his l r ~ i o n s I-lc . has nt, hicttrr.7 r,l henlatt~ria( d ~ w to inc~rasr.clrisk (if ~ ~ o ~ i t ~ f r ~ ~
glo~nel-r~lrint*pIu-iti'~;).

~ ~ o r o ~ c a l

PE

Ch:~r-i~c~cri\tic "honey-colored'* crusted lesions seen at mgle of mouth. ; i s ( )I li~rl nasal o l - i f l c . ~ ~ a11d . hrh ii1c1 C';ITS. Gram-positive cocci in chains (rr-RI P I ( )(.I ,r:c :I) i l l nrlrlition t r ) pus .cc.llu o n I.h-;~rns k i i l l (1I ' d i ~ ~ h i i ~ .pglt l ~r; n ~ ~ hs~ t r~ rp i tco r o c ~ i (y-oi~p :I L ~ I ~ C ~ ~ ~ C OII C I C hlnocl C ~ ) a p r cultlli-c: AS0 titer neg;lf ive.
Ervt ! i t ~ t n a t o lr<iot~s ~ ~ ~ sul-rnnnrling ~ l : ~ l u r a 01-ifice* l ruirfi whitish Inrnration.

Labs

Gross Pathology

nl- vcI1r)~'ich plu.~ilc.nie s ~ t d a ~ ;In(( e rl-list

Micro Pathology
Treatment

I n t l i ~ r r ~ r r ~ ;i ~ r fill^ ~ r ~ate r v nr PMNs wit11 v a r ~ i n g tIcgr~c.ts of iwcrr bqis. C;~pI~aloqp(>rin, penicillin. or C


I - ~ ~ ~ ~ I ~ ~ I il' T iille~.gir. ~ I V C ~ I I

Discussion

I nn>rti~n if :I highly cc>nlnlunic;lIde in r c c t i u ~ ~ disenrr s that is mo<l r,rtrn (.;ILIFCCI In- ~ r o i i p A ~ i r ~ ' p ~ o cC o I~ ~* T~ ~ ~ i ~,T ~F r i l n aI? l-i in p ~ . ~ v - ! ~ o f - ~ai~cl l c i - s ~ 111-~-cii\pc)s(t o ~ l ~ ~ ~ ~ ~ ~ i ~ 1~ t ~ l ~ ~ ~ i e ~ ~ l ~ r i ~ oct.111.u rnrw r o t n n ~ t ~ r l O ly I I 1ht2 !:re (13rrit1rhit;11 al-rrl). haiirlr, and ,II-I~. <T . ~ ~ ~ / ~ l (turmt,\ ~ ~ l i ITI:IY ~ r o r fr wxiht ~ ~ ~ (11. ~c-ausc I>II~!OIIS irirpctigo; ~ i - o ~R tp ~II-eplocnrra irnl~ctign l m;n bc stc-n it1

rrk

IMPETIGO

IDJCC

A 30-yra~-t ~lrlmale hcimr,st.sual iisirs his L~:,lrr~ily rlnctnlcoml~lainitlgof a no11prlu-i tic skin eruption rbn I I i.; upper limbs, trunk. arlrl anogenital area.
H 1 -

HPI

I I ~ C hren I HIV positive for ahnnr 3 ycai-5 dncl :tdmits r o having c n ~ ~ ~ i uunprocecrcd ~led intcrcourw.

PE

M ~ t l ~ i ppainless, lr pmrly-while. rlotne-\haped, w y , umbilicated nodules ' , to -5 mln in diameter o r 1 arms. trunk, anrl anogcr~ital area: palms and soles spared. Fii.ril. un~bilica~etl norlu2es coilraii~i~rg hick ~ ~ I I o ~ material. ~ish
Statncd histologic src tions confirnl diagrlosis wi lh large cytoplasmic incIusions ( h i nr-l.uscr;\r nor )rr;.s)in mairrial ~ - s l x - ~ wf c~ d- o m Ic.sions.
L e s i n n s mav resolve spontaneouslv or be rcmovcd In[ clrrrttnge. crvotl-lc~-apy, or pod opl~vllin; 110 ;in tiviral tlr.ug or wcci nr

Gross Pathology Micro Pathology

Treatment

iI\ail:lble.
Discussion

hlnll~~scarn coil tag.ios~inii s a I~enign, nutoinocnlahlc skin rlisrilse r>f r~hilclr (-11 mid yor111gacll~lts; il i s c a ~ i s ~ ~ d J aJ p o n i r ~ ~ (DN.4 s I ~ ~ L I S mnrl ) i~ transmitted th~-ougll sexual cr )nt;lct. clo.;e I>oclil y tcmlacl, clothing, nr towcls. T t i.; one uf niniiv opportitni?tic
infuclion\ seen in ATDS [yatien~s (dilticull to el-adirate).

Atlas Link

TTF PIC-143

P"b" MOLLUSCUM CONTAGIOSUM


-- .

ID/CC

A 30-year.01~1 black male presen 1s w i t h a n n n p r - ~ ~ I ir ci skin rash on the trunk. llpper arm, and neck.
T h e padent
iq

HPI
PE

othrrwise irl excellent health

Multiplc bypopigmented, scaling, confluent macules seen on trunk, upper arms, and neck; no sensory loss riemonstrated o v ~ r nreas of hypopigment atinn; Wood's lamp examination of uki n rnacz11e.idiqplays a pale yellow to blue-white fluorescence. Examinatioi~of KOH mounting of scales from lesions demun.;trates llie presence of short. ~liick, tailglcd h>.phae with clusrers of large. spllcrical btldrling yeast ct.11~ r5-i t11 characteristic "spa~hetti-and-meatballsv appearance.

Labs

Treatment

Topical.seleni~un sulfide: antifungal agents ~ u c h as miconazolc and clobimazole; oral itracor~azolein recalcitrant caseq.
Pityiasis versicrllor, whirl1 i q common in ynung acluIts, is a rclativcly asyinpto~natlc superficial skin infection caused hy the lipophilic fi~ngal organism Pityrospor~rmorbiculnre (also lrrrned !Ilolnrsmin~/ilrficr). The I ~ q i o nwhich ~, ucually have a follicular origin, are smalI. hypopigmc-rited-to-mn tnacuIes m'ith a hranlike scale; the maculcs are dis~rihuterl prcdomiuantly on areas of the upper trunk, neck, and shoulders.

Discussion

Atlas Links

I 'I. . r " I I M - M l - 0 1 2

b<l,I ' I ' 1 MC-148

PITYRPASIS VERSICOLOR

XD/CC

An 18-month-old male is brougl~t to the pediatrician f o l l o ~ t v i n ~ the ilppearancc of an cxtcnsivt rkin rash.
F o l u clay ago Iic suddenly cleveloped a very high fever (40C) with n o nt her s!?npto~ns or ~igns. Thc fwcr corltir~~ied For 4 clays untiI the, rlay of his admissiun, w h c ~ iit abr~rptly disappeared, coinciding with the onset o f the rash.

HPI

PE

Child looks well; in

acute dislr.es9:generalized rash apparent as rliqrl-rle Y-tn 5-111tn rose-pinkmacules and papules an trunk, neck, and exmmities (facr i s sparrd); lesions I ~ l a n c on l ~ presqurp; 1 2 0 Iy iipl~adenopat Ti?: spl~nomegaly: may also he present.
110

Labs

CBC/PBS: kI'8Cs vaiiahle: relative lynlphucytosis with atypical


I\Tnplloqtcs.

m 73 3
0
7

Treatment

Supportive: h c a r n e t .

D~SCUSS~O Roseola R infan tun. also caIlucl exanthem suhihun, i% ca~lqed hv human herpesvirus 6. 11 is 2 h r most conlmnrl exan tIle~natous rlicen';r it] itifan t~ 2 veal-< o f age nr vonngrr anrl is a frequcnt callse of febrile convulsions.

ROSEOLA INFANTUM

ID/CC

,\ 2-month-old rt>ti~ale i tlra~r t presents wit11 cst cnsh r bullae ;*tid I;~rgr.ai*e:ts OF dcr~~irlccl skin.

HPI

I !tlr rno~helhad suKt=recl Srnm staphylococcaI mastitis 1 week


:1go

PE

1%: fr.i,rr. PE: Iilrgc. areas of r ~ r lpain , fill, deli ~rrlerl <kin 011 perjn~~l~i~nl a n d p~tih11cca1 areas: flacrirl 1~i11Iac w i t h easy dislodpent of epidermis under pressure (NIIFDLSKI'S SIGN) :
r11~1tosi11 curface.; I;lrgrtv

l~nin~nlicrl.

Labs
Treatment

\:csicl~ I l ~ i i r lste1.i It=: . Y t r n ~ ) l t ~ / ~s ~rrtrrt>~~ ~ n r ~r~otl ~ r F~loorl culcurc.

IV penicillinace-resi~tai~t pcnicillin ( c . ~ .nafcillir~, . oxacillin l.


'Treat with CI-vthl-o~nycin if paticn~ is aIlcrgic lo prnicillir~.

Discussion

Scalded skin svnclromc i~ cal~sccl I>v thc csfoliatinq 'ffc-ct of staphy~ococcal exotoxin. T l ~ c ;~ctior of ~ t lir exotr lxin i\ lo

SCALDED SKIN SYNDROME

ID/CC

,I 30-year-tdd rnrtrl pi-esrn ~s \\ ith a I~ilnteraI red pruritic skin eruption i11 the groin area.
Bila~us;ll, c i d a r papulosquamous skin eruption on crythemarnus Iwqe wiih active. :~clvanciilg peripheral (serpiginous)border over SI r-nntnl and pcrincum .
Mirmscnpic cxamination reveals long scr~tale hyphae on KOH skin scrapirigs.

PE

Labs

Treatment

Tclpical :i~ltif~~ngal agen tx (Mhitficld's c~irlrment, rlotrimmole, n~ict~nazule) ; q y r l e n l i r therapy w i t h oral ~ i s ~ o r l ~ l v kutoconwole, in, nr i r r a c t ~ r ~ a z o lin e rrqirrant cases.
Tintha c r u r i ~ and tincii rot-porir; ( c , r ~ n u c ) KINT,WOI<M) ~ OCCLITsporadicallv: 7ii(hnj)E1y!n.n mthnrrn i s r l ~ e moqt commrm cause. T h e ir~fl:!mrr~atory form, which is l~suallv I o c ~ I ~ ~ P c to I the l i l ~ l k ~ , chch5t.0 1 hark, is rnmmonlv caused hv M i r r n ~ f ~ / ~ r rcrnis u m orT Y ~ ~ I U / Inwntnpoph~tps. I I ~ ~ O ? ? Rirlgwnrrr~of the scall~, knmm as tinea capitis, is cernlnonlp seen in children and is causcd bv T~-irJto/dt>/oti t o t / ~ w r t qn .

rn

Discussion

0 5

& t ,
<

Atlas Link

m MC-I51

ID/CC

A 2X-~~ear.nld male ptwents with a red, pruritic skin eruption oil hi5 trunk and his upper xnrl lawcr limhs or a Fr~v hours'
rl~~ra~ini~.

HPI

prcwt-ihed cnrrirnoxazo!e for a LTTI. Me hac i ~ o t esperienrerl any dyspnea.


CSiic- dav earlier. h e
~'ts

PE

Erythetnato~ts. warm, 11rticari;ilwl~eals (hives) 5ern over trztnk, leg%, anrl at-!]IS:t r o angit,eclema o r respiratory clistrcss.

Labs

C:BC: Ir11lcoc:~o~is ~bith ensinophilia. No parasites rcr~calud on


stool exam.

Gross Pathology

Linear or owT, raised papules or plaque-like wheals iip centi rneters in diameter.

tn

r~veial

Micro Pathology

Wide separation of clerinal collagetl filjers with clilatatior~ of Ir-mpha~irs and venttles. Topical age11t5 2 0 I.PC~IICC itching; avoidance of causativr agent (in this case, cotrirnox;lzoIr): antihistn~nines(primaiilv H I hlocker~ hut also I-I, blockers); consiclcr glt~cocorticoids.
Mast cells and harophils are Total to urticnrial reaction. Wlien

Treatment

Discussion

stimulated bv cert:iin immunologic or nonim~nut~c>logir mechanisms. stoi-age gr-all rile iri these cells relpaw histamine 2nd other ~netliittor~, SIICh as kinins atld Ie~tkotricnes, Thew agents product, the locnli7t.d vasoclilatatiorl and tratrqudntinu o C Ilt~icli Irxt chal-ac-rerize itrticaria.
Atlas Link

D L L . MC-021

URTICARIA

IQ/CC

A 7-vcar-old male is brought l o his farnili, phv~ician cr~mplaining of a thick yellowish dischzqe in his eyes that prevent5 him from opening his eyes in the morning: for the past iew davs, h i s eyes II:IVP heen blood-red, painful. arid watery. His eve pain ic exacrrhated ly exposure rn I i ~ h t (rt~r )ro~+~nrr~i).
Thrrt. of his classmates :1nd a nriglihur had a siniiT;~t epivdc ahout 7 days agr) (suggrsling a local epidemic nf such cases).

HPI

PE

L ' S :

11o

frrrr. PE: normal visual acuity: erythematolls palpehral

conjunctiva: w,-,rcery eyes: remains of thick mucus found clrl ir111rr

cantll:~larea: no cornra1 infiltl-ale on dit-lamp e a r n : normal antei.iot. chamher: mild prea~~ricl~lar lymphadenopathy.

Labs

Stainer1 rnt>juncti\~al smears 1.evea1lymphocytes. giant cell?. neulrophils, anrl l ~ a teria. c


Topical ail t i r~iiri.nhialeve rlsrjpq: ctlr~l cr)lnpr.pwes:t ~ ~ i r ~ i ~ n i z e rolltact with others in avnic2 sl~r.eatl: nvoirl rlqe n T topical stcrt)id pl-cparation~. a5 thew ran cxacerbatc hactcrial ancl viral uvc infcctio~~s.

Treatment

Discus~ion C;on,j~~ncti~itis i~ n common discasc of childhoocl tl1;it i.i mr~stly viral (adenovirus)and sel Flinri~ing; i t occiirr ill epidemics, nrld srcoilrlnl-v hact~rii~l i~if'ectiorls (sI;i~I~~lo anrl r~c qrreptococri) ~i mav re~lllt. l'is~ialacuirv is not xfferreri.

ACUTE CONJUNCTIVITIS

IQ/CC

A 3.-+~ear=nlrl woman cornpZains or ['PYCI- ancl p i n in the face ;rrlrl upper teeth (mrtuillnry sii~ris). c.sprcii~ll~) \\-Ililr I P X I I ~ I T ~

rf ir\v:lrci.
HPI
Shc 112s 11;rd a clir-rn~ic cougll, nasal congestion, and discharge Ibr- thr pas1 fPw m r ~ ths. n

RE

VS: fevcr. PE: halitosis: ~ c t n i s h - ? c . l l u w postnasal discharge: bilatrr-al baggy nasal mlrcosa; bilarernl p r r c l ~ s ~ i o tcnrlc1.nc.s~ n and erythema over zygomatic arch; clouding of sinuses by transillumination:cluntal and cranial nprve c,.iainF unrma!.
Nasal
c ~ ttl u r e ~ t*cveal S/rfl~/or.nrnr r J E I I P I ' III H O T I ~ ~ I ~ ~

Labs

Imagfng

CT, s i ~ ~ up ui: ~ - ~ ir>pacifir.;ttioii i~l of' rn;~sillary sintzs wit11 air-nilid lewl.
Ervtlicmatnus and rdcmarous ria?i;~l nlzlcosn.

Gross Pathology

Micro Pathology
Treatment
Discussion

Presrncc. of 01-pnisnrs a ~ ~Ieukocvies cl iu

lnrlcosa.

Oral dccongcslan ts: nrnoxicilli~~, Rac~r-im, 01. fluor-ocl ti no lone. Orher- p r h o g r n ~ inclucl~ r l t l ~ c strcptc>cocci, r Hr/f~rnrq')hi/r~+ in/lrrrnzl~e and ~ M n v n x ~ = Thc lh. ol~str~tclio (d r~ nu~ia in 111rarrtel-ioi. clhmoicl and rnicIdTe mc;ttal curnpltlx hy I-vtainerl S P C I - ~ t i o n ~ , rni~coqal erl~lna. o r polyps prc>rnolrli sinusitis. S t c t / ~ / ~ ~ / n r m r n ~ , + n/rrt,lr7 and gram-ncgaS,?zivc spsciec rrlit! cailw chr-OII ir sin usitis. Fungal siri~tsitis rnay mimic cllronic Ilacter-i;ll siii~mitis. C:rlntplicationq itjclr~cirr~it-ital: ~elll~li jlricl l i ~ xhsressec.

' ACUTE SINUSITIS

IDJCC

,\ 1 7-veai.-r,ld I)ny pi-esent\ wit11 itchy eyes, naml s t ~ ~ f f i n c s s . increased 1acrirn;ttion. sneezing,a tic1 a watery nasal discharge.

HPI

Mu 113s had similnr episotles in the pi191 that h;r~.e cnrre'ipc)ndcd tvith changing o f the seasons. His mother, ic known tn have hrrlt~chin l as1 hnln.
2 ' s 11 o ti.\cr. PE: pal 1 or: boggy nasal mucosa: naqal polyps present:
rnr;juncl
i\a

PE

congested: no c-uuclntc.

Labs

Conjunctival a n d nasal s l l l ~ i ~ dt.~norlsti l~ I I C Spt-evence of eosinophils; ] l o 1r:lcie ria c111 Cram c;tain: no neutmphils. Allcrgril skin ~ c x t < (scrlsilixrtl c u ~ t l ~ c rmast > ~ ~cr1l.i) s chew pcrui~ive ccnsi t i 1 . i ~ ~ .

GWSS Pathology
Micro Pathology

N a a l tntlrosa liy11ereniic nild s\vnllen with flllicl trxusl~dation.


Local tihuilc in fli~nimation;md cIv.;rrinction ot ~ipljei. ai rw;ly hrc;turt- OF t!pr 1. IgE-medint ed hvpersrr~ri rivif v i.t.uponse.
13ri1l derrmy=stant?ruitli inhxnac;~] ct~rtict>qter~r)ic?r; anti hiqt:irnintrs: intran;tu;~l cromr )l\.r~ horlii ~nr , esp~ciallv l~cCorc an~iripacerl contac~ with allcrgcbr~. AIIcrgic rhi11iti~ is cr~rr~rnonly c;ltt\rd h ! eupo51trr. to pollens, r111d1 C(JII~(~T ilrrcl I ~ , ii~rprr iiiatrel-: yiuptorns arc nlcbdiatetFbv ~ l i c rclcasc uf vasoncti~r and chemolac tic riirdi,~rc~rq from rnast cell4 ir~ld basoptlils ( r . ~hisla~nine .. atid Ir~lkotl-ieiles) with IgE
m
1

Z :

Treatment

5
0

Discussion

6 Gl

<

ph

A L L E R G I C RHINITIS (HAY FEVER)


-

IO/CC

h 2O-yeal.old male presents with a runny nose, nasal congestion,


sore rhmat, headache, and sneezing.

HPI PE

H e notcs that his wife currently has similar symptom?.


VS: mild fever. PE: rhinorrhea; cungeste~l and inflamed poslerior pharygeal wall; no lymphadenopathy.

Labs

Routine tests normal; rotttine throat w a h staining ~tncl culture nugative for barreria.

Gross Pathology

Nasal membranes edematow and erytbematous with water'v diqcharge. Monon~~clear inflammatiun of rn11cus;l;focal tlesquamation.
Svmptomatic.

Micro Pathology
Treatment
Discussion

Culdc occur 2 to 3 times ayear in thc avera9e person in the U~iited States: ihe prak incidence is in tho winter tnanths. Rhinoviruses account for the rnajoritrl of ciral URTF. followed coronaviruses. Spread act-111-s hy direct contact and I-espii-atnry drnple ts.

COMMON COLD ( V I R A L RESPIRATORY INFECTION)

ID/CC

-4 60-vcarald male presents with swelling and a vesia~lar skin en~ption on the left side of his face. Tlie patient rcports that before the rash rlrveloped, lie had sc-vere r a d i a t i ~ ~ pain g o r z the left side o f h i s fare. H e also I-ecalls Iiavil~g surered an attack of chickenpox during his childhood.
Unilateral vesicular rash over left fnrchcad and m ~ a bridge, l including the tip of t l ~ c nosc, indicating ir~volvcmer~t of the rrasociliary hranch OF ~ h trigerninal c nrr-ve (HI.I C H I N ~ O ~ ~SIGN); '.; ?kin nl' lids red and edematous; lit-lamp examination reveals nulnt=t.nIls ro~lnded 5pon rnt~~posed of minute white dots inimlv-ing el>ithelium and stroma, prod~~cing a coarse subepithelial punctate keratitis: cornea is kensithe.

HPI

PE

Micro Pathology

T~esicular skill l e s i o ~ ~ with s herpesvirus inclusions that asu intranuclear and acidophilic wi Ih a clear halo around them (Cowdry type A inclusion bodies); svncytial giant cells also Feen.
Z

Treatment

AqcFovir; steroids: cycloplegics. 'TrilluorotF~ymidine for HSV


keyat i t is.

1
( 3

--I

Discussion

Hurpc.; zostrr o p h t h a l a n i c ~ is~ caused ~ hy the varicella zoster virus, which causes chickenpox as a primary infccl ion. Zoster is helieved lo be a reactivation of the latent viral infection. In zoder ophihdmicu~, the chief focus of reactivation is the trigemha1 ganglion, from which the virus travels down one or inorr bmr~ches of the 0~11th-almic diviainn stlch illal i t s area of distrihutinn i s marked out by I-owsof'vesicles or scars left by t l ~ vesicles. r Ocular complications arisr during subsider~ce of the raslt ancl arc gcnrrally associated with involvernenl of thc nasociliary branch of the trigeminal ner-VP.

P 0
G I

<

E h l
+

HERPES ZOSTER OPHTHALMICUS

ID/CC

,411 l%veal--nld m;~lc complai~ls o l ' wvc'1.e irritation i l l the lel't cyc. blurred vision. exceqri~e lacrimation, and photophobia,
T-lr. reports tllat 1 - r ~ has I-rarl similar episodes j t~ thr past that were t r e a l r d witli ;in ;~ntit'ii.;ri clrttg. Hiq r r c r ~ ~ ~illrlicatc cls that Ile suLTel-ed rhe first attack at rlir agp o f 7. at which t i ~ n c hi\ cc>nrlirion !\*(IS diagrlo\prf it tic1 il-earrrl as a severe foUic111ar keratoconjunctivitis; h iq recnr-ds alsr 9 inrlica tc a l~istor*y of rccztrrcnt pis( ~ P CIFC herpes labialis.
'xnniinatioii of I r f t cvr r-e\*t:;~lq cii.cnmrn~~neaI congestion; Ill~c>recc ein ctainiiig o l ' cornea I-evral~ inli1tratc.s spreading i l l all rlil-ertions, r r ~ a t e w i ~ with ~ g each other and furlnitlg a large, shallow rdcer with crenated edges ("I IEXDRVI(. I 1 I:+ IZ") ; cornea is inqensitive.

HPI

PE

Labs

HSV-1 clcrnonstrntucl 0 1 1 imrr~rn~ofluoi-rsc'rr~t ~taitling of ~ p i t li i~ ~ lrcl-apin~s ~l as r z m e I 1 ill; i 11 tlze aqticouc Ill~rnoi:

Treatment

Trifluridine eye drops: ,~cvclrlvirh a s heen shoum t o rlccrc;~se i.~~riir~~t-i~~~e~.


Most ocr~lar hrrpctic ii-rf'ci.ctionsarc caused hy HSV-1. 1t is also llic prirnar\, cause OF coi~nral hlinrfiiel;~ it1 t h e United State?. l'rim,~l~v infrctionc prereni as uuilareral Sbllic~llar corljl~uctivil is, hlcpharitiq, al-corll~ill epithelial oplcitirs: recurrences rnay take tllc form of kwatitis (> 90$, <>IF casrv at-c ~ i ~ i i l ~ t ~blepharitis, ral}. (11.kefiltocc 111jurtcti\,iris. Branching dendritic ulcers, t~utwllv rlctcctc.cl b ~H . ~oI-cscri ~ti~illirlg. ~t i11.r v i ~ . ~ t l i i l lrliagi~ostic; v clecp s ~ r o m a involverr~er~~ l may r e ~ i ~ill lt ~cai.ring. rorncal thinning, ilrirl i ~ I > i llrtnal ( \,;1sr11lai.i7atir)11 with i.es~~tting hlinclrlt-ss clr

Discussion

HSV KERATITIS

ID/CC

,2 20-vear-olcl male swimmer complains of scrrcrc pain anrl itching in the righl car that is associ,atecl w i lh a d i ~ h alnnilnr t of ?rellowish ( 1 ' 1 - ~ R C I I FNT) discharge.

HPL

The patient has no previr )us hislr~ry or disclial-ge f ~ o m thc car and no history of asrociated dearness nl- tinnitus.
Rcd, swollrr~ area seen in right cxterrlal auditorv mealus that is partialtv obli trrating tllr I ~ ~ m emovement n; of t r a p is ruq~iisitrlp painful ( I-R-\~,AI. s~c;ru).
Gram stain uf aural s ~ t a ~lC~ ' V ~ A T S preSrTIce o f qr-am-negative rods:
clilr~~l isolates -e Pseudomonas

PE

Labs

aeruginosa.

Gross Pathology

Red. ,swollen area seen in cat~flagii~ous part of external auditory meat~ls; when ~isualizcd, tvmplllic rneml~f.;ir~c i';el-vthematous
and movcs nor~nally wit11 prleutnat ir ornscopv (vs. acute oti t i h media)

Treatment

Eardrops (rither a corril~ina~iot~ of polyn:-x?rin, neomvcirl, anrl h v t l r t ~ c n r ~ i sur ot~ nfloxacin); ~ gentle re~nowl of clcl~ris in ear:
O t i t i s ex ter-na is rrloni

\ 0
-0

I
---I

Discussion

cCltnmot1 in summer months and ic ~ l l o ~ ~ g h ~ tn arise from a change in the milicu of the exlerrlal a~~rlitorv w rrleatuq bv increased dkalizatiun and ex(-essiv~ moisture; this lead5 < to h ~ c r e ~overpowrh, rl rnnsi cotnmnnlv with gram-ncgalh o rods quch as P r m ~ d n t n u n(also ~ ~ causes malignant olil i s ercterna)and Prnlvus or Ei~rlgi such as rl.sj~u~gillrra.

>

O T I T I S EXTERNA

ID/CC

,4n lfi-mt)nth-old whitr fernale present5 with irritability together wi tll l~ilatcral, profi~se. and Fo11l-<welling ear disrhaqe of' 2 monll~a' durai ion.

HPE

The patient hacl re~urrent UREs la\t vvar, b111 hrt. nlnther clirl noL administer t h r cmi~plctr courqc of an~ibintics. The patient's mo!lickrl x t s a histol-y o C feerlirtg Iiev c11iEcl ~\.hilrlr~iilg down.
Rilatrr.al q t e n i s t ~ - w hlc i car clischarge: perforated tympanic membranes in a~~teroinfe~-ior quacl~tn t of I>oili C ~ T - Y diminished ; mobility nf tympanic membrane on pirrurnatic c)~oscopy.

PE

Labs

Gram-negative coccohacilli on Gram stain of dj~~11ill.g!? from ty1np;lr)ocenteqis: H I I P T ~ ~ ~ ) i/ n I~ , f!E ?~ .L ~I ~recn nxa~ on cullure.

Gross Pathology

Po\ciMc. cornpl icilrior~ s include ingrowth of squamutw cpitheliurn on upper middle ear ( r : ~ i c ) i I ; . ; [-FA I \) ii' Inngstancling: c o n d u c r i ~ ~ heating e loss: mn~roiditis:3 r d brain ;ll>~rec<.
H!ycl-rrnia ancl e d c ~ n a of inner ear ancl throat mricosa: hyprl-cmia of tympanic nlcrnbranu: deposition 01' cholcst~r-01 crystalu in kcr+atinizcd rpidcrra~oid cells in cholrslr;~ tvma.

Micro Pathology

Treatment

Kccp car clry: amaxicillin-cIan11anic acid; s u r ~ c a cI~-ain;jge l I'or sevtrc. nla3gi;i: nlx~ir~grlpl:~st):
Otitis media is the most common prdiatric l ~ ~ c i c r i , iitfertioii il and ir causer1 hv IIvc.hwirl~i(rrrdi, .C/u/)b?lnrnt-tr/~ nurrtlr. ancl k'ltah~wlln pv~~lrnrn? in i nnronatcs: ~ i n olrlcr childr-en i l is l l s ~ l a l l ~ cn~~rccl I>v pnrurnococcirs {.7/7Y~tur~rr(.r11r p?lr*r~morlicr~), H. irrl711mtrrr.

Discussion

* O T I T I S MEDIA

IIl/cc

A fiveari)ld male presents wit11 complaints oF a miId sore throat and eye irritation.

HPI

His mull1c.r s;~ysthal lie has spent hours at thc community swimming pooI t h i s ~ l u n m e r .
Milt L rhinopharyngitis; hilaleral conj~~nctivd congestion i v i ~ h
srxn? mucnid clischarge.

PE

Labs

Vir ;(I clrl tlirc rlf cor!jt~r~cl ival art r l nasopharvugcal sxral> yieltfq

adenovirus.
Treatment

Nn qpt~rific trcat~null t; sclf-lirrtitill# illnew.


Atlt.novir~~ i tlferti~111 < s OCCLIT most ertcn in infants and young

Discussion

children, rvho acquire the xirus IIT tllc respiratory or fecaI-ord rrjrlte. TIIE. most common respiratory ti-acl synrlt-o~ne in this age grnup i? mild corym with pli;~r-yngiris: in older children, tlrew svmptnms may hc accoinpat~ieci hv cnnjuurtivitis. May a150 cause l~enlorrhagic rystiric in chilclrrn. OII electi-on mirroscopy it is reel1 as ; I double-stranded nonenveloped DNA v i m s srsrro~u~ded by a 20-faccd icosal~rrll-al protein capsid from whicl~I f ? antennalike fiher5 or p ~ n c o n s e x t ~ n r radiaIly. l

PHARYNGITIS-ADEHOVIRUS

lD/CC

A 9-~e;~i.-old nlalc ct~mplaius of pain d u r i n ~ swallowing ( ( >mr\nrr~,~.lc:i i) fur 2 days, a c o o r ~ ~ pird a n b y nil~rclp acl~eq, hr:~d:irhe, a n d l'e~er.

HPI
PE

I-IU has uttlcrwi.;r h c c i ~ in xood health.


hrS: ii.\.er. PF:: n113tieratr rryt11r~na nl ptiat ynx; erlli~r-ged. erj~icrnatous tonsils rovered with white exudate; tcildcr r~~.vic.ilI a~~ei~o[x~tt~~~.

Labs

CBC :: ncutrophilic l u u k r ~ n ~ ~ o sStreptococcus iu. rill l l l I 7 ) 3 1 5 ~ l i l I ) i l 1 l ~ I~ l l l l l ~ e .

py ogenep isolated

Gross Pathology

I-lvprr~mia xi~rl welling oI' uppel- teqpiratol-v tract mucom: cryptic cnl;u-gc~nrn I o f ~.IIIIS wit11 ~ I S pi1ru1ri1tC - S L I C eii1:a~gemen ~~~~; 1 of regional lymph ilrldrs.
Act~te infl;~lnrrti~tnl-y response with p n l ~ m t ~ r l p l ~ o n ~ infiltrale, ~cl~i~r

Micro Pathology

h\rpt.r.emia a n d e d e m a wir 11 pus lormacion: hyp.purpla$ia of regional Ivniph tlodes: clilatntion of'sinrisoitls.
Treatment

Oral penicillin V.
Strcptococcnl pliii~~v~;llgit i~i~ an ilcu te h : ~ c t ~ r i a in l kctinn produced lw g~-ari~-posi~ i5.e cocci in chains ( . 5 f t q i o ( o r ( - 5) ~~ : pharrngitis is most crr~niiint~lv causuti bv k ~ o l A ~p .;trcptotucctls. Cor~~plicat iotw C ~ I I P lr, irr~rrlunc-iiiediafrd cr.osw-eaciil itv anrl rnnlerl~lar mimicking ma:, include glnmert~lo~iephr-iris and rheumatic revel:

Discussion

PHARYNGITIS-STREPTOCOCCAL

ID/CC

.\ SO-year-old female preseni c; to ~ h FrL T T ~ ~ = I ER C ; I ~colnplaining of ; I sr;thlli~ig light upper quadrant abclominal pain.

HPI

wl~o has Irren I . P T P ~ V ~ Itrr;t~rrle~l I~ I fur gon~coccal pelvic inflammatory disease.


SI-le is a prtjstitutc

PE

Right uppcr cltiatll-rilI t trnr1r1-ness;ccrvical rnnrinu tenrlurnrss ; t n d rn~~cr>l,~u-ulrnt cer~icitis fcwnd 0 1 1 peltir exam. Ccl-rical swab rraining :~nrl rult~~r idcntificv c Nrisseria gonorrJl woe.
VS: nn ecirlence of chcllcc~r;ti~is. Pri.iInneoscop!: prrsonce ol' ''violin string" adhesions between liver capsule and peritone~un.

Labs

Imaging

Gross Pathology
Treatment

.'\rllie~ion nrltccl ~ I_rut~'een liver r a p ~ l l l e ant1 pc.ritoti~uin.


,An~ihiotictherapy { c c f t r i a s o n ~ anrl doxvcvclinc) for pxiie!i I (and Tur pat-tr~eriI'warranted).

Discussion

Acrrte fibrinous perihepatitis (Fi 1-7-H I l ( ;I I-Ct I<TIS ~ I Y D R O M L ) occur? a s a complic;ttion r ~ gonococcal i and chlamydia1 pelvic inflammatory disease and c1inic:lllv rnirr\icq cholecystitis.

FITZ-HUGH-CURTIS

SYNDROME

ID/CC

r4 25-year-old male presents wir h s~~rlrlei-i-onsct, sewre vomiting. nausea, abdominal cramps. and diarrhea.

HPI

ML' hacl rctul-nccl home ahnut 2 hours after i~ttenclinga hirthclav


and milk were vesvecl in t arioti.: lol-rn\. The friend who tvi~s celebratit ig I 1 is l>ir.thdar:reported similar symptom*.
pilrtl at which meat

PE

VS: no fever. PE: mild c~rhydratio~i; diff~tsc nhclominal tendernrss: inrr~;~sctl hor\.el ~nilticls.
Toxigenic staphylacoccus rerovered from rr~lrrlring food. C:oagulasr-positive staphvlococcu~ cultured from nose of one of the cnoks a1 party.
Nn ITIL~C-OWT

Labs

Micre Pathology

Irsion~.

Treatment

Fluid and clcc ti-olvtr hnlnncc: antibiotics iio t indiciltctl.

D i ~ c ~ s f i o n4 C / ~ ~ j ) J ~ r l o t . ntr tr rt lm t ~l r rood 1>r,isoning result<Crom the ingcstir)n of fc~ocl containing preformed heat-stabIe enterntoxin B. Outhl-uaks of stapllylococcal rood pcliso~iing occur wherz food handlers whc 1 lime c c ~ n ~ a i i i i n xsupel-ficial (~d wo~uldu or who iirc qhedding iilfcctcd nns;ll dropluts inoculatr fborls w a f i as meat, rliiirv prc)tluuts, ~;ll;ld ~ I r c ' i ~ i rcrraiii ~ g ~ , sailces, and c~~starrl-fillcrI ~~;iulrirs. The incubation period range7 From 2 to 8 hours; thc

GASTROENTERITIS-STAPHYLOCOCCUS

AUREUS

ID/CC

I I-vcar-uld w h i t e male preseil ts with jaundice and dark yellow urine h a t 11;~s been prewnt I'or the lasl several clays.
H e also complaiils of naliqea, vomiting, a11d malaise. For the past 2 weeks. hc has hacl a Irw-gr-adc kver anrl mild xhrlorninal pain. He t-ecen tlv ~.eturnccl from a vacation i n Mexico, where he said he coilsuined ;I lot of shellfish.
Ir-terus: tender. firm hcpatr~nlegalv; no elidrncr of r p l r n m n e ~ d y or Frcc fluid in the peritoneal caviv.

HPI

PE

Labs

Direct hypcrhiIin~binemia; elei~atedwrum 11-ansaminaqeq (,\I ,T > XST) : n ~ c ~ d erely r a elecxtrd alkaline phosphatilse; prr rlrlllged PT; itlcrcasccl urinarv 1 u-obiiinogcn i ~ ~ l bilirubin; rl positive IgM antibody to hepatitis A (IEAV) iirrlicati~,eof active HAIT i ~ i f e c l i c ~ ~ ~ ,
May ot'ten appear nornlal.
Multifocal heparocellular nccrosiq wi 111 CounciImar~krorlies; Ivnrphorytic infiltratcs around necrotic Fc~ci: 10s.; r )E lobular
architecturc..

Gmss Pathology

Micro Pathology

Treatment

Suppoi-tive matlag-cment; pas.;ive vncri~iatiou availa?>le.

Discussion

w In hepatitis A infection, virus i s shrd 14 ra 21 davs beful-e the 0 onset ofjaundice; patients are no longer infectious '7 d a y after 2 m ~~irc~use~~f,ja~~nrlicc.I~isspr~eadl~yFecal-araltransmi~~ionauclis ; ; I rn e ~ ~ d e min i c ar-eac w l t ~ r e there arc contaminated water sources. 0 There i q no chronic carrier state: rerovcrp takes place it1 6 to G m 12 monll-ls. IC4V is a naked, singIc-stranded RNA virus of the < pasrirc immitt~iration picorna family, killecl vaccinr is :~vail;;ll~le: in the forin of irnmrinr seI.urn glohuliils is alscl available.

H E PATITIS A

A T~-~~;LI--oIcI malc iilrcli~.al qtuilcnt p r c ~ r ~ lwill1 r s jaundice anrl dark yellow urine.
Tic. aclmits tr) haying cxprricncccl ;tn at-cidental needle stick f i n o n t h s ago. which hc rlirl not rcpol-t. HC :ilsr.~ complain4 of nitilwa, I n t v ~ r a d e Ft.1 er, i ~ n d loss nf i ~ p p e t i t ~ .

PE
Labs

Ictr-rus; te11dr.1; firm hepatomegdy: n o evidcncc r ~ ascitcs f 01~l~l~nom~gnly


Direct hypesbilintbinemia: c l c ~ a ~ uscvlrm rl ~raris;~~nin;~~es (ALT > XST): ~uilrll!,el'vaterl alkaline pl~oqphartqse;HRsAg positive: 1gM anti-HRc positive (pr~.;~rr t cluring wi~idow pei.iod) .

Imaging
Gross Pathology

Micro Pathology

1,ivnhiclp\~, revrillc 11epitt ~ ~ ~ l l t I.i ~ l iil~ c r - n ~s \.i ir~ % Councilman l bodies m d I>alloor~ing rlcgcncrarion; inflammatiun of'port;tl ,trtas w.ith iufiltr;~lirmof moiic~nucleai~ ar119 ( m a l l I~n~phoc-yrrq, placiria cell.;. C O F ~ l ~ o p lils); l pron1i1i~1isr of K I I ~ Ccell5 ~P~ :1nc1 hilt rll~rrs: rholt-ctasic \\,irh h i l ~ plugs.
Suppor1i13r.r a w : fr~llnw I I 10 ~ determine conu1111edprescncc nf I-IRsAg Ibl-a r 1 e a ~6 t nmllths as sign of chronic hepatitis: c;~ccirle availnhlr Col-pl-evrntion.

Treatment

Discussion

Hepatitis B immune globulin plus hepatitis 5 vaccine ;Ire I-vcomiii~r~rlcrl for pat-en rial ( 11- rnt~cnsal.cupow re cr, hlood and T o r n~\%.hnru nF s I-IR4g-lmriti~c ~nothcrs. T h c infcctio~r is dividcrl into rhc prodromal. ictc~ic. and ~ O ~ l \ ~ d kphase\: ~ ~ ~ l l t 5 % proceed to chronic hepatitis lcith ~IICT.P;ISPC~ risk 1hr rir.1-hn5i.r; anrl hepatocellular carcinoma. r n likc hc.p:ltitis A. hcp~titih R Eias a long iracrzhr~tionpuriocl ( 3 rntllttl~s).Hrpatitis R vir-us is a n cii\ cl t-rpccl,pu-tiirlly circular DK.4 virus of t Iir hepadna t i i i~ ly ~ t h;%t cr ) i i taiil'c it DNX-d~pcndrii t DNA polvn~el-a\r. 1'Ilc ctoi~tinued prcscncc of HBsXg aftcr infection 11:1srlirlicallv ~-t~sr~lvc.d i ~ i t l i c a ~a rs c h ~ ~ n ncat-rirr ic .;late.

Atlas Links

pJ,

HEPATITIS B-ACUTE

ID/CC

jaundice o\.rr. r 1 - t ~ pas1

A 50-vtrar-oltl male is refi=rrerI 01. an evaluation or intermittent 2 \.ear.\.


Hc also complninx ordiari-hca. skin rash, ancl wuight loss. He receiver1 a blood trmsfiaion 3 w a r s ago. WIICII Ile was injul-etl in 21 nlotorq-clc accidcnt. Hr: rlrilic~ anr- TCT clr~rg usr. us ; I I I ~ history 01-nrri~.opcyct~ia~a-ic diwr-ders in h i s larnilv. Ictenrs: firm. tender hepatomeply: splenemcgaly; no c~idcncr. of ascitcs; n o klvscr-Flciscl~cr rings Found on dit-lamp cvamination (vs. ll'ilhr~n's rlihcast-) .

HPI

PE

Labs

Dirt-c~ h!71crl1iliru t>i~ic-rnii~: rn~kerlly raiser1 s r r l l r n transaminase levels; hepatitis B / m V ) serology negative;ennmc immunoassay of antihodiec tcl s t r u c t u ~ ~ ancl l nunstructural c n y n e protein4 nf hepatitis C (C200. C33c. Cl23-3) positive.
[>[I

Micro Pathology

livc-I-lliopsy, p r e s e n r e o f 'halloot~ing degeneration: h t t y

c h a ~ ~ g rportal s; innammation with RIXTOS~Sof hepatocytes

within parenchyna or i~nnlcdiatclv i~rljnctntto porAt;tl areas

( U r ~ ~ i NECROSIS"). ~ . ~ ~ - \ ~
Treatment
Discussion Rl havirin and n,,,-interfcron; supportive .cnan;~gerne~~ t.

Hepatitis C: I>eIongs lo I hc flavivirus Iirnily at~rEi q curt-entIy the ~llost irnpor~atl t cause of post-transrusion viral hepatitis: 90% of C ~ I S P Si~ivolve P C - T C I X ~ ~ ~ P O rransmi~<inn. IXS Greaier- rhail 50% or cases progrcss to chronic Irepatitic, leading t o cirrhosis in 20%.

Atlas Link

'

1-1

LIM-MI-031

HEPATITIS C-CHRONIC

ACTIVE

IDJCC

A 10-war-nlrl nlak rnrnplaitlli oI'genel-alizedweakncs~. faintness o r 1 cxcr-tirln, and r~ccasiorl;tl rpiFsrric pain.
Hici1ir1tlir.r-I~asrioricedrl~ai heof'trneat.;soilandnther

HPI

inedible things
PE
Labs

(PI(.\).

Pillor: plrffi. Licc and clcpcnclcnl crlurna.

CBC: rnicroqdc. hypochromic anemia: eosinophilia, Low serum iron and ferriiin; rle\,:3~ c t rerurn l I rai~cfrrrirr :r,rrlt~c~ I)C)IIP rl rslar-T-rnv Iicmo~iclcrin: hypoproteinernia: stool cxani rcr.caled egpr. of Anc?lostomn drrodenal~ (rwclirl eggs wmith rIi i l l tr ;lllsparrn t uhc-ll tliat r c ~ e a t l h e s c ~ r n c n ~ rc tr lnl~ryc ~virhin). ~
Albendazofe 01rnehenda7ole: iron supplementation to treat iron deliciencv anemia.
I ~ r k c t i o wit11 t ~ Jior~k~vorm<, ui111e~. ;1n($n\/nmn ~lllori,'?rnlr>ol"Cirntcrr r r r n r n m n ~i.; ~~ rnor-c, likel~ wl-tere invai I itill-v c.vi~ditions exist: inrlividuals at risk inchlrle chilrfl-en, gardeners. pl~unbcrs o r c1rctrici:tns who arc in contact wit11 wil, ~IIICI itr~~~ecI-Force~ prrsonrirl. Hr>okrv(>r.rr~ rggy cxcrrtrtl i l l [he ~ P C P C ha~cll ; i r l the s c d . rcfe;iqirlg larvae ~ h a ckvelop i i t 1 1 0 inrective Iarvae. I'err~1ta1lro115 larval p e n e r r a d t ) ~ is~rhc principal I-nodc of 111sn1;1n infrclion. Froin I lie skin, hookwr>r-mlarvar I t:wel via the hlrrodstresm 1 0 the I~uags. entev the nlveoli, a ~ c c n d tlru hrt~nc11c)trnchral bcc t o thc pharyns. ancl 31-t. wallor\.ed. A11 Eio11gh tr-irnsp~~lrnunitr!. I,ir\,al p;tm;lKe rnav 4 c i t a transien~ easinophiic pneumonitis (LCII I 1 1 , ~ rhr.1 ' s ~ I O S I T I S )this . pIlenomcnon is rri~rch Ic's\ c o r t ~ n l will1 ~ ~ i ~ W V ~ W O infections ~ T T ~ rl~;ln with r o ~ ~ t ~ r l \ t . o r m irzFttcliorlu. T l ~ tl-i;!jorr hralili impac~ of hook~vot-111 inrertinll, IIIIIVCI\YI; i~ i1-011 IOSS resulting li-om the 0.1 to 0.4 mL of hlood itrgc'~~trd claily hv cach ad111tI~OI-ln. 111 malnouiishcd I~osts, s~rclr hlot>rl Ioru rixri IracE I O severe iron deficiency anemia.

Treatment

Discussion

ID/CC

A I 4-i~car-olrl malnourished child diccl stlorl after- Iirlsjli t ali7ai io11 tl~ic. t r j a n extensive small bowel rupture and shock.
c-niergencv room with massive hloocly diarrhea. I-Ii5 l ~ i s t n r v at arlrnis5ian rcvcalrd t11c presence of' a h d n n ~ i n a pain, l revel; ant1 diarrllca of a few clitvs' c l ~ ~ r a i i oIr~ ~i :c %\niplorns1 ~ 1 rleruloprtl d after he ate leftover meat ar a Fasl-filutl rcsiaufiili t.
I-Ie tmrl
P I - P F P I I I P TO ~ rI1e

HA1

PE

H e was dph.i.rlrxrer1.pale. and hyporensive a t time or admissicrn a n d rlevrlopcd signs of pcritu11iti5anrl .;liock ~ f i r ~ l - hefnre ll~' hi5 dc;~tl~.
Cz~lurrcanrlrtsarric~fr~c.cm~ii.irrgir~tr~~ir~;~lle~ir~~~~i~r~laterl CIasfridium prrfringens type C ~ x - o r l ~ ~ cb ir ntg a roxin. ,4111rlpw ~ - ~ w ~ ~t l -t ~.~rp lt ~ u suiall c d in~c.;tint-. r r ~ t i c c ~~ i~ ~ll c ~ l x t i o n ~ . ancl p production in 11ic wiill.
M i c - r r ~ r c o ~exam ~ i c revealed necrnqis ;uiid : t u ~ ~ t infl;~rnrt~atit~n o in the ileum.

Labs

Gross Pathobgy

Micro PathoIogy

Treatment

Pictic-ilt died tIer;pit~ aggreruivc. flnid nucl elccti-olvle i . e [ ~ l a c ~ r r ~ ~ ! i t . hnlvet r l e c o t ~ ~ p r e s ~ i and n n , ;intibiotic therapy (ppi~irillin. clinclarnycin. or tfox\~cvrli I I Y ) : snl-gery Eiacl I3cc11 plannucl in view ol'rr~prlu-en C the r n ~ a l lhc)weI.

D i s c u s ~ i ~ nNct-rotizing cntrl-ocolitis i s n contlitio~l i~ffPcting prtnl-ly noul-ishcd lw~-snn$ ~t'hu c~ ~ ~ d d r n fi-ar;t l y o n )next (pifibel). It is assur.i;i~cd with Clostridi~rtn perfringcns type C ;uid beta enterntoxin; beta losin paralv~cs rFir villi a t ~ r ralluP5 l I'riahility and r ~ e c r o ~rij fs' the howel wall. Irr1tn1ini7:uio11 or cl~ilrh-c-n i r l Ncw Guinca with I~cln-tosaid wccinc hiis clc~m;~tic;~llv tierl-eased t h e inciclcncc or

Atlas Links

7 M-MI-033A, M-MI-033B, PG-MI-033

NECROGIZING ENTEROCOLITIS

ID/CC

:2 '~-vP:II--I>~c! tn;ll~ who hm I>ren hocpi tilli~ed for treatrncnl of acute lymphocytic leukemia rompln~ns of copious watery diarrhea. right lorvrl- qr~aclrant abdominal pain. and fever.

HPI

Hc was diagnouetl ;is neutropenic ( d u e l o aggreqsive rytotoxic cl~urr~r >tlirr-ap?.) it ti.w days aqo.
YS: fc\,cr; tach!*cardia; tachypr~ea. PE: pallor; ster-n;tl ienderness: ;~sill;~ry lr.rnphnclc~lop;~tll).: I l u p ~ t u s p l c r ~ o n i e g abdominal al~ tliqtrntion: morleratc dtIivt1nitian.
(:I$(:: sc-VPI-e neu~openia: aiwinia: thro~11hncvtnpeni;1. PBS and honc miu row x t ~ t d i r s stlggc~t Ilc is in rcrnis5iot1; T>louclc ~ ~ l t u r e grxxcs Clastridirrrn s e p t i c u ~ n .
CT, ;~f>rEon~en : thickening of cecal wall.

'PE

Labs

Imaging
tress Pathology

Muctnal ulcers and inflammation in ileocetral region of smarl intc~tinc.


:lggr'eli~Iursupportive measures: surgical interve~ltion: appropriate

Treatment

antibiotics (penicillin G, an~picillin, 01- clindaniyciri),


Discussion

Ne1u1-opcnicpi1 terncoliri~ iq a fidminant form of nccrotizing c nt c r i u ~ that occlm i t 1 ne~ltropeuic patients; neutropcnia is often rc.l;~tcdto cyclic nuutropcnia, Icu kerrlia. aptastic ;~nernia. or che~tin~liel-apv. In po~tmr)l-tern exam5 or padcnts w h o have clicd of lultkcmia. i n f ( * c t i o ~ or ~ stlte c e ~ arca ~ l (nrlrr.rr!s) arc f i - c q ~ ~ r n tfound; lv Chsdn'rli~~pn septinrm is the most curnmon n ~ . p i i i ~isnl:11~~1 m Iintn tl~e 1>?~1nd of ~ 1 1 ~ 6 patients. 1

lD/CC

.I %-vrnr+)lcl mxle romploins ~Trnidepipfric pain that t~su;~lFv Iwgins 1 io 2 hours after eating arlrl orr:lcionally arvakcns him nt niqlit.
T h r p;11 11,iu bee11 di;ignosrcl H irh duodenal ulcem several t i r n r 4 in t h ~I);IFI. ' ~ I I llis I symptoms have c-on~irtentIv recurrcd r v r n al'trl- I I I P I . ; 1vi~11 I ~ ~ IJ2 bloc ke1.5. ai~(a(:i<l\, and sucl-alratc.

HP I

PE

1 ' s : stable. I T : pxllr) I-: e p i p s r 1 . i ~ teltcIel.nes~ 0 1 1 clecp pal patioil.

Labs

CEC: norir~oc-vtir, noi,mr)chi-ornici~riernii~. Stclol positive for occt1It. ~)IOO(l. UCiI: ulccl-atiuns in : ~ n t r ~ ~ o rr In ' ~ t o ~ n a carid ! \ rltrocleniim: ar~traI 1)iopsy t;l>ecimcilr:yield positive ureaqe test.
(;rossl~,rnilnrl l ~ l c c r (lrl;~~ also . hi- o~mal) weiz 3s ~ l i a r p l ~ , p~inrliptl-0111 tlelcrt wit11 r-clatirTly straight wi11l.i ,111d sliglll rnvcrhanging OF 11zucos;lI ~n:i~-gi)l (hcaped-up rr~ilrgini 5 t l 1;imct r.ii.;tic n T ;I r i ~ i l l i g i t~leqic111) a ~ ~ : \ i i ~ ~ )11o it ~ n d clcan ulcer hast,.

Imaging

Gross Pathollogy

Micro Pathology

No e v i d ~ ~ of ~ rmalignxt~c\-. e antral 'biopsies rcvcal prcstnce of chronic rnucosal inflammation.


I 3

Treatment

Triple r h e r a p ~ with ;umoxicilIin, metr~~liiclazolc, and bismutl~ ~~tl~salicvla sliplr t c : tIlcrYll~y wit11 C~I-ith~lonzvciri, c~rnepra~r lle, anti t i ~ ~ i c I ; ~i~~ now o l ~ -cor~uiclerc~rl ~(Irrtivr anrl t - ~ l a t i v r l frrt. ~ or
~irle efrcctr.

> Y
0

;d
rn

z
7u
Q

n --I

Discussion

tielicobncter f i ~ l o rg i1-1j12,s OIJP t-I\ ing I tie 2 1 3trnI gastric I ~ L I C O S ~ I ~ cells; tOri, uof' Ilc;llthy i n t l i v i d ~ t i anrl ~l~ approxi~~~at 50% c l y of ~mticrlts wit11 peptic disrnsr. h;tr-l>r~r tl ~ i u t>rg;~rlis~n. "41 ~hough H. $q/!.lrr~ does nnt breach the epithelial barrier. c o l u ~ l i ; r ; ~ ~of ion r t i r anrr-al rnllroqal I:~yci-by this r,r~ani.iniis associated \rrith ~t1.11~t11ral altei.ations r,T the ga.;tric nltlcoqi1 a n d Iwnce wit11 a I1 iglt p r v i ;lIrll(.c of' an 1l-;i1 gp;a$tr.ili.;. I3rspi IP the h c l that 11. j ~ d o l i clorq not grow o n dr~nrlerlnl nlucosn. it is .;t~-onglv ;~s.iociatedwi~lz rl~lc~clcnal r ~ l c r r;, ~ n r cl-acIicatic)n l of' 11ieorgnnicm it1 patients wit11 rrfr actor). lreplic ~ ~ l c e tliseas~ tclecrpases the risk rrf rccun-r.11cc.
'_:1_[_L_ M-MI-035A. M-MI-035B, M-MI-035C, PG-MI-035

G,

<

Atlas Links

PEPTIC ULCER D I S E A S E ( H . P Y L O R I )

ED/CC

4-year-old male is hrought tu the physician hy hi5 parents. who rnmplain that the chiid t\;tr, had intense perimd itching, espcciaIlv during the night.
T h r child is otherwise Iical~Fir; anrl his d ~ v c I o ~ ~ ~ ~progress erltal is normal.

HPI

PE Labs

t'etianal excoriation noted.


Cellulom aclhcsive tape ~ e c t ~ r e to d pel-ianal area during the ti ight reveals presence of Enterobius vermic~cJaris e g F that were flattened on one side, were emhryonated, and had a thick shell; nn parasites fbund on SLUC)! exam.
Strict personal hygiene: drugs used include albcndazole, mebendazole, piperaxine, a~rclpyrantel parnoate.

Treatment

Discussion

Infrction is caitsrd bv E n t ~ m h i u s vermicalaris. Adult worms at-e located pri~rlarilvin I h r c ~ c a rrgion: l Female adult worms migrate ta the perianal area during the night and deposit their egv. Dircct pcrson-to-perso11 infection occurs by i n ~ c s t i o n and swallowing of e m ; autoinocdation occurs h?~ conmrnination rrC fingers. The l i f e cvcle is coinpleted in ahol~t 6 weekr.

PINWORM INFECTION

ID/CC

-2 10-month-old male prcsents with fiver a i d severe vomiting f'ollowcrl by watery diarrhea.

HPI
PE

1Iis stools are loose and watery wi.\?thout blood or mucus.

VS:frver; tacl~ycardia.PE: child i s E I - I - ~ L ~ ~ ~ P ; moderate


dehyrlrarion.

Labs

Abwnce of leri kocyte.; on frcaT .itdin: ri)ta.t.irus deiectecl wirh

FUSA, electron mimascopy wit11 negative ~caining identifies rotavirus on stool u1trafiItratc.s. Micro Pathology

Major histopatholug~c lcsions are char.acterizecl by reversible inmlveinenl of' ih e 111-oxi~na! slnall iH I P S C tie; ~ IIIUCOS~ re~nains intact with shnrttning of villi. a mixed inflammatory infiltration o E lamina pmpria, a n d hyperplasia of the mucosal crypt cells; elcctroi~ ~nicroscopy rm-cals distcndud cisttrrls of endop1;lsmic r e ticulu111,mi tochondrial swelling, and spal-se, i rreqllar nlicr.ovilli.
Flr~id replacement therapy.
Kntavir~~ pu nnp t Z is the single most important cause of e~idcmic, severe diarrheal alness i n infants and y o u n ~ children worldwide; it nrclll-s wit11 greater li.equencv during minter n~onths in temperate climates and during thc dry scason in tropical clim:~tcs.In the Uilitud Statcs, rr.>ta\-irusaccounts for ,5057 of ;ill ~hilclhood diarrheas, has a n incuhatinn period oC 48 hour-s, is transmitted by the fecal-oral route, and lasts only a fw d q s . Son~e children suhsrqurrt tly develop lactose in tnlel-an re, which Ifisis for a few weeks.

Treatment
Discussion

CI

%5

4 5~

g Z:

m I

G l

ROTAVIRUS D I A R R H E A I N INFANTS

ID/CC
HPI

I \

91j-\~e;11.c1ld rrlale presents n - i t t i quriden+)nsrt, c ~ u r r p y abdominal pain and diarrhea.

The diari-Ilea is watery anrl contains mucus. Thc. palie~ll ;tlso cct~nplainc c~f'lo~t~-glark k v r r with cliill~, rnalaiw. nall3c.a. and \ , ~ > nt li ng. i Careli~lhistory r e ~ e a l s t11;lr hc 1i:id i~igcsled partially cooked eggs at a poultry larm 24 h o ~ ~ lI3cfo1 - s u his \I rnplorns
I>l-&~!l.

PE

VS: k ~ e r tacf~ycarrli;~. : PE: rnilcl dif'fiise ahrlomiual tenderness; 111ilc1~leliv~lra~ion.


Sturd c ~ t l t ~ ~ >r ir u bd~ Salmonella tyfihimurium;stnillerl stool clemc~nstl-;~tec PMNs.

Labs

Gross Pathology

SII

Inte\~in;~I mucow1 p r y hrmx (linii~erl to thc colon) ancl some p~rliclal~llcel-s.

Micro Pathology

Mixed i ~aflammntol-v itlliltl-ate in Inrlcrbsa; ~npcrficial epitllelial


PI.( finn

nu.

Treatment

Fluirl a~lcl c1~ctst)Ivtc rcpl;lcorncu t t l~'r.;ipy: antibiotics withheld. ac I Eiev prolong carrier state. i2ntil)iotic thcmpv OIIIV for ~nalnnurished. sever-elv i l l . hactcrc,mic, :inrl sickle cc-ll disc;lse pnt ipnt5.
S;~lmonc=l I;[ in k c t ion i~ . ~ r q ~ ~ it r tll-ollgh - ~ d thc inguslivn of food (eggs, meat, poultry) ur 1vatr.r cr~r~t;iil~ii~;-~tecl with a n i ~ n a 01l liur~~ Scccs: a ~ ~ indi\;iclual< 1vit11 low gaqtric acidity are also

Discussion

PJ-

SALMONELLA FOOD P O I S O N I N G

IDJCC

;Z , S ~ ~ ' P : I ~ - O !alcohrjlic C~ white 111ale pi-csur~lc \,'it

I i fever, abdominal pain, anrl ri~pi(llypi.ogtesqivc rlistcntion (11' the ahrlnnicn.

HPI

H r was rli;ix~lns~cl wil I1 alcoholic cirrhosis L innn tE1 ;~gu. when Ilc JV;I\ ;~tliiiil t c ~m l the hospiral z v i ~ l jat~izdicc l and Itvtnatm~esis.
L'S: trlcr. PE: i c ~ c . r - ~ o~ n\ palpntion. : ;thrlonlit~id~rrlde!-ncsc witll

PE

gu:~~-clin~; flaid ~hrill anrl \liifting rIrilI~~ess to pl=~.c\r~cic~n (cluc tu =cites) : splenomeply: rlpr t.raserl 1~owt.I so\i1ld9.

Labs

CBC;: leukocytosis. Ascitic fl11irl Iruk~ tcyie coun L > 5OO/cc; PMN5 (S.iO/rr) clcvatrd; a s c i t i ~ I>r o ~ ~ i and n s gll~ct)ut. drpses~rcl: gr-am-l-iegalivebacilli i l l arciiir I711icl: E s c h ~ r i r h i aroll' icolatccl in c u l ~ r ~ relcx'il~ c ; r d ?\ST ancl ALT (XST> iZL1').

Imaging

Kt%: gt.nt~r~rl-glars hnrincss (rliie l o a~citcs) ; Ira r~virler~ce offrcc. ;lit: CS. ;it>rlnmcn:r:irrhu~ic s l ~ r u n k r n livcr: ascites; splenomegaly: increased portal vein diameter and flow. EGD:
rrt,pl~age;d~'ariccs.

Gross PathoZogy

Fihl-inr~l~m-ulmt cxurlillr r-orering surklrc ~>1'perirnneum: filx-oqis r31a-y 1c.d t o for.inatio11 o r adl~cqioiiq.
PMUF :tnrl lilxir~ 011 .;cr.c)s;~l s ~ ~ t i h c in c s varioll\ q l ~ ~ gwit11 rs pr-(3~encc r J y , r ~ n [ ~ l a ttissue i o ~ ~anrl hI>i-clriu.
CY

Micro Pathology

cn
{~uch

--I XI

Treatment

c>t,patlistn-sensitive- ailtil,ir>tirs or cmpiric her-,lpp a5 ccfrmsirne or @-lactamase-rcsist;m t pen ic'illin Tor


Rpccific

ywr r l - ~ ~ r ~ a d ac vrec ~ l ~ b;~c.ill ic i ;~tirl gran-r-p(>'i.i cclrr i: s~ippor-tivc rl-cnrtnrnr FC)I- cirr.lzosi5.

; o

rn z 4 rn

6
G)

<
Di~tussion TJlr spr,n tntleolls or prir~i,u,vTonn or prritor~itih occur\ i I I parim t> with adt ailccrl chrt~ilic livur di\eaw xtld coucnmit;~n t ~ t ' ~ ~ ) n d ipel-ironitis. ~r!' :i.ic.iles; I,,', rob is thc ~ ~ a o s Lr OIIII~IO~ C I; I I I ~ I '

SPONTANEOUS BACTERIAL PERITONITIS

ID/CC

A 25-ven1=nldrnale I1.S. citi7en on vacation in Mexico pl-esenh w it11 n hl-r~pt+n~e t cxplo~ive watery diarrhea, abdominal cramps. and a low-grade fever a i ~ d chills.

HPI

The patient does not complain of tenesmus (11. passage of hlood or rnlrclts in his tools, hut h r tIoths cr~znplainof'a ft.rIing olurgency to defecate.
1 % : low-grade fever: PE: ~inrernarkahlc.

'PE

Labs

Nn ervrhrocvtes. 12rSC.s. or pm-jl~ites Feeti in stained stool; hioa~~a Cor v~ enterntoxigei~ic E . ~ c h e r i ~ h coli i a (ETEC) rcrcal presence or the lal~iIe enterotoxin (LT) (tcsts a\aflahlr nnh. for rrsear ch purposes) .

Treatment

Fluid replareme~t; antihincics (fluoroquinolonc or TMP-SMX) wit11 lopera~nidc; prevention with carch~l hygicnic prarticeq and prophylactic nuoroquinolonc or hisrnuth snln:~lic~li~tc uith lo~c-mn~icle.
Traveler's diarrhea i~ a ~r.1~limitt.d condition that clrvc.lop~

Discussion

nithi11 1 to 2 days of ingestion of conli~mi~riltrd food or clt-inks. Over thr-ee-fourihs of' cases oj tr:weler's rlimrhea are caused by bacteria. with enterorosigenic E. trnli tli' mcl-t frrqiirnt c;ulse (may aI<r>be cai~srd I,! rt~~ernpathr>genic I.;. tali ancl, in Mesico, bv an enteroadherent 1:'. mli ) . Other commor~ pathugcns incl~idc SJ~ig~.l?n speuirs. C n m p l u b r i r l ~ r j t j ~ ~ ..lmnrrronn,r. ni. sppcies, Prv~ion~onrrs .shi~w!loidrs,8Sulrnrrri~llrr specie.;. and nuncholera vihrios. Rotavirus a n d Nnrwalk agent are the most co~nillon viral tnmorhrr / t i ~ l n ( ~ l i r n causrs: C i d rflin. $:ryI~t<>q/)~n'd?tr ??I, a n d rarelv, art. par;isiric patfio~ens. Enier-mtc~xis~ilir j:', roli proC111c~ euterotoxins r h i ~ thind to intestinal receptors and activate adenyl cyclase in thc inteqtinal cell ' t oprorluuc. an inc-1.c;lsr in the Fevrl o f r l cyclic ~ ~ n~~cleolirteq CAMP (ET, lahile toxin) and cGMP (ST. stable tuxin). which markcclly nugmcnts sodium. chloride, and watri. loss. il~rrrb! producing a secretory diarrhea.

TRAVELER'S DIARRHEA

ID/CC

X YO-year-old malr preserl rs wit11 sudden-nrlsct revet". colicky


abdominal pain, ancl watexy diarrhea.

HPI

He had eaten raw oysters a1 a friend's party thc day hefolr (incubation period -1 hours 10 4 clays).
VS: frvcr; taclipcardia. PE: nn dehydratiorl; cliffilse ahdornirlal tcntlcrnes~; increaser1 I~nwelsor~nds.
ViRrio parcrhaemo7yticus ivolatecl from stool in a high-saltcontcnt (l~alophilic rihl-in) culture muclir~m: PVNq in stool: Kanagawa phenomenon ( beta-hc~nulysis on merlirlm containing Iiurnan hlonrl; done as a11 irldicator rnr pathogenicity) positive.

PE

Labs

Treatment

Fluid anrl rlecmoly~e balance: antibiotics not requirecl (since thev do not shorten course of infect inn ) .
Seafood i~ the main source of the organism. After ing~srion.

Discussion

lrtlnio ~nrnhnernohtrms rnl~ltiplies i n thc gut and prodl ~ces a diarrhea1 enterotoxin.

VIBRIO PARAHAEMOLYTJCUS FOOD P O I S O N I N G

ID/CC

it 33-1eiw-c~ld malc prrscnts

to thc rnicul-KrHlc\.rooni with highgi-;~dc fclrr. n~arkrrl wea knew, a n d a he~nnrl-hagic vesi~~lob~~oas skin eruption.

HPI

I I c b harl~j~r~trct~~r-nccEfi-c~nldc.t.p.icali~l~in~inll1~G~~1Tnf

Mrxico, ~ t l i e r r h o Ii;~d ror~slriii~rrl large qt~mntitiel; oCseafood. He l~ns IICPII cliagnnsed with chronic liver disease (duc to l~rmoch~.om;zrt~sic).

Labs

Rloocl c u l t ~ ~ lnn - c high-salt medirrm (Ilalaphilic ihact trf:~)I-~YP;II< ~i'cr\rlT~ (11 Vihrio rrtrlnificlds; e y i d ~ i i c o~ I'~i~t~~~~rh!-~inalr~~i~ Il~r-pcrglvcurni;i.11~~~er.hili1-uhi1~t'111ia. iircreac~rlwr.lLrn iron). Ceftzidimeatitldouy~cline.ciprofloxacin;suppc~rriuc.

Treatment

Discussion

H;tlophilic. 1 i'brir~ 7~7rlniJinr r, sho~ilcl hc, 51~ ~ ) V T mlri I PtC ~e Ii l t e i11 ~ l ilnV inrlit iclt I;II with clii.nn ir liver di.;e;~se w h o presents with stpticcrni;\ anrl skin Iecions 1 to 3 clavc lirllrnciilq ccafoncl iiijicstisr~.

V I B R I O VULNIFICUS FOOD POISONING

lD/CC

i\ 5(i-vearUolrl white male complairls of' diarrhea and I-hoating for several months iilnng ankle swelling.
t[e also ct~mplains of meinosy loss. fcver, arthritis in thc knees ai~d hanrls. aild weight loss.

MPI

RE

VS: fiver. PK: thin, g a ~ m r~lale: t m~rwls w ~ s t i ~ r swollen, g: tcndelrig11 I ~zrristaticl anklu: asillary ailil Te1lio1-alIvrnphadenopathy: ~c 1 tvmctsrs of' chcqt i111d;jr-~~ls.

Labs

CBC;/PRS: tnacmcyti c. I1ypoch1-arn ir anemia: l-iypc~;~ll~urni~~c~nix: incseascd fecal fat (st~'n1c)rdiraj. !IC;I/SBI;T: nonqperific rlilatatior~ r ~ small f hnwl.
.%trophvor ir~lesrinalmucosn: inflanlr~~xtory infiltrate in synovia
o r j r ~ i 13. n

Imaging

Gross Pathology

Micro Pathology

Sln;~ll l~oweI I~iop~v reveal%characterktic macrophages con tainirrg 1,:1cilli wit11 PAS sragellt shining: cllin-ilcte~.istic gram-nevi ivc acrinonlvcrte hrtcilli in ~nacropli:~gr 5 , PMNs, ancf q ~ i t h c l i accllr l of lamina propria: c1il;l~e.dlymphatics: flattenitlg ol' intestinal ~illi. Rnciritn (TxlP-SMX)
rlt.

Treatment
Discussion

cel'rriaxonc rot- 1 war.

Caused b y ir~f'rc tion with Troph e ~ m w nh i p p e l i i : p ~ . n r l ~ ~ c e s malabsorption nf f i j t - ~ r > l ~ ~vitalnius, t>l~ protein, iron, foIic acid, and viti~nlinR,?.

WHIPPLE'S DISEASE

It'D/CC

.A 28-vear+lrl Ternale romplxi nq of painful swelling of both knees and tencler skin er~~ptiens o n hot11 shins.
Fr~rthe past 2 wccks she has also had watery diarrhea that rleveloperl after she c o n u ~ m e d some saw pork. She also rnr~~l>lait~l; o f int\yg~-acl~ fever and inilrl :iI>rlotniui~l pain.

HPI

PE

V,C:low-grarle Te~rr: tacl~ycarrlia.PE: mild dchytlration; swolIen and l\rarm knee,joincs with painfilI restriction of all mo~~cmerlts I,\K rt IRITIS) : mu1tipic. tender, erythematow plaques and nodules (E I ~ T F I F M\ ~ o r > o % t , h iseen ) over IX>I 11 4 1 i i i q .
CIEC:: 1r11 ~ ~ C ~ I O FY~rsinia ~ S . ~ ~ t t r m c o l i t i iwlatcrl ca tiorn slool; patient is F U - B 2 7 positive.
Oimal
ft)
~ I ~ C P wit11 ~ S 1 c ~ s - iaxis ~ i l l rhe rlirection of horvel flow, similar ulcers r a ~ ~ ~hy e rh-phoicl l fever (intcstina1 tuhcrcu1:~riilcrrs

Labs

Micro Pathology

are ~ranwerse).

Treatment

S t ~ p p o r ~ ia11 ~~ tihiot e : its (att~it~ogIyro~ic~es, flunro~l~inolones) indicated in severe inrections.

(91 r f ~ O l ' l l / j / j ~ ' is ( l a11~ T I V ~ S ~ai.a!n-negative VC intracelldar D ~ S C U S S ~ ISs\ivin O~ pathogen I hat rallqeq pqtrnentcritis, nlnst fi-eq~~en tly irwolving t liu d i ~ t n l ilr~~n ;~ i nd colon ( cn lcrotosin nludiatrd) , mesenteric adenitis ( r l t l e In t>~r1.01i7it1g ~111~ ~t111p11ra1i1.e 1 gilt l e q i c ) ~ and ~~) ileitis (pseudoappendicitis),ancl ~cpsis; ir~fectio may ~~ trig~rr a teariety of' autoimmune phenomena. including erytl~erna r~c)dc)s~~in, rrac.1ivr a]-thri~ is, and porcihlv Gmvrc' rliqeaw, cspccially in I ILY-827-positive i ndirirl-t~al~. Sprcad is by thc kcal+r;ll n m ~ and c occurs via containin;~tedmilk pr.c)clr~cts or

YYERSINIAENTEROCOLITIS

lD/CC

,A 3-vear-nlrl albino male is referr-ecl to a specialist for-;In e ~ a l t ~ a ~ iof n t; ll suspucrcd irnint~ne drficivncy.
Hi.: paretlts report reowrent respiratory. skin, a i d 01-a1 infections with pun-negative and gram-positive o tgan is rns. I-Ic alscs h;+ca h i ~ t o r y of bruising easily.

HPI

PE

Partial albinism: liglil-brown hair with silver-v tint: nystagmus; photophobia at-t eye rcflcs exam; chronic fiingi\itis and per.iorlr)11t itis: pwpuric patcheq over areas of rcpeated minimal trauma: inild hepatomegaly; no Ivmphaclenopath?: CRC/PBS: decreased neutrophil count ivi 111 nor-ma1 platclc t c(mnt: large cytoplasmic v u f e s (r:rn;\r I z.L.\( )S(M~FS) in WRCs on M'yight-stained pcriphei-al bloc )d smears. Prolorigrd bleeding time: impaired platclef aggregatinn: r~rmtlaI cloiling rime <uld PTT: normal 11i truhl lie 1~11-a7nIi~irn test.
Largelv suppc~i-t ive: ascorhic acid. prnpl~vlacticanti bimics, ac\~~lol-ir.

Labs

Treatment

Discussion

ChPrliak-Higashi syn drumc is ari autosamal-recessive rlisordelhat is clue to a defect i n polymerization o f microtub~des in leukocytes illat causes impairrrlent ot chemntztxis, phagocv~nsif, ancl f c li~rnatioiior phagolysosomes. Pal ients with this disorcler I tsual l y [wesen1 with remrrent pyogenic staphyIocnccal and
streptococcal infections.

ID/CC

hri

8-year-ald child tvitll sickle cell anemia i u w e n with cr~r~lplninili 01' s~~rlden-onwi pallor of the skin and rnnctlu?
r n e t n h r a l ~ ~ lI; '. a t i ~ ~ a11cI t ~ , inalair~.

HPI

The. ctrilcl suf'fi.rcrl a mild prodromal illness belnl-P d e w l o p i n g srvcrr pallor.


VS: 11c> fe\*er:lac humi-tlia: t a c l - i ~ p n ~RP a : nor-ma]. PE: scircrc ~ ~ a l l n mild l - ; irterus; no I ~ n i p h n d e n o p a t h ~ splcnomegalv, . or hrpitonicgr11~ nr)tcd.

PE

Pa bs

CR(:: severe anemia ( H h 2 %/dl.,);reduced letlkocyte and plat eler cnlm ts; rn ild hvperhilil-ltbinemia: absent reticrrlocytes :111cl G--klerl RBCs on per-iptler-a1 hlnod smear.
Bonr inarr o~ hiop<! r-evrals i n c i - e a ~ r r~runl>ers l of' giant pronormohfasts (rliagnnrric nt pal-vovir~~s infection).
Bloocl trnusf~t.;ionsto t idc c~vui[ h e crisrs. S p n t ~ r a n ~ o I-ecnverv us ill 1 io 2 weeks.

Micro Pathology

Treatment

Discussion

l';~rvmiru.;infcctinil is U1c cnusr of transient aplastic d s e s (may ;11w h r dilr' IO frtlic acid rlc~firiencv)thar occur in patiellt? who hmc wve1.e hcmolytic disorders; cessatir~r~ of c i . t ~ h r o p r ~ i r for ~is i~bout 10 clays in :I 110rlnaI aclult xq a result uf parvovjr~is irfcctior~ woulr! producr ;I 10% r l n ~ p i n Ilcrnoglrrbin c o ~ ~ c e i l l n t i ( ni ~t l. a, fall nf IY O il;~il\~ tvor~lrlleati to a rlecline in liernng!nhin conccutriition of t t o 2 g/dL ;tftc.r 10 day.;). 1 2 patient with seterP liemolysis in w h o m i h e hclnr mai.rrlw i s t ~ ~ r n i lover l g at a ralc scvcn times normal wo~tlcl rxpel-iencc a 70% dccrt.:isr* in 1ic.inoglohii1 concc.ritrali(>n(i.e.. ;I drop l'rorn I O f i J r i r . 1 0 3 g/dI,) xs a rrsltlt o l a 1O k h v crs~ation or el-vtIlrt~poieqiq. AIttrrn~ghparvo~irrr~ can ;tSfcc-c.cr;ill pl-ecur\c~r. tell.;, I h e i w l cell prc-c-11rs01.care [no<{

psofbundly affected.

I A

AN EMIA-APLASTIC

CRISIS (PARVOVIRUS B l 9 )

ID/CC

2 2 3 . 7 - v r a ~ l r Finnish l man complaint r h easy Fatiphilily and shortness of breath.

HPI

H r trften eats undercooked or raw freshwater fish. He also


rrports \agllr tligt- ti\-r cliutur.bancr~~ s11c11 as anorrxta. he.;ll-th~~rn. and nausea.

PE Labs

PE: pallor.
(:RI :/PRS: mega1ohlastic anemia. Rlr )or1 vitamin BIZ IeveIs low: siool r u m revr;~l.; pr-esencr nf operculated e g b and prodoftids o f Diphyflohothrizrm latum.

Treatment

Niclosamide (31- praziq~i;tn tel . ~ i j l l ~ ~ l i r ~ b~ o I i IInf71rn lr~i~ (fish ~ rap~wrm) i nfertion i~ found in colrl climate? whew raw or undercooked fish are eatrn. Thc adult worm a ~ t a c l l e to ~ t h e human .jr$umum aancl competes for ahsorption o f vitamin R,,. prt~ducii~g a cleficier~cv zlial s e s ~ m h l e s p ~ r n i c i o u anetnia. s Preveni icln incllldrs pl-riper preparation of

Discussion

A N EMIA-DIPHYLLOBOTHRIUM
-

LATUM

ID/CC

iZ 4:byear-otd male with refrxctory acute myeloid leukemia w l ~ o unrlerwetit a bane marrow transplant frnn~ ; I noniclcntical rlonr~rpr r s m t 5 ~ vt fi i XII extensive skin rash, sever r diarrhea, anrl jaundice.

HPI

Prior to t h r transpbn t. which o c c ~ ~ r r e 2d months ago, hc received preparative chemotherapy and radiotherapy along wirh I>~.U~I~-.~~ antibiorics. ~C~I'IIT Engi.aftrnen I~ t rv;~s confirmed within 4 weeks bv rising Icukocyte counts.

PE 1%: BP normal. PE: patient is c a c h ~ c t i c and modcratelv


clehytIratrrl: icterus noted; vi,iolaccous. scaly nlaculcs and i ~ r y t l r e ~ n a t r papi~les s~~f resembling lichen planus I;em over uslrcn~itics. Labs

C:RC:: hilling blootl cuurits: I-elativc cosi~lopl~ilia. Elrcatrrl direct srrum bilirubin a11d tr;dns;urli~lasc-s: stuol C = > E ~ IT I I~ * V ~ ~1I1I0S
iilfectiou~ e~iolopk : i n hiop~y lnken.

Gross Pathology

Skin biopsy specimens rcvcal r~acuolaic hr4ngc.s (6 bi~valtell laveiw i t f i pvri\c~iuFi~r Ivmphuc-vtio inf Itrates (I:DX+ T cells).

Treatment

High-dose cyclosporjne tl~ecapy. I-irhbi t ;~i~~i-thynt~ glo1~1 c v t e~ l i n , ~ n ~ . ~ I i y l p l - e r l n i sor n l an1 c~~ i-TxelI ~~ ~~lnntrclonal antihndies. Approxi tnately 30% sf'honc marrow trangplan t rccipicnls rlevrlop gsafra.~r.r~~~-I-hc l q t disrase (G\WD) , T l i i c ar rack is prirnal-ilv I:u~ncherlhv i i m m ~ m n c o i l ~ p ~ tT en IyrnpI~r~cvtc~ t derived Crom tlic donor'? InilrrnW agninst the crllr 2nd tis~ues of ~ h I-or-ipiuril, c

Discussion

'

CRAFT-VERSUS-HOST D I S E A S E

ID/CC

1-2 20-vt.ar.nld inalp preqents with an cx~encive purpuric skin rash, oliguria. and marked zvcakness; he also complains of bloody
diarrhea of 1 week's duralion.

HPI

The p a t i ~ n ace t a bamb~~rger at a fast-Cood rrstaur;lllL Z! to 3 days prior to the onset or hi5 dial-rhea. T-JP has no awociatrd i'ev~r.
VS: n o Fever. PF,: de11yrlr;ltion;pallor; rxtunsive p n r p t ~ r i c ckin mqh.
Stotrl uxaminatiorl reveal5 presence or RBCs hut no inflammatory cells or parasi1t.s: C L I ~ ~ Li~c~lates I~P ~OI-bitol-11egaliw Excherich ia coli: wrolyping s n d i e s and crfcct on HeLa cell culture reveal presenre o r enterohemorrhagic B. cnli (EHEC) serotype 0157:H7: ulevated RLTN and creatini~ze. CBCJPBS:rnicroangiopathic anemia and c h rom bncytnpenia. PT. P I T nor-nlal.

PE

Labs

Imaging

Sigmnidoscopy: n~oderatelv h?;pcrcmic rnlicoua tvirh n o eviclcnce


ui anv ulccratian.

Micro Pathology

ParF~nlop 1ocali;l~rl to kidney. w h c r ~ Inaline thrombi WPI.F: seen in afferent arterioles ancl g1omcruliu-capilla~ics.
Dialysis alld blond ~ransrusian for managt-mcnt uf HUS: fluid and ~ 1 e c t r ~ ~ maintenance; ly.t~ atitimici+obiaItfirrapv. Most paticilts whv dcrclop I-IUS as a complica tioil of K. (rjli hemnrrhagic colitis die as a result of 1re1110r.rhagir complications.

Treatment

Discussion

l3ernorrh;lgic colitis assoriared with a Shiga-likc toxin p r o t l ~ ~ c i n g EHEC 0157:H7 is ccharacterizccl by grossly blncldy dial-rhea tvivitll rcmarkat>lv little rever or inffa~nmazory exudate in smnl; a significant n ~ ~ m buf e rpatients deve1op pr>te~~tiallv fatal I-IL'S. EHEC Tnfet tiot15 can he largely prevented through adequate cooking of beef, rspecialIy I~aml>llrgers. ElIlY!E R-Eli-049

Atlas Link

HEMOLYTIC-UREMIC SYNDROME (HUS)

ID/Ct A 34-vcar-old male prescnts to his primary care physician with a


h;trrI. I-rrl. painFess nveUig on his lefi mandible t h a l has slov~ly l~een w ) w i n g over. tlre paat few tverks nrlrI haq nnw h e ~ l n to drain PIIS.
HPI

The pariuilt recently had a tooth extraction.

Labs

Gram stain of cxltdatc re~cals branchimg gram-positive filamentanrl rharacreristir "srrlfur ~granulcs": nc~rl-arirl-last and anaerobic (di.;tinprisher actinnmvccs Froin ~lrornrdin).

Imaging
Gross Pathology

XR: n o l,r)nv dcstr~tcti on.


Sinus tract! front region nf infection tn wrCace with granular exuclatc.

Micro Pathology

'r;lnulation tissrle ancl fibrosis sur.rnuncfin,qa c~1itri11 s~~ppl~l-arivr ueri.oqis; granl~latio~i risque inn? a l w enclose foamy histiocytcs xnd plasma rclls. Xrrlpicillir~ follotvctl 1)y a ~ r ~ o ~ i c or i l lpenicillin i~~ C ; Enllowerl hy oral peniril l in V and, if neressarr: s11rgIca1 rlrainagr: and rumolal of' nccl-otic tissuc.
rlrtirloni?~~~ rrrorlii is a part of thc normal flora. of llie mouth (cl-vpt.;(~I'tonsils and tart:lr of teeth), SO most pa~ientqhave a h is1 or'y of MII-%el-y or t rallma. T h p t ~ i s no person-to-person spread. Actinomycosis i5 n chrorric srippurativr ir~frctiorland nln alst) involve IIIP aktlornen or lungs, especially LblFowin~ a ponetratirl~ m u m a uitch as a bllllet wcutnd or an intestinal pert'or.atirm. P ~ l v i rlisease r i s associa~erl with I t'D u w . Spread occurs coi~tiput>uslv, not hcrnatog~nolislv.
1-1

Treatment

Discu55ion

Atlas Link

! I i M-HI-050

ID/CC

; I 7-month-old girl is hl-or~ght to t l ~ c pediatric clinic with wheezing, rrspira toi-v di lFicz 11ty. anrl nasal congestion of 3 110w%' duration.
She Iixs liar1 rhi11orrhc;i. frvcr. and cough and had l~een s ~ ~ i : e / i rf ~q r 2 davs prior to hrl- visit to t h e clinic.
VS: tachypnea. PE: nasal flaring; mild cvutral cyanosis; arcessot-y muscle use during rcspirirliou; h!'petrxpansion of chesl; e ~ p i r a t o rand ? ~ illspir-at orv wli~ezes; rhoncli ovul- I>othlung lit-Ids.

HPI

u
Z
-rl
r) rn

PE

c
yr

Labs

C;IEC:/PI3S: rclativc lyrnphacytosis. ARCS: hypoxemia with mild hypercapnia. Respiratory syncytial virus (RSV) dornr)n~lratect on viral cz~lture clf throat swab. CXR: hyperinflation; sepncntal atelectasis; interstitial infiltrates.
Sevcrr hrollchiolilis prorluces hrnnrhioIar cpithclial necrosis, lynlplio~vtic infiltrate. ancl alvcolar t-xudates. ti~uniclificcloxyg-ccn, Imonchodil;lto~-s, arl-osnlized ribaviri.

a m

Imaging
Micro Pathology

Treatment

Discussion

R5V is the most conimon cause of hmnchioIicjs I n infants under I? years or age; othcr \i]alcauses inclr l d pxrainnuunz;~, ~ inlluc~lza, a n d adcnovi1.u~. RSZ" <heclrliligmay last 2 or mc~l-e
weeks in ctiildren.

ID/CC

An %yeweldfemale presents with pain and swelling of hci- knee joints. elhow5, and lower limbs along with fever for ihe past 2 wcrks: slirr also co~nplainx of sl~urtness of breath ( I ) Y ~ N I ~ % On ) exertion.

HPI
PE

The palie111 hati a sore t h a t 2 weeks ago. VS: fc~cer: PE: blanching. ring-shaped erythemato~ls rash over trunk and proximal extremities (FR\TFIT;M,I LMRT.IN.ITL~KI); subcutaneous nodules at occiput and l>clcnv extensor tundolls ill elhow; swelling with redness o f both knee joint- and elhows ( 1 ' 0 1 V$TI.I'HRI~IS): paitlfilllv 1-estricte~1 ztlovemerlt: pedal edema; i n c r c a ~ c d s ~ hip$-freqliunc~. T: apical wstoIic- m tirnlur wi tll rndialion to axillae (mitral valve insuff~ciency due to carditis); hil;iteraf fine itispiratorv t>acal crepitilnt rales: 111iTri. tender t~epaf orneply. CBC: lcukocvtosis. S l r ~ t n r n r m / J~ ~?OFY~ on P ~ 11iroat wab: rr~arkedlv elevated A S 0 titers: elevated ESR; devated Greactive protein (CRP): iieaa live l ~ l o c ~ c r lr ~ l r t ~ KC(;: ~ - ~ prolonged . P-R interval.

Labs

Imaging

C : m cardiomegiil~; increased p u l n l o n a r ~ vascular ~ markings. E c l ~ o vegetationu : CIPPI. ~nirrrll ~ ~ I Ywit P 11 ~ . ~ g ~ i r q i t i l t i ~ ~ i ~ .


, J ~ C H ~for111 P chmcterizcd

Gross Pathology

hv endo-, m y , and pericarditis cllaracler.i~ed hy Iihl-nus scarrit~g with ca!ciCicatio~~ and mitral stcnosis with vetritcolrs Lhrin dcpouils.
( P ~ N :AR~ITIS) ( ; vhmr~ic Torn)

Micro Pathology

Myacardii~lmuscle fiher ~ ~ e c r o s~ in s lneshed in collagen; chw~rterisric fit~ding is f i h r i ~ ~ o i necrosis d wrl-ounrled b ! perivascular accumulation of mononuclear inflammatory cells
(h5c:trom-(:FI I S ) .

Treatment

Aspirin. corticosteraids, and diuretics: periicillin or


erythr-r~rny-it).

Discussion

. \ c I ~ ~r P h ~ ~ l t n a c Fever ic i q a S P ~ ~ I P I : I 01' LIPPPT respiratory infection wirli group .4, fi-hemolytic streptococcus: it causcs autoimmune d;ima~c to scrcsal orpins. prim;tril~ th t hcar t. Thy $yl;temic eflects of' acilir rheumatic (WPI- are i m ~ n u n e lnediatcd a i d are secr~ndary tr~ cros+rractivitv of host antistrcptococc;~l il~ltibodies.

Atlas Link

t_l_[_CI M-HI-052

'9 ACUTE RHEUMATIC FEVER

ID/CC

A 4X-yea~old iniwinnal-y wltn I n s lived in Can1crot)n. West Africa, for 20 year5 ic airlifted homc huu:~userlf lethargy, nuchal rigidity, persistent headache, and drowsiness that have nnt I-rspc)nclerlto antihiocics and wppr~rtivc tr-ealinenl.

HPI

H e states hat over t h e years h e has becil hitren in lllc neck sever-;tl t i i n ~ by s a m uturn~ ~ tor. n .tqetse fly ( &or,sr,\:l I:II.P~i.r.5). lle has alqo had intci-inittcnt. guncralizcd urvthenlalrrt~qr-hes

Z n r) rn
L

PE

Alpst hut s n ~ t ~ p w l ~ incoherent ar and confused:sornetirnes rlclr1~iona1; uuchal rigidik and tremors of face and lips; spler~olnepaly: generalized rubbery, painless lymphadenopa&y. prrdominai~tlv in pnstoiior neck ancl supraclavirular areas
( ~ 4 + l ~ ~ l ~ l : k l 3 o hl(;N). ~rl~~~~1's

LA

Cn rn

% rn

Labs

PBS/W: hvpercellular, trypanosomd forms presenL: Iynmphorytcs in CSF. Elevated IgM.


C~~IIT with I - Pe s $ ~ e n ~ a and i~lclurationas bite site: c h a n c r ~ resolvcq spontanc.ously; splcr-n and ! ~ n l p hnc KIPS enlarged (luring svstcmic stage: lup~orr~eninges et~larged dru-ing CNS invnlvement.

Grass Pathology

Micro Pathology

Skiri: erle~na, monant~clear cell inflammarinn, organisms. and enrlothelia1 prnliret-ation; rpleen and lymph nocles: histiocytic I~vperpIasia; CNS: rnnrlolluclear ccll rncningr~cncuphaliti~.
Surntnin: penrar~~irline o l -eflorni~hirle.

Treatment

Discussion

Also c:kllud sleeping sickness, hfricar~ trvpanosntniasi.; is a ws~einicI'ehriIe rliwase enrler~~ic to Africa whose chroriic form cauce5 a rnmin~oerrcepl~ali~is. T i i s cau.iecl In. the ilagellaterl protozoan5 'li.pnnnmnrn I P I - I I C ~p t n h i ~ n w(Mrest African) and Tr?/~nnoqom*r !T?ICI+ rl~nrlr~inzre {East X f i i c m ) . which are tr*anslni~red bv t l ~ c tsctsr fly.

AFRICAN TRYPANOSOMIASIS

ID/CC

A 213-yc;ir-oId male homosexual coi~iplair~s of' con tinur )us Iorv-g~~~clc. fever,weight loss,and diarrhea or 1 morlth's dl~ration.
H e also cornplains of' an extensive skin raqh, mucous membrane eruptions, recurrent herpes zoster inrection. and oral ulcerations. IIc rcl~oi-ts practicing I-eccptiveanal in r ~ ~ ~ c o u t - S F .
T7S: low-gr-adc. fc-vc~;PS: cachcctic: generalized lymphadenopathy: m;~c~rlop:~pula~rash: sevrrc sehorrheic dermatitis; aphtholts ulcers: 1% hirc con flue111 pa~ch i t it11 corrugaterl surCncc ( ( m u I I ~ ~ I K 1.Tl1KIIP1,4M L' I) along laternl hol-8r.nof tnnype; p e d e warts
( c : o K ~ ~ T . o \ ~ ~.\(:UMIV.\TA): \T.\
(:Oh7.4~;1051 I\!).
~ X ~ C ~ I S ml~ltiple ~ V U prtiritic,

HPL

PE

pink,

r~mt>ilic-;ltc.rl papulcs 2 to 5 mm in cliamrter- ( x i r l ~ ,i.lqst :t

Labs

CRC:: nnemia; leuknpenia with Ivmphnpenia: thromt~ocvtoper~in. Low CD4+ count; elevated C D 8 t T-crll ccnint; ELTSA fbr HTV-1 positive: Western blot confirmatory; PCR for viral R N A itivestig:~l ion 01' choice in winrlow periorl) positive.

Micro Pathotogy

Oral hairy lenkoplakia: Iesions s! low keratin pi-ojecriotic rc-.iulnl>l irlg flai1.s. koilocvtc~riu, and lir ilr atvpi,~; 11 yhl-idi~xtion ;tech11iq11~\ reveal Epstein-Barr v i r l ~ s in lcsion5.
Propl~vl;+r~ic anril~irrticc: Col- p r e \ e ~ ~ t i o~Kopportunistic n inrpctionq tvhilr monitorinfi Cn4+ T-crll cnllnl.5; an~il-etroviral d l - I I ~(ziclovucline. S cficlanosine. zalcitabinc. ;mrl prutcaw i~~llil>i~oi ~. ~ c )~ :~l~i~ and e l ii.ellahilit;ltive ng meRsui.es. AIDSrr-littud cornp1c.s ( N I C ) corlGsts of qmp~orn;~tia conditions in ;ill HIVinf~ctrr! ~x-ltient th;~ are ~ nnl i n r l t ~ r l ~in d the AIDS surveill;~nce case clefinitinn and that meet at lcast onc of the f'~)Ilowii ci-iter i ~ ia: ( I ) he ctuldi tioiis ,we ii~dicatibe o f a d e f ~ rin t
cnnrsp or

Treatment

Discussion

matiag~men t that

i s cc>nlpIicatecl I3v

HIV inrection.

Atlas Link

z-M1-054

AIDS-RELATED COMPLEX (ARC)

ID/CC

-4 2S-y-uai--old inale from India romplains of v~duat-unqet., intermi (rent, m p y abdominal pain with one tn tinu fod-srneIling, frothy loose stools daily.

HPI

His stools sonletimes contain blor)rl and mucus. He also complains of f l ~ t l ~ l ~ i i relieanus, ce, and, at titncs, alternating diarrhea and constipation.

tendetmess during palpatioil of c c c u n ~ and ascerlding PE Sligh~


colon; 110 hepatomegaly.

Labs

CRC:: mild le~~korytosis; n o cosinophilia. Fi-esh stnnl exzm-Iination reveals presence of Entarnoeba A istolytica cyst9 and motile hematophagous trophozoites; surolo~y for ant ianirhic antibodies is positive.

Imaging

Color~oscopy: mrrltiple colonic mucosal l~lcers that are slightIv r a i d a n d coverer1 with shaggy exudate: Inucosa hehvecn ulcers
normal.

Micro Pathology

Biopsy specimenf reveal lesions extei~dillg rindcr arijacenr intact nit Irma to produce classical "fEask-shaped" ulcers; amebic tr-ophozojtes demonstrated at base of i~lcer.

Treatment

Metronidazole (drug of choice) followed by paroniomycin or iocloq~~ino!.

'Discussion

Kn!rrrno~Oa l h 7 ~ o l ? r j t r r cyst? are infcctivc and are t r a ~ ~ m i r z e d through contaminated water, raw vcgctables, food I~atadlers, and
fccal-oral or crralanaI contact. The si I PS of involvement. in erdcr of freqlle~x-y, are the cecum ancl asccncling colon, I-ectlim, sismoid rolo1-1, appendix, ancl ~crrrlinal ileum. Trophwoi tes are the invnsire forill of the organism, cat~sirlg colitis or rli~tant infection by hernarogeno~~s spread. Complicatioils includc perroratiot~or the bou,el; livrl- ahsccss with pleu~xl. pericardial. or peritui~caI rupture: bowel obstruction hv amehoma; and skin anrl geuitali;~. ulcers around the pcritl~um

Atlas Link

T 1 M-Mi-055

A M E B I C COLITIS

IQ/CC

X .15-yu;lr-uld inalr Peace C:CII-~'~ v~11~nteea. I V ~ O r ~ s e i ~ tsperl't lv 2 vcat-5 in 1-11ralMexico corllplains of a spiking fever, m;~lai\c., headache, ; ~ t l r lright upper quadrant ahdomind pain.

HPI

I-Ic ,tclrnitq

to

I~avii~ I] g arl bloody diarrhea with muctis


mine

( 1 n s 1 xr 1.1n.) a11rE irnesttiul; t11a1 rli.;apl~rai.erlwit11

pills that

IIP took several n l o n t t ~ s ajio.


PE

\'S fever (99.IiUC:). PF,: pallor-: ~ l i g hj ia l l t ~ d i c ~ tender : 3+


hepatomegaly w i ~ h no rel>cmncl ~ c u d c l - u c pain ~ o n Iis t 11~rrt1wi(>ti riqhi lovver r i l x

Labs

CB<:: I c ~ ~ k o c v ~ ~ovth si i ~ r>cutrophilia. . b n c h i c cnts ill stool ~pcciln~ (1101 n cc)nclzrrrnt with abscess) : ptr$itir.u srr-[)Logyfor ;u~lil,odicst o B n t a t n o ~ h aItisto!ytica.

Imaging

CXR: elevar in11 or rig11t hemicliaph tagm: vnall right plcur;~l


ef'filsinn. C:'I'/US: c w i r a r i i ~ g Ipsirrn in right lobe of liver ( d u c to

abscrsl;) .

Gross Pathology

Multiple color~ic ~ r ~ i ~ c o11lce1.s. sal qlightlv rnisrrl xtld covered with S ~ I ; I K PSII([;I~P: ~ e t l l i ~ l . g Iiver ~ ~ l uri111one large abscess on rig311 lohe c n n t a i n i n ~ clinrolate-colnrt~d pus; a h s c c s ~ m;tv rupt~ir-c iiiicl 5pl-c;1clto lungs. l>r;~in. or olhci- orFln\. Srei.ilr pllr: atnel3a ~ n a v I3c ohtained from p c r i p h u ~ t. of l e ~ i o i i . Mcln,nicl;tzolc; nrccllr evac~ratir)n: sI1rKer.v in case 01' treatment t-dilllrr 01. T.1Ip~ltt-e.

Micro Pathology
Treatment

Discussion

Prior ri.;~vel tn enrletnic a r m 4 ~ I I I Fa ~~-i:lrl nf'fcver, hrpatornugaIv, i l i 1 ~ 1 rig11t u p p w q11adr:ttl I psi 11 are f~;~llin;~rlis nC hcpatic liws abscess. Colitis prcccclc.; lthc liver ahsccs5; ; ~ r n c h ; ~ 1lbc11 s ink<~rl~

A M E B I C L I V E R ABSCESS

ID/[[:

A 15-year-old male ~-11o rusides ill Florida pr-menuwith nausea ant1 vomiting, fever, ancl marked neck stiffness.
H r ; ~ I F o complairlu of ;l sc-vere l,itrc~nlal I ~ ~ a r l he. a r Carcf~tl hirtol-v reveal5 that IIP .swam K o r several hours in brackish water app-oximately a week ago.
i1.i.ital ion (neck rigidity, ~ I J G ~Tieriiig's c \igti and Rr11d7inski'ssign):
7

HPI

PE VS: frvci-; tachycarcli;~.PE: signs ot meningeal

r)

=
m

--I

Labs

LAP: hioorlv CSF (raiscd RBC count rnw ;tistr bc doe ru eanrniner75 inahilih~ to rrcogn ize prolircratin ji arnchas) ql1ow.i irllrllse IICU 17-upliiliil, pleocyt m i s . high prot ?in, ani[ low Fupar; no organism seen on Cram. ZN, or India ink s t a i n i n ~ of (SF: wet preparation of CSF I-cvealsviabre Nneglrria ttophozoitcs; cli;~\mosisconiirrnecl us in^ direcl Ilt~o~-escenr anti hnrly staining.
L ~ ~ i n are n h tnos~lv preqenlt in the ollac torv nc.1-veu and brain, Frrcal h e m r ~ ~ - r l ~ a g uxtur~sivu cs, fibrirloid nrc~.nsis, a n r l hlnocl vcsfcl thronibosis w i r l ~ nerve tiwue t i ~roris. r

z
m

Gross Patholagy

Micro Pathology

,\!ntyI~rrn]n~rllmi tl-ophozoitcs w c n a\ '0- 10 PO-pill-diamerer o i y ~ n i v n wiih s I;lr.ge n~trleuq, sinall g r a n ~ ~ lrvtnplasm, ar distinct ectoplasm. and I>ull>ou~ pscudopodia.

Treatment

Iritrncistur~lal;ir~clW amphotericin B, inicor~wole, rilkrnpill: pr.ogt>osis is verv poor.


Primal-v arnubic n ~ e ~ ~ i ~ i icep guttlalirjs r is causer1 bv amehas of the Krrlilc l'o~,q/~rirt or A r n t i ~ l r ~ ~ n o The ~ h n . roriner most often affects chilclrc~ ancl ~ you11g ad111 ts, appr-ars I o hu itcqlii~-rcl bv ~ ~ ~ i n i r n i n g in tlal-ni, lresh/l>l-ackishwater. and is almost alwxrs fatal, with 111canlclxi gaining rntrv into the araclmoid space lhr-otigli i l i e 1-til~a1 cl-ihriform platc. Am~~fhnrf?nr~bn i r ~ f r ~ ~ invr i ~ rllvr l s older, irriint~i~r>com~~rornised indixirl~lalq nnrl are ~orrleti~nes characterizerl bv sporltancous rctovcry.

Discussion

@,- *

AMEBIC MENINGOENCEPHALITIS

Ia/CC

12.?(I-year-old male goes to the emergency room hecausc of


dyspnea. cvano~is. Ilcmop~sis. and chest pain.

HPI

He Itas had a high fever, rni~laise, 2nd a nonproductive cough b r 1 rvrrk. TIlr patient is a sheep farmer w h o rernernherq having twcn rt-cited for dark black skin lesions in the past.
TTS: fcvcr-. PE: dyspnua: i7)anosis; bilateral rales li~arrl nver 1111rg.s.

PE
Labs

CRC:: nr~rmal.Neaa t ive blrmd and splltllm cul tul-es: diagnosis oCa~lthrax coi-tfirrned 1 w fnut-fold increase in indirrct ~~aicrnhrnraggl~~tiriatiori titer.
C X R medi;lc;tinal widening. CT, chrst: evidence nf "hemorrhagic mediaqrinitis."
P;ltclly consr~tirlation: vesicular papulchscovel-ed hy black eschar.

Imaging

Gross PathoEogy
Micro Pathotogy

Lrtnx.; show fib)-irlous ex~rriate wit11 manv organism? hut few


PhlNr.

Treatment

Isola~r nnrl treilt wilh TLTpenirillii~ C; or. ciprofloxacin Anthrax is ci~uscd by infection with Bacillus anthracis. ,I cell-rrean thras racci n r is availalrle 10 prukct tlinse employed i 11 inrl~tst 1-ips awr )ciated t\-iti~ a Iligh risk nf anthrax transmission (farrrlers. vetrlini~riaris,taruiery or wool workers).

Discussion

ANTHRAX

ID/CC

?',X-year-old t~lale receiving cytotoxic chemotherapy ( i m m u t ~ o ~ u p p r e r s cfor d ) act1tc lcukemia present c with pleuritic chcst pain. he~nuptysih,fever, and chillq.

HPI

I H c aluo colnplair~s of rlysprlea, tach!pner~. and a productive

cough.
Y

Imaging

CXR: rlrcrol i ~ i n g 't~tnnrlinpneumo~iia.

Micro Pathdogy

Lt111g biopsv icleuti l ies A . c / ~ P ~ I wit11 ~ / ~ Iseptat~'. E i ~ c t ~ t ehranc ly hing I1ypll;tr: (\i.;uali/ed by S ~ ~ V P I ~. t a i n s )nccrotizirlg : i~lflammaiion: vasri ~lar th rornhi with 111-phac (clur t o Mood vexqel invasion).
rV aznphorericin B or itracona7olc..

Treatment

Discu55i0~ The most Icthal form of i ~ i f r c l i r j nirwziq~ve , aspergilloxis, is seen prirri;~r-ilyi l l sever-rly i m m ~ ~ n o c o ~ n p r o i n i individ~tals. scd i.e., patients with AIDS: pativn ts tll p r c ~ l o r l ~ severe r ~ l , neutropenia folltnving ccvtotc~xicc l ~ e r n o i h ~ ~ apatienrs py; ~ i t chronic h

gxan~rlornatous disease; and patients rrccivinp glucocorticoids ;jilrl r )r her immunosuppressive drr~gs(c.g., t r i ~ i ~ s p Il a teripien ~i ts)
Atlas Links
1-,I
1 ' 1 , M-MI-059A, M-ME-059B, M-MI-059C

ASPERGILLOSIS

ID/CC

A 50-vcar-old male presents to the ER will1 complaints of recurrent, cuddcn-onsct. severe breathlessness, ~r.I~eezing, f~ver, chiII7, anrl a productive cough (sorr~c~irnes pi-or1t~r:iiig brown bronchial casts).
T h c patic11 t has had stcroid-drpr-nrlcn t chronic bronchial asthma fnl- 111anv vears and has no history of F O I - P ~ ~ Iiravel I or conlact wit11 a TR paticnt. H c l ~ i :Is 11islor.yof occaqional hemoppis.

HPI

PE

YS: fever: n~ar-ketl tachycal-dia; severe mcliyptlea. PE: r ~ ~ p i r a t o r y di~trcss: LC'IIU-;Z~cylnr~sis: wheezing: rhc~nc!iiand coarw rxles urrr hot11 lung ficlrls. CRC:: eosinophilia. Oxvguil s;~turaliorilow. Very high 1iter.s or spec ilic IgE antibodies against Aspe%tillus presrn (qpecific trrarker- for the dise;~se): <piltllm colrizrrs positive for A rjwq+ILus; skin tests m A.~Jl~\g-i/ll~s antigens positive. 1'ETii: ohstructivc pictlu-e (due to nncIcrT~ing a5tlima).

Labs

Imaging

CXR: segmental infiltrate in IIPPPI.lobes (shese infiltrates arc SP~IHPIILXI Iwrause thev col-l-espoi~d dirrctlv to the affcctrrl bronchi) ; branching, fingerlike shadows rr-orn nlr icnirl iinl~action of rliIatcd ccn tral bronc hi (\it.tuallv pathongnomonic of allurfiic brclnchopalniri~ asprrgillosil;). CT, clie.;~:evidence 01' proximal bronchiectasis.
Os;d col-ticastcroirl~ or I~cclomctliasont.. hllergic hronchop~~lmonxt-y aspergillosis {ILRPA) is a hvpersenqitivih disorrlel- that primarily affc-cts thc cmtral hrortchi; imrr~rtliafe arid Arthur-iyw h!per'~ei~ririvity reaction< arc involrcd in its piehogcncsis. Tlrc onsrt of thr disease occurs m r )ct oE'ten ill the I'ourtli at~rl f ftli clecarler;. a n d virtuallv all pstirnlq ha~meloii~-sranding ;tropic irsrhtna. Y n r r r a r ~ d hBPA4 lead5 to proxima1 bronchiectasis.

Treatment
Discussion

IDJCC

,-2 5(L)-)ear-nIrI alcoholic male prcsrnrs w i rh a high:?~ade

fever, cough,copious, fad-smelling sputum, and plcuri~ic righ t-sided clieut pain.
His wire reporrs that he was 131-ougllt llotn~ in a semiconscious state a few days ago. when hc was follncl Ivinlr; on thc roadside he-d\iTy under the influence of alcohul.
VS: f'ever. PE: signs of consolidatior~ elicited nvcr right middle a u r l lower pulmonary lobes.

HPI

z
71

rn

PE

s 5
S m
rn

r)

Labs

Spiiturn revpals abundant PMh' lte~~knwtes and rr1isc.d oral flora; cultme yields Bactpmides melaninoxenicus (Prevotella n~elaninogenica) and other Bncteroides species, F t r s o b a c f ~ r i u m , mimaerophilic s~eptococci, ancl Pefifostmpt ococcus.
CXR: consolidation involving apical segment of right lower lobe and posterior segments of middle lobe: la~~gt: cabvl~ wit11 air-fllrid Icvcl (nnsc~ss)also seen.

Imaging

treatment
Discussion

Clindmycin.
;Ycoholism. d r ~ ahtne, ~g acl~ninistrationofsetlatives or anc~theria. hear1 trautr-ta. and scizures nr. other neurologic tlixor-clrrq are innst often rc.spc)rlhihle Cnl- t h e devcloplnerI l of aspiration pnc~rrnonia. 13ecauw zu~aert)bcs are lhe dnminant flu!-a of the uppcr GJ iraci ( outnumberi~~g aer-ohic or facultat i1.r hacteria by 10 t o I ) , thev are the rlunlirlan~orgnnisms in i~spiration pncurnonia; of particular importance are I!orr~roirlr~ rnrl(rnino,ymrirsrs ( 13rm~ot~llo ~nrZnni7topirri) ar~d ot 11pr. Bnrtrrni(I~~ speci eq (dendm, pIeo~norphic. pale g~ain-negative rods). F~rmbnr!r..liium nurlrnt7im (slcndc-r gram-rrega~ir~ rods nit11 poirlrrd ends). anrl anacl-obit or- t~~icroacrophilic strupt ocorci and Pr$fo~/r.$fomrrr~r (qmdl ,gram-pr>sit ivr cnrci in chains 01- clunlps) .

*a

ASPIRATION PNEUMONIA W I T H LUNG ABSCESS

IDJCC

X 3Rvcar-old HIV-positive malc is adrnittcd to thc Ilospital with fever. rigors, night sweats, and diarrhea.
He I.t-ports e v c ~ r s i v e weigI11 1 0 5 4 oxel- ~ h cpaqt : f'ew weeks. fle was tl-earetl I'nt- Pnett rnncyysfis pneumonia a few werks ago ancl still reports a persisrent pl-nd~tctivc co~tgh.
is extl-en~elv emaciated: Jlep;~tn~plenomugal!.~ly

HPI

PE VS: li.ver. FE: patient


Labs

and l ~ ~ n p l i a d c n o p a r notccl. h~~

C:D,L+ count < .W/cc: M y c o h ~ c t ~ r i r r ravitlm-intrar~ll~rlorere n isolatr*d on hloocF r u l t t ~ r c srnrars : of tissue-5 ohtniiierl fi-om Ivmpl~ nodes, I m i ~ e rnarrrxv, r;plet111. liver, 3 1 1 ~ 1 111nfisreveal evirlence ol'acirl-lkqr harill i, arid r ~ ~ l t ~yield ~ r ek s1 . m~til~m; intrxtinal infection ~ i t l 11 l f. n l ~ itn l~ proven hv cuFt1lt~ of stools a ~ l r crllnnic l Iriopsy spccinirnc.
C T . ahtlorner~: hr~>i~tc~rpler)omegalv: I-c~rroperitot~eal Iymphnrlennp:~~h howel ~ ; nlucosal SrrIrI tllickening.

Imaging

Micm Pathology

neqpirr rllc pr-rsei~ce of'rnar~l; myc.rbh;lr~eria atlrl ~narr-nphages, well-former1 g ~ a i ~ ~ ~ l n were rr~a tvpicallv s : ~ h ~ e rluc n t tn profound impairment or cell-mediated immunity.
'I'he pl-i rnnrv rrratmenl regimen i n c l i ~ r l c claritliromvcin ~ and ctl~i~ml~r will1 ~too l r ~vitl~rr rifabutil~. u~ T11c failure rite o f 111erap is l~igl~.

Treatment

~ ~ S C U S S ~itO l ~~ t r ) J ) / w t ~ In II i ( r1 7 1 r ~ o 1 1c~lrnplex i'c 1 1 ~ 1 % tl~ lr

most frequent

opportunistic hactcrid infection i n patients with AIDS; it rr.pirally orczu-5 1:1tc in t h e i-onrre ol'11ie c \ ~ i d r o ~ n when r, other oppol-t~~nift inftwtions ic and ~ieoplasia have a1rc;lcly occurrcrl. Prcyltit 1i1xi.i ag;iinst .Idn o 7 ~ ~ ~ r m - i ~ r t m r ~i~ ~l recnlnlnet~drrl I11~1~rt~ in .%IDS par i e n t ~ nit h a CE34+ cnuut of' < 1 fl0/1nm'' (;tdmirlistcr a7i tl~romvcin. claritlirornrpcin. or 13f;ll)ir tin ) .

ID/CC

A 20-~ear-old 111alefrmm India presents to nawea and vomiting.

t 1 1 r ER

~ v i ~severe li

HP I

Carefill histclrp reveals that 2 hours ago hc ate soinr cmefrigerated fried rice that his wife had cooked the night I~eforc. He does not complain or any fcr-cr or diarrhea (mar;or in;~y T I C I ~he present).

2 ..
rn
0

PE

X S ':

11c)

fevrr. PE: mild dchvrlratio~l; ciiffils~ ~nilcl abdrminal

tenderners.

3 c
V,

Labs

Frcal staining reveal? no RBCs, T+%E:s. or parasi tcs; Bacillus cereus, a gram-positive rod, isofared from vomitus a n d stool and ~ I I o M - ~ to procluce the ernetogenic enterotoxin. St~ppmrtive.

rn

% rn

Treatment

Discussian

Rn,rilE~is rerrzis causes lwo clisi inct syndrnmcs: a diarrheal form (mediated b y st1 1:'~rIi~rirlti~ rnfi LT-type enterclloxir~with an incuhatiort period 0 f 8 to 1B hours: calrsed hy meat< anrl vegetables) anrl an emetic form (mediared hv a Stnpl~~~ortlrrrrs nuwu enpr cnteroi oxin with an illcubation period of 1 ro 8 hour-s; caliwcl hy fried rice). Proper forlcl hanrll i t ~ g and

BACILLUS CEREUS FOOD P O I S O N I N G


--

ID/CC

X :An-~c:ir-r~lcl lnalr w h o r r c ~ atlv t ~ernigl-alrcl Sroi-n Peru presen l q with ;m c-xtcnsivc. nodular skin eruption, inil(l ;rr~liralgias, at~rl orracic jn.11 ~ P V P I :
C)r~em o n ~h ago. t l i r patirn t had a l1igli-:;.r:it2~ fever I hilt lzpas ;rcco~np;u~ictl hv cxcc.ssivc w c a l n r ~ h rl~.spnr:~, . ill ~d Ixl'isiage nf colacolored urine: t lie Ft=ver ~ul,sidrtl;qf rpt- 2 weeks, 12111his ivr:tknp?\ ha\ progrrsserl qirire z h a ~ tilnt.. I'i~llol.:milr! ictc~.~ls: c>~rrnsivr %killIXIFII cornpi-i~it~g p11~1ish nodr~lar lesions or fiirl-irlg G/cs s r c n c)n Facr. ti-i~rlh. ;u1(-2 l i r r ~ b ~ ; mi Id h~p;ufi~p~(:il(3!~1egi11~: fltnd~tscopv rcv~als retinal hemorrhages.
1ntraerj.throcytic coccobacillary-for-111 hnc~n-i;i\isi blc i l l 1 1 1ick it1111 !li i l l liliiiu htni t1r.d nil li I:iorn\;i; hartelia seeti ~ I I isolated I ~ fmm skin lesions; irtrli~r.c~ srriltn hilir.ithit~rle\sarerl. PHS: rnnrrorvr ir, h\.pnchrr-rr~~ic ~tletnia wit 1 polvr-lirr~masia: nlilrkrcl r.ctic~~lm-\,tnui< (rlirr to hcmolvtic an(,rnia): C(~r~rnlw' lest rtegatitme.

HPI

PE

Labs

Micro Pathology

Skin 11iopzv of v.a.icl~Fa~ski11 Ic.iioriq rt~vratq pnrlr~lielialpi.nlili=mtion niicl I~iut~r >cnich\~l>r~~.plasia: rlectron n i i c ~ - o ~ c oorvcrl-i~cr,~ts pr~ tisuuc short's R ~ r t o n e ! ~ bn r r t i f f i f o m i sill i l l terstilial lis%~re. Chloramphenicnl, penicillin, erythromycin, norfloxacin. :u~~rl tetracycline art= d'rcctixrc: r i L ~ i i ~ p i ciic~indicatccl i fol- trc;Itmulrt of tmer-ruc( 111~ fi)i 111%. Rxrrnn~Tlosis i q n canrlfl17-hnrue hnctcr-ial discasc ocr~rl-ring r~lly o n thc western coast o f So~ith America ;I? IligIl illti l~ltleq: the c a ~ ~ f a t age111 i ~ r - ic :i ~ ~ I o t i lpIeomrtrpl-tic r, h a c ~ l l i ~B s .~ t / o r ~ r l / n /)nrrl/i/i)r~r~ir. Twr) .it;ige.i c)f 11rc rli~e;~\r. are ~,rt-o,qt~i/ed: ;ln initial f~hrilc stage awnri;~teri rz.i~Fi a hemolytic anemia /ORO\ \ ~r:ien) ;iiiri ; 1 I;II c a r . C I I I ~ I I P O I I S s1ngr r l l i l r n ~ tr t - i z ~h d ! hemangiornatous

Treatment

Discussion

PC"

BARTONELLOSIS

ID/CC

a trr~inl-v rare rcntri- 15i1li ,% 32-yrar-nld malt is rcfcr rcci chronic pneumonia ;~rlcl warty lesions (311 I1 is left ripprr limb.

HPX

'Thc p ~ t i c rti ir 1'1-on) ahv southeastern United States. His rLin le\~ntlsa r e n o n p l - i ~ ~ . i t and i r painlcsc. M c a l w r c ~ ~ l l p l an i T n~

PE

15: revrr ; t acl~vciirtlia: miEil t achvpt~ca. PE: bilateral rales and rhanchi; raiscd. verrucons, and crusted lesinns with scrpiginous IIOSCIPI- locnterl o n lcfi tippur exlrrmitv: small >~l>scccccc clctz~ons~rablc wl~cr si~l)ei-lic.i;\l ~ r r l ISI M.as relrlorccl.
rl~tzineous Icsio1-13 c l c ~ n o ~ i c r r .spherical ~t~e in dia~nercr) t h a l h a r e ;I thick-walled, refractile double cnntor~r xud slr ov 1111 i p r ~ l ; ~ (111.oxrl-based) r Inldtii I I ~ :r ~ ~ t of ~ pus ~ r nnd c 5p~itlirtro n S i ~ b < ) ~ r . ; ~ ;IFr ud'~ yiclrls p w t b or B I r r s f o n ~ v ~ en uo ; el itlrl~rto f i ~ c i ~ l -tlacilli ~ i ~ t Tc)r~n<l r i i l ~ e io - n 5t;iining ((11- on r ~ ~ l u u - C;c,n~ori's e: r l l c ~ l i r l ~ ; ~;iI\,el~~ni~~~ s t a i 11i r i r,f' ~ Iutzg I ~ S S I I P C J ~ P Fn o t rcvral r ' n ~ ~ l ~ n oF.q s / ~
Sputum
;tritI 1111s From
tnln

Labs

cells ( X to 1.5

Imaging
Micro Pathology

CYR: bilnlcr;~l ;~lvcr>Tart > o ~ ~ ~ ( l l i r lwith x~io air ~> II ~I - I ) T ~ C I I C ~ ~ ~ . ~ I ~ I S .


Epi~lielioidm a c r o l ~ l ~ a a jnid ~ gianr ~ ccllh . ; ~ i r i . o ~ i i ~ r l ai i ~ g +ii~'pnra~ive center: skin Irsiollu cllou! p.ir~rclc~t.pitl~elior~~alo~~s h! pcrplasia vcrr ~imililr lo ' i q ~ i a n i o u cell ~ cxrcinnrna.
Itracona~ctlc.i.i ~ r - c - ; ] ~~nerir ol' rhoice in mr>slp,~tic~n ts: ampl~otel-icin R, flurnnxzolc. :ultl k r t c ~ c * ~ ~ n ; ~ w /o el altcr-nalivc. c~

Treatment

CI~LI~S.
Discussion
Rlas~nnlvcocis is a svstcmic. i ~ n r c ic. o ~i I ifrcr ion of I i l ~ i n a n ;~ncl ~ d o ~ thi~t s is cl1:t~-actei-i7rd In) rizpp~u-atinn ancl qI-ari~~lrlrn;jft>~~s Ic.iions and i q c ; l ~ ~ s ehp r l the dimorphic fr~nps Blasto?nvces d ~ r r n a tidis: fi t lip rlise:~.;c. is endemic in the sauthcastern and sotrth-ccntral portions of the United States, and scvcral j~or kets or i llfectioi~eklei1~1 nnrth aIonfi thc %li.isissil>pi: ~ r ~ Cll-iio rl river'; i l l t o rent ral ChnarIa. Clinical cl ~se:~\r m n n crm~rnonly invrilvcq L ~ IC r ~ n g s(;~cr/iri rrtl hy ~il301-P inllalation) ancl 111rr1, !I\. hcrnatogcuor~\ di~serninati<nn, thc skin. t h r 4cc-let al .;!,strln, arlcl thr mnlr gcnitr>~u-inar-y 1r;lct. lrll'cr'tio~~ c;lnnoI Ile pacsccl from pcrson tu pel-.;on.

Atlas Link

I_'

M-MI-065

BLASTOMYCOSIS

IDJCC

A %-yeal.old ~ n a l e presents wit11 si~clden-onsetdouble vision ( r , ~ r a r , ~ i . ~ ) dry , mouth, weakness, dysarthria, atwl wphagia. He has nn previous I~istnrynf rpisorlic weaknew or uf dog or tick bites (vs, myasthenia g r m i s . ral~irs, or L ? m ediseasr). I..ast night, t ~ ate c some homecanned food.
VS: no F r v ~ r PE: . pa~ient alert; P I ~ F ~ Sbil;l~el.ill : third m d tenth cranial nerve palsy: qyymni~tric flaccid paralysis of all rnur limbs; dcrp trndr~n rcflcxcs rcducecl: n o selrsol-y loss secn; clucreasod
hc~wel cout~ds.

HPI

PE

Labs

Rntt~lin irru toxin c l e ~ r c ~ c in d patien t ' q serum and canned-food sample wit11 ~pecific ailtiserl~n~.
Anllitoui~~: dose monilwring of rc.spir;uory statlls: intlthatiot~for rrsl-liratorv I:aill~r-e. The discasc is charactcrizcd b?.gradiral return of muscle sil-rilgth in most ciises. R o l ~ ~ l i ~ i toxin ~ t r n is a ;rinc rnct~llopl-oreast. that c l e a ~ e s specific- companrnt.5 of synnptic v~\icle rlnckinq mict fusion coniplexer. tllus inhibiting the release of acetylcholine at the ne~~mrnuscular j~lnction. The C ~ ~ ~ P in ~ Z arlult.5 S P i s drle to ingestion of thc toxin mther than to bacten,jl infcctiun. Botr~lisn~ is alscl seen ill infants secculdary to tlbc ii~gr\tionelf C:lorlridirrm I~trrlintrm spores ill honey.

Treatment

Discussion

BOTULISM

ID/CC

-428-year-old wltir e male {isits his family rlnctor cornplairzi~~g nF acute pain in both hip joints together wit11 wcakrl~ss. Iiarkache, mvalgias, arrhralgias. 2nd nndttlating fever of 2 months' duration; this morning h e woke u p with pain it1 his right ieslicl~.
For t11r past 3 yra1.s he has ~ v o r k ~ a[dthe largest dairy favm i n h i 5 stair. He enjoys drinking "crt~de" milk.
T S : fever.. PE: pallor; marker1 pain rrn palpation OF sacroiliac

HPI

PE

joints; mild splcrlorrtegaly: generaliz,cd l\mpharlennpatii~;.

Labs

( . : R C : : relative lyrnphoc~+toriu with normal t\'RC: ccit~u t. Pnsitive


;1#~l~tilla~iolI titer (> 1:160): rising serologic tiler over time; 3in;tll gram-negative 1-od Brztcefla abortus on I~lnorl culture.

Imaging

XR, hips: join I ~f'filsion and soft t i s \ \ ~ eswelling with0111 rlestruction. MR, spine: evirlence of ~pc~ndyliris.

Gross Pathology
Micro Pathology

L~~~iphacl~.nopathvclnptlv a t ~ r splenomegaly; l hepatomegaly CIF-e.


Gl~xnr~lomatnns foci in splertt. liver. and lymph nodes, n<.ltli ~rrnlifel-atis11 o S macrophages: cpilhelinirl and @ail1 cells inay hc
seen

Treatment

Combination ~hw-apy cvith doxvcyclirie or 'ThlP-SMX and rifitrnpin or stseptomvcin. :llso callpd Walti~ fever. a microbial discase of animals. hruccllosis caused hy set cral cperies of U n i r ~ / l n a, gram-~~egativc. aei-nhic roc cohacillus. It is transmit trd t u h ~ u n , u ~ throl~gh x the drinking oS containin;~ted1rii1Icor through dir cct rntltact with pl'od~~cts or iissr~es from animal<such as goals. 511clep. canrels. cows, hogs. and doffs. T h e clinical pict~ii-t. is orten ~ : ~ g i l111115, c ; a high index ol' wrpicion t m y he tlrressary for-diagnosis.
iq

Discussion

BRUCELLOSIS

frmalt. prewn ts to the ER rvitli i i t ~ e n sacutuonsel ~, left lower quadrant crrampy abdominal pain, Ihul-smelling stool5 tr it11 rtreaks of hlrrucl, 11 rgenc!: tenesmt~s, ancl lever.
X %+ear-old

For ihe pact 2 clays, I ~ L palient ' has also had heachchrs and n~rafgias. She frequently drinks unpasteurized ("ra~v") milk thal shc h11vs ;lr a health-food store.

L7S: frvcr (30C:);tadl~carclia; normal 1tR Lunrt EP. PE: rlo


dehvrlt-ation; rlifT~~se ahclon~i~lal 1ptldernes5 more ~rtat-kerl it1 left lower q~ladrant.
StnoI snirar shows leuk s c y ~ e s (due to inmsive tissue darnage In tl1e coion) a n d gam-negative, curved bacilli, often in pairs, in ''gull-uri~~g"-sllt~~~cd pat ler 11; rlarkficld exam shows motilitv; cultl~rc ir~ inicrn;l~rophilic,42C c o l l d i t i o ~ OII ~~ spccial agar vielrl~C a ~ n p y l o b a c f e jejzrni, r inrlicaced hy oxidase and catalasc 13( )sitiviw.

Gross Pathology
Micro Pathology

Ft-btl~le colonic Inurosa.


Nonsperific inffammator? reaction consisting of'nei~trophils. lynphocy tus rid plasm a cplls with Iiyprrcrn ia, e d ~ m ancl a damage lo rpirlielininl. gland11Td1clr~cnerxtinn, ulccratior~s, and crypt at>scesses causecl by c u l o ~ ~ ~ i~ o S S I Iinvasion P of the nrPtt ism.

Treatment

Sclf-liiiiiling d i ~ e x ~Scvcru e. cases ( i .e., high fflcr. srvcr-e cliarr-Ilea) can he trcatrd wit11 fluoroquinolones.
One a [ rhe priinary cause\ of "I rav~ler's diarrhea. " Sor1i~ces of inrection incluclc rmdercoriked food ancl contacf wit 11 infected animals arid their-eu~l-eta. Prevent by impl-oving 1)1~13Iir sanitation. pasreuri7ing milk, alzd propur cooking.

Discussion

"AM

PYLOBACTER ENTERITIS
-

I D/CC

X 49-yea~olrl mol-hirllyobese, diahetic woman presents with prurituq in the skin folds beneath her hrcasts.

HPI

She admit? to having tliis proF)I~rnrhranically, especially in the warm w m n i r r months, when she pel-~pircs more heavi l p

PE

St [pel-liciallvdenr~ded, bee5-red areas bcrleaih breasts with sntcllirr V ~ S ~ L ' C ) P I I S ~ ~and I~PS whitish curd-like concretions on surfilce. C l ~ ~ r t eof r s budding cells with short hyphae seen i~ndelhighpilvi7el-lrnc aftel- kin scales have brtn p111 in 11)5$ KOI-I; C l l t d i t l n cll/~ir-(rn r isolatrd in Silmr ~raucl's mccliur~i.
Kash has whisisl~-cr-earnv p~eudomembrane [hat cover<an rrvthema~ou.:rtr rfare.
\bast invadcs sr~prrflcial Iayct-4 nf epithcli~rrn.

+-I*

rn
I ?

3 c
W

Labs

z z
vr

Gross Pathology

Micro Pathology
Treatment

Iiecp affcckd areas dry: rlotrimazolc or r )ltiel- a11tifr~ngalage11ts IocaIlv.


Other superficial areas of inrection irlclttdc the c)raF lnucosa ( t111nis11), v;t,qi~ial nlllrnsa (vaginitis), a i ~ r esoph:~p~< l (GI tailclidiasis). Sysretnic i11vasir.v ciir~didiaqiq tnw be sccn ~ i t h tmniunosuppres6on irl patient5 receiling chronic hmad-spectrum antibiotics. in /\IDS patients, or in those receiving

Discussion

CANDIDIASIS

IIl/CC

A 23-year-old female presents with painful h m p s in her right axilla and neck together with low-grade fever.

HPI

Three weeks ago sl-ie was scratched on her right forearm by her pet cat: an erythema~ous pustnlc initially d~velopecl at the sile but rcsolved spon rm~eously within 1 4 J days.
VS: fever. PE: tender right axillary and cervical lymphadenopathy.

PE

Labs

Lymph node biopsy ddiqnostic; serologic indirect immunrifluorcscerlt anriborlv test for Bartonella henselae is p o s i t i v ~ .
Hen~atoxylin and eosin stnining reveals gandomatour pathology with aelEare necrosis and surrounding palisadrs of l~istiocytic CPIIS; WarthhStarry silver stain reveals clumps of pleomorphic, strongly argyrophilic bacilli.

Micro Pathology

Treatment

Syrnplomat ic; fluctuan t node may need aspiration; aai thromycin given to imrnunocompromisrd patients.
13n~to~wlln km~rlrct is dle agent that causer cat-scratch disease. Lprnph;ldenopathy can persist for-months and can sometimes be mistaken for a malignancy. Individuals who arc im~n~ir~ocompromiscd may present with seizures, coina, and meningitis.

Discussion

c"?r

CAT-SCRATCH DISEASE

lD/Ct

,413

R-yenr-old white female enters rhe enrergeilcp romn

cr>mpl;iining of hradachc., malaisr, and 11ipalpcbr.alswellins of the right eye.


HPI
She r.rcchntly r-rtur-rled ii-om ;I year-long stav in Brazil, whrre herl i ~ t h r works r as a logger in the .h~lazon forest. Ovcr thc p i s t week shc B~ncla hifi.11 .ver. which ~ms trcatcd at 11omc as in;~bria.

5 . rn
>

t l

PE

1%: k r e r (39OC); tachvcarrlia. PP: right evelicl slvolleu sllr~t 2 0 (Rc)ni 4Q,\'s S F ( ; K ): lnnrkedly tlvper~rnic coil junctim; ipsilateral C Cn retroa~nicular and cervical lymph nodes; l ~ c p a t t ~ ' c p l ~ r ~ o r n e p l ~ .

z cn

Labs

PBS: trypanosomcs on thick blood smear. ECG right bunrlle hl-anrh hlock: v c n t r i c ~ ~ l ~xtrasvstoles. ar

Grass Patho Logy

Er~c~~pn~l;itecI, nod tilar are;{ (r:t-l..rr;t 1 ~ 7 4 )or- Romatia's sign lnw IIC scen at point of en1r.i; corntnot~ly the face.
Iilte~lsr nenti-nphilic iiifillr;~~e with ahizt~dxrrt maci.ophafi-rsat ire of'entl-y: nlynral-dial ~lecrosis with mononnclcar ccll infiltration; psez~clocysts in ink-ctetl t i s n z ~ \ crm Li~in1~~1.;14ites 1 hat ~r~llltiplv within ceIls: dc.nu~-\atiori nf rnyr~i lrric ~ i plexlks. ~ t
Nilul-timox for acnte disc;~sc.
(;haps' rliseaw is a parasitic: dise:~sethat is restricted tr, the Ameriras (endemic in Snnth anrl Central X~ncrica)and is pmtluccd by Trypanosoma crr~zi. 21 rhin. iimduTating fl:~grTlntrd protryzclan; i I is 7 r;~nsrnitted hv rtmtn~ninalinn of n reduviid bug tire rvi.iti1injection or its f c c c ~Also . known :t.i /\rnericai~ trvpanoromiasic. Long-standir~g ci1sr.s show rnyr>ci~r.di;~l involvc.ino~~ t ~ 111 . diJated i cardiomyopathy, life-rflrearet~ing r ot ~ c l ~ ~ c delPcls. ~ i c ~ r ancl ~ apical a ~ ~ c u r v Ibrm:~tion ~m anrl m;iv nlrt 3 show megaesophagus or mcgacolon.

Micro Pathology

Treatment

Discussion

Atlas Link

m l I_ M-MI-071 ' -

CHAGAS' D I S E A S E

ID/CC

A ?15-!lear-c~ld male complain^ nf cough procluclive nf m u c o p ~ ~ r ~ i lsputum ent and breathlessness.

HPI

Before the nnqet of thcsc symptoms. he had a snrc throat wit11 I~aarsencss. I-Ie has n o hislory of'hemoptysis. sharp cllest pain, or high-grade fever.

PE Crepirations heard over left lung base.


Lab5

CHC: normal 1ri~knc-p~ cnllnt. Spl~rum pxilnl revealed no bacterial organism: mirrc~immunofluorescericed c ~ e c ~ e sp~cics-spccilir d an tihotlies directed against I;lt!nydicr o111er-mem hrane proteins: rulrivation of 6. pnmimoniaedernorlsrrated on HEp? and HL cell lines.
CXK: left lower lnhc suhsegrncrl~aI i~lfiltrate wi t11 interrtizial pattern.

Imaging

Treatment

Doxycyclhe is the drug of rhoice: erythromycin and fluoroquinolones Inav also l r r c used.
The peak incidence nf ch lamyrlia pnc=u~nonia is in vorlrlg adults. Tllc rnorie of iransmis~ion would appcar to be from person to person.

Discussion

czra

CHLAMYDIA PNEUMONIA

IDJCC

A 30-year-old rrla11 has sudden severe. profuse (several liters per day) watery diarrhea, protracted vomiting. and abdominal pain.

HPI

HP has just returned From a tr-ip to rural Jndia.


Severe dehydration; low urine ot~tput: genemlizcd mild ahdominal

PE

tendei.tl~sq ~ 6 t n0 h signs ot peritnneal irritalion: stools havtl

characteristic "rice-water" appearance; (grav, slightly clol~dy fluid with flecks of mucus), ~ i t no h I>lood.

Labs

Stool czllturc 1-weals ST-am-negative rods wit11 LVdarting motility";

0 1 antigen detected; VElrrio r l i n l m ~ isolated ~ on culture inedi:~: swum chloride levels dccrrasc-d: wr.utn sodium lcvuls increased.
Treatment

Vigorous rehydration thcrapy wi tl? ~11.al a n d / o r TC7 fluids; tetracycline, cipmfloxacin, or doxycyclinc.
2 2 heat-Iahilc cxotoxin produced hy Irilm'o rhoIrrnr that acts by purmane~ltly stimulating Gsprotein via AI)P ribosylation. I-euulringin nrtivation orintracellular adenylate cyclase, which in turn incrcasrs cAMP levels arid pt nri~lces secretory diarrhea.

Discussion

ID/CC

X nr\vhorn hahy is t.errl-rerl to the petliatr-ician lin-fitrther. evaluation of a11 rln~rsually small hcad, low birth weight. and an rxt~nsive erythematous rash.
Inbauterine growth retardation was prenatally cliagnnscd on 11ltr;zsound. Tile child's mother had a flulie episode during thc first trimester of hcr pregllanq.

HPI

CI

z
71

PE

Small fnr g e ~ t a t i o ~ ~ aRe: a l generali7~d hypo~rmia with sluggiqh neoil;~t;bl reflcsrs: rs~r.nsivr 'LpinpointJ' petechial skin rash
( M 1.1 .HL. IUZY VCII FIN K.ISH)

; rnicrocephaly chorioretinids: mild

icterus: hepatosplenomegaly; sensorinewal hearing loss in right


rar.
Labs

";I=I

c m

C:I3C/BI%S:mild throinhocvtnpenia: a h ~ i c a f 1~mph~)cytc~si~. Moderately rlevatcd direct serum bilisr~l~in and rranraminases. L; )kcells in I I ~ - ~ I I I'OUIIC! P tn have large intranuclear inclusions (r ,u't.'\ ICLT.: IN(.I.I SIOKS) : c y t o m e g a l o v i r ~ isrllatecl ~~ an tissuc culturr.

Imaging
Treatment
Discussion

XR/CT, llead: periventriclllar ~ a l c ~ c a t i o nmicrocepbaly. s;


G~nciclo\ir(onlv for irnmur~ocnmpro~ni~rd patic11ts)

A congenital herpc~c-irus infcction involving the CNS with eye and damage, congeriilal cv~ornegplovii.~~~ i s a c o m t u n n caltse lller~tali-eliirdar ion.

CMV-CONGENITAL

ID/C[:

A 13-war-olrl whitc lkmale v i ~ i r s her fever, srvel-e dyspnea, ;mtl a dry cough.

complaining of

HPI

Sl~c was rcccn~ly tli;tgno~rcl with acute I~~~mphr~c-vtic leu keniia, Tor which she received a bone marrow transplant. S l ~ e is c ~ ~ r r e n t l y on immunosuppressive therapy.

PE

VS: fiver: tachynea. PE:: pallol-;-pitant

rales oirr h t ~ h ILIII~ fields; mild c?-,lilnsis:no hepatu~plcnor~~egalv.

Labs

CBC/ PRS: anemia: leu k n p ~ n i aABGs: . hypoxemia. No organism ill i i ~ ~ l i l csputum ~rl stainer1 with Gram. I;ierrisrl, %N,and rnrf henatninc sil\~c.r.
CXR: dif"fi~s~, hilacera I in tcssti~ialinfil tratcs.
Interstitial pneumonitis: hcpati tis.
C:hi~rxctcristicintranuclear inclrrsions with surrounding halo ((TI{I.''~OR HL-L~.'s-I:~T,c FI I S) nn tr;lnshrt>~rchial 1i1ng biopsy.

Imaging

Gross Pathology
Micro Pathology

Treatment

Ganciclovir (C;hW is resistant t (1 acyclnvir).

Discussion

An errtelrrp~d, rlouhlc-strarldr-dDNA virus bclungii~g s o he Iie~lwwirus gruup; the most ralnrnon cattsr of pneumonia a tld drath in bone marrow transplant patien&. I t i s also cr,lninon in AIDS patients.

CMV PNEUMONITIS

IDJCC A 30-ycar-old homosexual white ~ n i ~pl-escnrs le to his family


physician with a rapidly progressive diminution of vision.
HPI
i-le i q knowm to be HIV positive ancl periodicaIly comes in for chrck~rps. Cotton-wool exudates, necrotizing retinitis, and perivascular hemorrhages fi~nrl~x~cnpic cum.
Ganciclovir: roscarnet (CMV is resist an I lo acvclc~~ir)
w

PE

Treatment

Discussion

CMV retinitis is an imporrant treatable came of blindness that ncciu-s in 20% or AIDS patients; 5.03, to (iOL% of patients dc~rclop retinal detachment within 1 ye.lr. Toxnplnsmosis a i d progressive multiIcjcal Irukocncephalop~tt~ (PML) are other irnportntlt callseq nf I~lincIness in ,%IDSpaticn ts.

CMV RETINITIS

A 19-veavold migr-ant worker From the southwestern United States is brought to thc familv doctor coinplaining. of cough, pleuritic chest pain, fever, an rl i n al ai w.
I l c also complains or a llackacl~e and headache along with ;Iri erythernatous skim raqh (due t a Ilvpcrsrnsitivi lv react ion) in his
lowcr Iimhs.

IPS:fc.ver: rac1kvpnt.a. PE: central irach~a; coarsp, crrpitant raIes ovei- both lung I > ~ F P ~ :~ e n d ~ eryhematous r, nodules over shins (I-KYTI IEMA ~ o n n s r n t )periarticular : swrllir~g of knees and aizkIes.
Labs

Posi tive skin test with coccidioidin; dimorphic fungi (F~yphae iiz wil; s ] > h ~ r l t l t in ?~ Imrly T ~ S F U ;~ C ) orridiaid~ iinmitir ~ on s i h w stain and s p t ~ m ~ rultm-e: n po~i tive latex agglutination test. CKC/ PTIS:
eosinop hilia.

Imaging

CXR: ~ ~ o d u l infiltrates nr a n d thin-walIucl ca\~ities in b o ~ h lower III~~s.


Gaseating granulomas: orten srtbple~rral ancl in luwer lober; necrnsfs and cavitation may- alsu br pr-esenl.
Silver.-q!aii~~~d tis\tw enrlospores.
SPC~~OI st~nxz, ~S ~p11erulcs

Gross Pathology

Micro Pathology

fi llcd ~ i t h

Treatment

Discussion

Endcmic in tl-lt- ~c~~iithweslrr-11 United States, cnrcirlioidonlvcosis is [>rnlrl~~cecl hy C i r n w ~ ~ f and ls is transmitted l>v inhalation of arthrospores. S ? f % t e l ~ disser~iinat ~ic ior~ i s frequent in blacks as wcll as in ~ ~ I ~ I I I L ~ I ~ ~ F I I ~ xild ~ I Ipregnant I . ~ S S ~ C patients. ~ Mcnirlgitis nr gran~rlomatous l u n g cliscasc may result. wlrich mm Irad FO

death.

COCCIDIOIQOMYCOSIS

IDJCC iZ 2Ryai.nlrl rnale who lives in the northwestern United States


cornplnins or a Iligh-grade fever with rigors. ger~~ralizerl achcs, mvalgias, hearlachc. and hackache.
API

Four d;rvs ago h e r-e~uriled li-om a hiking trip during which he was bitten hy a tick: he took m~~oxicillin as prophvldxis wainst

LJIIICdisease.
PE
\IS: i&ver.

rn

4
LA

Labs

CRC: leukoprnia: relative lymphocvtoris. Vir-nl atltigeil detected in RHCq bv i ~ ~ ~ ~ n u n o f l u o r e s cColorado c ' ~ ~ c t l ~tick v i r u s cultured it1 ruckling mice hv intracurrbr-al inr,~~~lrtrion of blood clor; indirect flriorescent Ab teqt podtive.
Syn;vlnt>toinatic.
Colorado tick Ft-cur virt~r: i q a n XO-nm rlouble-shulled reovirus tha~ i s cover-ed with capsomeres: its icosahedral core contiins 12 segments of dsRNA. The discaye is a monosis that is t ransm itterl I n , ,va wood tick, Drmtnr.~nrrn n??/ln~nn It i . occtlrs pl-ima~ily in thc Rockv Mountaitz region. primarily ai'recting hikc-rs. Since n n specific therapy cxists, p l ~ v e n ~ i o is nkev (wear rlr~thing that covcrs thr. Imrly).

rn

rn

Treatment
Discussion

ID/CC

A 2-year-old malt. i s brougllt to the KK hv his parents with sore throat, inspiratory stidor, anrl a harking cough of 1 clay's cl~~ratioi~. The patient has no significant pas1 medical histnl-y.

HPI

PE

16:Frvcr (SP.li0C); t;~chyprtpa. PE: respiratory distress; ~~asopl>a~-yngenl discharge: di muse rhnnchi and w h r e ~ r s ; esamina~ion of extrrmirirs rcvcals some cyanosis.

Labs

Th t-nat and nasal SWIM

isolilte pafainfl~ienza virus: sercrdiagnosis xtlrl lieinagglutinin itlhibition rests reveal hpc I (niost common cauqe) .

Imaging
Gross Pathology

CXR: nil- r 1-ilpping.XK, neck: subdottic narrowing.


Inflammatinn and edema of larynx. t r a c l ~ e a and , bronchi.
Mos~ cases rrrsl)oild 'to supportive therapy such as llumiditied ail; t-ernoval of wrretions, ancl her1 rest. Swurc cases tnalj require

Treatment

ZiumirliGed oxygrn, racemic epir~rphrine, or high-(lose curticr~s~ei'oirl~. Discussion Difl'orenlii~te crclup f'rotn Hrn~nlof)hr'hr~ rn/7jrpnzn~ tvpc B and influenza A vil-11s. R4nrle~ of transmission inclr~dcrcspi~ atorv C I I . C ) ~ I P I S a n d person-tmperson con tact: tcnds to peak in the fall ancl w i n k s . Most ca7c.i of croup are rl~w I n parai~lfluenza virus tvpe 1 : t ~ p e 3 i s prominent cause of hronrhiolitis in I~abies.

ID/CC
HPI

A 3(tyeiil=oId tnan with AID5 presents with c hl-onic, recurrcnl profwe, nonbloody, watery dimrhea. Thc diarrhca tias r ~ c u r r e rover l 1he past 2 mon tli\ with i l l tel-miurnt ctampin g, and pr~vinur trc-atnleil ts h a w not bct.11 ~'fftlcfi\~e.
1 1 0 reves. PS: rnoclrrate dehydration: thin; generrdized l\-rnphadenopa~ly.

PE

C'S:

Labs

Acid-fast mining dpnlnnstrates oocysts OF Cryj)tn+j~nrirJiun~ in f re\h <tool.

Gross Pathology
Micro Pathology

I~iresrinalmrlcosa appears normal Blunting of it~te~tinaE villi: mixcd inflan~ma~nl-y re11 infiltrates uith eoxinophils in 1:tmin;l prnpr-ia:orgnnisms irisihlt. on hru9h hrrdcrs.

Treatment

No treatrrien t found effectivr: suppor.riv~ managernrn t with mait1 tenanre 01- fluids and nu triliorl.
(.'r~/)fo$/)nr*idi?~n~ /~nrr)wn infrctiorl presen rs as acute diarrhea in malnourished cllildren a n d a5 'ievere cliarrllea in immunucornpr~~tni~ patiet~ts ~cl (part of H T V wasting sy~rlmmc): the r l i s ~ a iss ~ trlild arlrl self-limiting iri irntnlrne-compctcm patients. Thc cliwnse ir arq~iil-ed through the ingestion of oncytq (fecal-nt-al trans~nission)that rriav be billed lly chlr>rir~ation.
I_U_LI M - M l - 0 8 1

Discussion

Atlas Link

ID/CC

A 5year+ld wliite male presents with malaiqe, anorexia, lowgradc fcvrr, sorr throat o f .3 days' duration, and dvspnea 011
exertion.

HP I

The child was raisccl abroad. H i s imt~innizatinn status cannot he durermined.

PE

17s:fever; tachvcardia with occasic~naldropped beats. PE: cervical lymphadenopathy ( H I 11 I .'+NI:~:K A P P ~ W . \ N C E ) : ~ ~ n o o t h , whitish-gmy- adherent membrane over tonsils and pharynx; n o l~~parosp t~ornegaly; le diminislled in lensin, nF Sl .
Metachromatic gran~des in bacilli m g e d i n "Chinese character" pattern on Alhcrl stain of thr-oar culture: C o ~ n ~ ~ t a r t ~ W z c n ~ drphthnne confirmed by growth observccl on Liiffler's blood agar: erythmm and ~zecl-oGs follorving inzraclernlal ii~jection of C.'. dij)h/hr?+ir!r (oxin ( ~ o s r . rS(:I ~ \I~CK'S ~ TEST); i ~ n i n t ~ i ~ ~ d i C ~ u s i o ~ ~ st1 ldies ( Elek's) confirm toxigu~iic slraills r )f C. d7plltl~m'ne. ECG: ST-srg~nent rlcration; sccond-rlegi-ee heart Idock.

Labs

Imaging
Gross Pathology

Echo:

F~~~UTIC of C myocardi '

ti'^

Phal-~ngcal murnbrar~es not resiricted to anatomic landmarks; palr and enlarged heart. Polymorpltoi~uclcar estlda~e with hacleria: precipitated fibrin and ccFI drbris forming a pseudomembrane; markt.d hyeremia, cdcma, and izecr[~sis nI upper re~pirator-y tsact rnlrcosa; exntoxin-indk~ced inyofihrillar hyalinr rlpgenet.ation; lysis of myelin sheath.

Micro Pathology

Treatment

Brgin treatnlerlt o n presumptive rliagnosis; specific antitoxin and pr~lirillit~ or erytlxromycin: respiratory a11r1cardiac suppol-t; confirm ci-iidication by repeating throat culture.

Discussion

,IZ hacturial infecliclrl o f I lie tlr roar, djph thcria is p ~ - e v talde ~ n Ilv vaccine and i? rallserl by toxigcnic C.i~ynr/)l,rkr./wi~~m diphthkzp. a club-shnpccl. gram-posi(ivc aerohir bacillus. Diphtheria tc~xinis pn,d~~cecl hy P-prophage-infcctcd coryriebaclel-ia; it blocks EF-2 via ADP rihnsylation and helice ribosomxl fimction in prolein srnshesis. Thc toxin enters the blondstream, cai1sin5fever, myocarditis (tvi thin the first 2 wecks) , and polyneuritis (many wecks later).

Atlas Links

e 3MMl-082

1 " 1 MI-324

DIPHTHERIA

ID/CC

h ST~ycarold rnale p1-okssoi- of \,etel-ii-iarymcdicir~c horn New Zealand experiences sudden high Fever with cliill~, jaundice, arlrl right upper quadrant pain wl~ilc ar ienrling a cnr~krrnce in t h r Uriitetl St;ltes.

HPI

Ilis past 11islo1-y i q unremarkable. He 11x5 been hcalthv and has bcrn phv'iicillll: ar'tive t+orkinxin llle field with dlccp and Z,r-et.dirl~ dogs.
VS: lever; l~yputtnsic )ri ( R P 90/50). PE: hepaiomegaly jaundiced sclcni: on pal pation or cpigast~iiun and riahc hypucliondl-i~~rn, abdr ~rnen ic: tender with n o rchu u tic1 tenclcsncs~.
CkC: leukoq-tosis wilh ueutmphilia: slight rosinnphilia. Strurlglt p c , \ i t i w immunohlot test for antibodies to echinococcal antigens: elevated dircct hilil-uhin and alkalinc plioupha~x~c.

rn

9 I4
c
vr

PE

D u
v ,

rn

Labs

Emaging

( : I ' / IJS, ahdomrn : rn~tltiple large septated tiver cysts impinging 011 hilt. duclr, prodl ring biliarv dilata~ inn (due tu ot)ut~.~~ction).
I ,iwt- i s most romrnorl site 01' invasion. Imt cysts m;>y also Tnrln in h~ngq. kidnev, horlr. and brain; each c ~ s conlaills t nlilllut~s of sculcccs and ~clrlrists of twn Iwrrs: aIr inner germinal layer. and arl otuer. Ialninntccl lavcr; t~\\rallt sz~rrnunclcd Ilv fil~rotic
reartion.

G~OSS Pathology

Micro Pathology

Giarlt cell 1,eaction s u r r o u n d i ~ ~ cyrl g wit11 r n s i ~ ~ o p h i l i c infiltration. S~~rgicallv rcmriye cyst.; il'po~sible:albcr~d;~/ole innv Ile efrcclivc-. Eclii~-iocr ,ccosir is a zooriosis protl~rcerl hy Erliinotocncs cfr~~t~trlrrvns. Tt is i~cc~ttir-srl through t11c ingcs~ic~n c~1.lbnclnr clriuk con rat~~inalerl wit11 Lhr=kcec of dngf 01. othcr carnivor.e%that I ~ W P eaten cnnraminatccl rriea!; humans are tlic i~itcrrriediale l~clst n f parasitic fat-vac. Acciden~al spilling of cvcl fluid, either sponti~~leotislt. or d ~ ~ r i surgcr!; ng nrilv 1.es~1lt in scconcl:r~-?I wetiing or slnaphvlaxis a n d evPn clcath. Also known 25 hydatkl tiisease.

Treatment Discussjon

ECHINOCOCCOSIS

I DJCC

A 28-year-old male who is a rcsidc~it of the southeastern United States presents with a high fever with chills, headache,
and myaleas.

HPI

1 T c remembers having been bitten by a tick a week hefore dcveloping 11% \yrnptoms; hnwcver, h e reportc no ski11 rash.
VS: fever. PE: no ski11 rash noled.

AE

Labs

CRC: le~iknpen ia and mild tl-trombocytopenia. Characteristic intralcukocytic inclnsion bodies and s~roIogic response 10 I:'lrrlwhin antigr:t.nsdenlonstrared: E, chafeensis c ~ th i~red horn blourl and clerecrerl by PCR.

Treatment
Discussion

Doxycycline.
Ehrlichieae ai-e gram-negative, ohliga trFy i l l tt-acellular bacteria. The nvo typrs of Ehrlirltin species h a t affect h u m m s are P:, r l w ~ f i ~ n (which ri~ attacks rnacrnphager and monocvtes) anrl an I;.'. ~qvi-like organism (which attacks granulocvtes). P r e r ~ n tivr measurcs i ~ i c l ~ l d weat-ing e rEothi11gthat cover-s the hod!, and

using itlspct repellants.

EHRLICHIOSIS

IDJCC

.2 SfLyenr+lrl male horn Texas pruscnts with fever and a skin raqh t ha1 I~eqanahour 2 wecks ago.

HPI

Tl~c o ~ l s cw l a q gradual, with prodromal ~ynptoiris of hc-adacfie, m;~l;~isr, hackacllr. ; ~ n d cl~ills. T h e w y m p t o m s wcse followerl by h a k i n g chills. f e ~ e ~ and ; a more fevcrc hraclaclle i~ccompaniecl Ilv rlausea ant1 vomiting. A remitten 1 pattern 01' Ccver accon~paniud hy tach!,cardia c o n t i i ~ u ~ Tor r l I 0 lo 12 clays, with thc rash appearing around the fifth day of fever.TIlr p a t i r ~ l l worked at a rat-infested food-storage depot this swnrtlrr.
VS: fcvrr. PE: discl-ete. irr egul;ll-pink macdopapular rash reerr in axillar anrl on trtunk. thigh^, and uppcr arms: b c e . palms, and soles only spars~lv involi~ed; mild ~plcnomegal! nnlerl.
'I'he WeilkFelix ngglutiilation reacticlr Knr F S - ~ I P %strain ~F OX-19 was positive; complcmen t-fixing t.nntjllorIie~ to thr tvpllus grflltlp an tiqen M e 1 e dernonqtrated: endemic typhus (dur lo Kr(f<~rtsi(~ { p l r i ) waq confirmed serologically hv u ~ i n g speci lic washed

PE

Labs

~-i~k~tt<iaI anrigells in IFA tcsts.


Treatment
h r til~iotic ~ trm tinent will1 doxycycline (chloramphenicol is used as an alterrlative).

Discussion

Mnrine rypliuq ic: a n;atlttal i n k c ~ i o n or rats ancl rnioe 13y Rir-k~th~ ty/)J/i; n spread of infection to humam by the rat flea i s incidental and U C C L I I . when ~ ~PCPF rrnm inf'cctrd fleas :ire st-ralchecl in lo he ler;ion. C :asps can occur war-round; l~nwever, rnc).;l ouc~u dl~i-ing the < a m m e r i~lottths, primarily in southern Texas and CaIiforni:t.

ENDEMIC TYPHUS

I DJCC

A '28-yeas-olcl G~raturri;tlai~ [male is brought tn I he hospital c ornplaining of severe headache. photophobia. and fives over
[ h e past 2 w c c k ~ .

HPI

As ;t political dissiderit. he spent 4 1nontFlq in a refugee camp in scn~thern Mexico bcfoc-e entering the United States.

PE

VS: frver (40C). PF:: papilledenla and delirium; hilatcral


swelling ol'parotid glanrls 1 weck later; ioxic racics: iilar~~lopapular rash or1 t r ~ i n k and extrcmitierj; facc, palms, and soles spared mild splenomc.galv.

Labs

Positive Weil-Felix reaction to OX-] $1 s~rairls of Prnfvlr r; rise in coinplrment lixation titer fat. Xirku//w'cr prouinz~rlrii: specific ijn ~itlorlies. U h : pro teirltiria; microscopic hematuria.
Mvocarditis arid ~ > n e ~ u n n n ]nay i a he presst~t: cerchral edema; maculopapular rash.

GFOSS Pathology

Micro Pathology

Zenker's degeneration of striated muscle: thrombosis and c-nclollrclial proliCeration of capillar-ips wi tEl abundan~ rirkettsiae and per-it~ascl~lar cr~ffing; :~ccuni~ilntiot~ nf lympfiowtes; microglia ancl inacropl~ages (typh~~s nodules) in hrain.

Treatment
Discussion

Doxycycfine: chloranlphenic~nf.
Epirlemic typhus is ;I febrile illness caused by Rictw/r'sEnp ~ m t ~ n w / < i i , : I am-neg~iive, nonmotile, olllipte i t 1 traccllular paraite; it is irat~sn-titted via body lice and is associated with war, famine, at-rrl crowded living conditions. The rash slinulcl bc diffrretltiated from Rocky Mountain spotled fevsr, which stal-tsperipherally on 11le w r i ~ t s ancl aiiklus and also irlcludcs rhr palr~ls ancl soles.

Atlas Link

1n _ 7 MC-169

EPIDEMIC TYPHUS

IDJCC

A 4-year-old n ~ a l e pr-eqents with rever, hoarseness, and respil-ator-vclistz-es~ hecause of partial airway ohstruclion.
T h e cli ilrl ir: also unable to speak clearly and bas pain while

HPI

swallowing (oi,wo~1-1.~~,1,2).
PE
VS: fevex-;lachypnea. PE: patient is leaning forward wid1 neck hyperextended and chin prom~ding; drooling; 1n;lr-kett suprasternal anrl infrasternal ~ , e ~ r a c ~ or i ochest: i~ inspiratory sb-idor on auscultatinn.
C~ul~ur-~~f~hrnats~vah(norolei~~n~ana~cmc-nloTacut~ disease) reve;ils pcnicillinase-resistnil t H,~~wnf,hrlu r injZu~11~a~: blood cultures alro positke.

Labs

Imaging

Xlt, neck: marked edema of epiglottis and aryepigloltic folds


( ' " I i u h i v l ~ s . ~ ~SIGN). r"

Gross Pathology

EpigFolti~ ir c11err.y-~ed, s~voll~i a~irt i , "angrre-looking." Rapid cellulitis o f epixlottif atid axrrorlnding risrur leads ro progeqsive hlorkage of airway.
P1,rserv;lriun of ail-way: IV rei'llroximc.

Treatment
Discussian

l ' h e principal carlse o r acute cpiglo~ti [is in oli i Iclr-en anrl aduIts is If. r n f l u ~ n z n epe ~ b; other pat hogens i tlclude If. pn~nintTumza~ and group A streptococcus. Clr~~ac~crized 1.r~ rapicL onset.
f '

Atlas Link

I I I

PG-MI-087

alelcc

14 30-ycral-old soldicl- \\-I10 had been admit red fnr a gumhot wound i r i the right thigli ~ I . I ' F P with ~ \ I Ssevere pain and swelling at rhe ~ i t o e f his injury

HPI

Tl~c pntirnt'~ right lower lirrlh hacl hvcoine (lis(-nl~)red. and rc\ cra1 biillae ll;~d appeared n n thr ckin. M e has ~ ; I F S C C ~very lililr lit-inr over t h r Iyast ~Eav,anrl the urine he 1135 plsscd has heen dark ("cola-color-erl").
X'S: Iow-gfatlc fcvrr: ma1-krd tachyc;u-ilia. PI;: iliaj~ll~nt.~sis; skin of rig ti^ thigh discolored (hton7e t(r purple red); sitc of in,jur>, cxquisitcl~ tendcr ailrl hcilsc and oozing a thin. dark. inid Fouismelling fluid; crepitus whiIe p;ll~,;~ling thigh.

Labs

C:13C: low heinntocl-it. Gram stain of cxudato and nccrotiu ~ilatcrial at wotmtl sitc ~-t~vealu prewncc of large 4gmm-pnsitive rods: ar-litcrfihicc~tltut-e oI'ex~lrlate and hlonrl 14,irlds Ciostriditi tn p ~ r f r i n ~ g type ~ n s A. clil rilrr isolate clernonstriltes positive Nagler reaction ( r h ~ e to pl-er~nce or alpha tc1xi11 lrti tl~i~last.) :[i~i.~lirr 1x11s corliirrn prewncc of intravascular hemolysis, myo- and hemoglohinuria, artd acute tubular necmsiq.

Irnagng
Gross Pathology

XR. rigllr thigh: prcsencc* o f gas in soft tissues.


Ovel-lvinq skin p~u-ple-hl-owe. inal-krcllveclcmatous uitli resict~lohullo changes ~ ~ ~ with IittEe wppurativc re;lctioil. Coagdative necrosis, edcma. gas formation. ilil(l 111;lnv 1;tt-ge gram-positivebacilli fclund E n affected 1111 twlr T i s q ~ w :l-elativelv sparsv inffllr.arion of PMNs norrd in IIW hnrch-ingmu5clc
L~FSIIP.

Micro Pathology

Treatment

S~~rgical clrhriduincnt; a~itihiotics (p~~iicilliii, c lindamvcin, t r . t r i ~ c ? ~ l i n~t~troilidazc)le); ~~e, liypel-ljaric oxygen thel-apv and p o l n ' i ~ l rI ?IIII ~ rilositl: S C I ~ I I ~ I -nlatlagetnen ~ ~ V ~ t of ils5ociatrd nlr11tiorgan faiIurc..

Discussion

A rapirllv progressive mvtmecl-osis causcd In. I:lc)s/~idirr 7rl /~n.friny~?lc h p c A, tr;l~nni~tic gas gallgT-mt.dc\~elr~ps i 11 ; 1 wound with luw oqguri tr-nuion (rrnherFcl~tl Ibi.eig11 horlie~ cr~ntaining c;ilrit~tiinr- qilicateq cause Inwering o f oxygrn tcnsir)ri, leacliirg 10 gcnnin;~tio~i of thc- spores).The most. inlpr~rtant toxin is thr alpha ~osin Iecithinasr, which pl+orluces31~rnolvsis aiid ~nvor~ccroqi.;.

Atlas Link

GAS GANGRENE-TRAUMATIC

ID/CC

,\ +veal--old frrnihc i s ~ I ~ D L I ~ I Ito I I h e perliau-ician hcc:~useof lack of appetite; nail uea ail tl vomiting; chronic, foul-smelling diarrhea without hloocl or mucur: and a bIoated ?ensation. S h r hat kven in several day-care centers over the pas1 3 years.

HPI
PE Labs

Low weight m d height for age: ~ nlrl i epigastric tcnclc-mess.


Binucleate, pear-shaped, flagellated trophozoites ( G I - \ R ~ I A I- I,LII{I./,I) nn li-eshly passed stool; cyst4 t i ~ ~ r o ld n stool cxam.

Treatment

Mru-onid;~zole.
T h e morr common protozoal infection in children in the United Staics, giarrZiasi5 is trans~nittcd rnairlly l h l - o ~ ~ contaminated gh Soocl or water and carlws rr~alal)~orl~t it 111.

Discussion

Atlas Links

I T 1 I-MI-089A. M-MI-089B

ID/CC
HPI

,4 3-day-old femalu rlconate preserlrs with n thick eye discharge.

The mother ad~nirs to traving mrlltiple sexual parhers and complains of a vaginal discharge. She did nor receive adequate
a n terlatal care.

PE

Exan1 or hot11 eve? reveal<a thick purulent discharge ilnd t~tat-ked conjmctival congestion and czdema; con,jurlctival chemosis is so marked that cornca is seen at hottom of a crater-like pit; corneal ulceration noird. Conjunctival w a h s on Gram staining revcal prcscnce of gramnegative diplococci both intra- and cx~racell~il;~rlv in adcli tion to many PMNs; coii,juncti\jal swab and m a t e ~ n a l cervical cultlzt-e yield N ~ i s s e r i a gonorrhocae. Aqueous penicillin G o r ceftriaxone For a total of 7 day%. Also trcat mutllcr ancl hcr scx~ial contacts. Educate the m o t h e r l-cgal-cling ~11r irnpor~mcr of safe wx.
I2atlwcl iw Neissen'o gonorrlaoeae. gonococcal ophthi~lmia nronator~im is contracted from ;I nio~lielwit11 gnnorrhea as the fetus paqaes down the Kith canal: infectin11 does not occur in utero. Corneal inflammation is the major clinical siqn that may psodticr. complications such as corneal opacities. perforation, i~nlrrior. s~,n~rhia an r ,[prior staphvlotna, and panophthalmitis. It is n o w common practice tu prevent this disease by treating the eyes o f the newborn with an antibacterial compound sl tch a ? er-yth~.ornycin oii~tmrnt or. 1 4ro siker nitrate; however, hume rhildhirth tyyasses t h i ~ pmphylactic procedure. and thus some casrs arc still occtirring in the United States.

Labs

Treatment

Discussion

"

GONOCOCCAL O P H T H A L M I A NEONATORUM

IDJCC

. 41!I-year-old white malc presents wit 11 burning urination:


pnjl'r~w, greenish-yellow,ptwulent urethral discharge: staining of hi5 rmrtcl-wca~.: and urethral pain.

MPI

Fu~lr days ngo. he hacl ~rnprotected sexual contact with a prostitute.

PE

M~~copurulent and sliglztly hlood-tinged rlrelhra4 disrhargc: nvrn~ll lesr es ; ~ i ~~picliclynlis; tl n c ) ttrinary retention.
Smear of urethral discl~arge reveals intrrrceUular pm-negative diplococci in U'&Cs: goi~ococcal infection conlirmecl Iw inoculation into Thayer-Martin medium.
Abundant. puruTe11t ul-ethral exudatr*.

Z n n rn

Labs

Cn
V1

rn rn

Gross Pathology
Treatment

Ccftri:txont. pl115ctoxyyclinc or crvthr-omycit~ Car possible cclinfection with Chlnmydia. hcommori S T 1 ~ c a u ' i ~ r ~ I 3 y ~ \ ~ ~ . 5 ~ ~ 7 ~ ~ ~ ~ 0 n o 1 r ~ ~ n ~ n , r , g o 1 1 a r r l ~ e a m a y iiixolvt. t h e t hl-oat. anus, rclrturrl. cliclirlynis, c e r ~ i xFaIlopia~~ . ~ I I ~ ~ prostate, P S . and-joi~~ts; con-i~~nrtici is t ir~ t l s o fol~rkcl in nronatcs. Neonatal coniunctivitis may be prevcnr~d through die i nstillai i c l n or qilrcr nitrate or er~y~ltl-nm)~irl eye drops at birth.

Discussion

ID/CC

,A PH-year+dcl malr immigrant prtsenls with inguinal swelling and a painless penile ulcer.

HPI

Hc. ; ~ t l n ~ i 10 l s ~~npt-oteclccl i t 1 t e l ~ n l l wit r ~ 11 ~ m~iltiple sexual partners. mnny o f wIir,m were proqtitr~tcs.Mu first noticed a papl~lr on Elis penis ~ t r ~ ~ ~veeks r a l ;i~o.
.Soft, painless, raised, raw, beef-colored, smc-mth granulating ulcer ~ ~ u t e 011 d rlis1~11 per1i.c; m11Itipl~ .mbcutaneo~~s swellings (PU C'I)IIIICR~KS) nntccl in inguin;il rcgirln, .;ctrrle clf which have ulcc~+;t tcd.
( ; i e m ~ ~ - % t ~ ismear n e r l rrnm penile and inguinal rrgirms rlplnnnstrate cl~al-actelistic "closed safety pin* appearance of unr ap.ir~Fa tcd o i ~ ~ a ~ ~ within i s r ra~ la ~r g histkyle

PE

La 6s

(Dc IL.E>V'INR O ~ T F S. )
Micro Pathology

Chnrartet.istic histologic picture of c l o u o r a n o ~con~pl-ixes i~ snme r l e ~ ~ cuf - rr-pithtlii~l hl;purplasi;~;II m;\rgin.; 01-Iesiot~s: rlensc

p l , ~ ~ ~cell n ; i iiafiltratr ?carter.;hiqtincytexontaining Dorlwan horlies.


Treatment
Ti-rn~ will1 daxycycline
C I dou'bfe-s&ength ~

TMP-!!MX.

Discussion

G r i ~ n u l o r tinguin;~le, ~;~ a slowJy progr.essi\.r, t~lcel-ativr, ~rat~~~lonia STU ~ o ~involving lss he genicalin, is causvd hv the gratll-negative hncillus Cal~rnmataGacteriurn grcznulomofi s (forrncrk Donnlnrlio gn~zrrlnmoti\) : it i s weti in Gietnsa-stailled srrtio~~ as s :1 dark-ftainilig. e n r a l > s ~ ~ l a i e intl-acelllrlar d, roclshaped i n c l ~ ~ s i n in n inacrt~ph;~gcs. thc. so-cdTlutl Donovan body. T h e rliscasc i s undrmic i l l tropic;ll ,Irra< w c h ar NPIV G~ritlea,

r"b GRANULOMA I N G U I N A L E

IOJCC

A 60-year-old male present5 with cough productive of mucopnruIent sputmrn ~ U ~ F C I I w Ci i~ 11 mild f'cvc-r a1-rtl wo~-senirig l~rralhlr~snc\s.

HPI PE

He i u

;ichronic

smoker who Ii;~s 1~ec.11 tliagnocrd

will1

COPD.
H

VS: k r ~ c r1' . E: in mocleratr respiratory clistl-cxs: cn~phvsrrnatous rhest with ohliterater! ral-diac anrl liver rl111lnc.s~: wheezing and crackles hcard rwer both lizng ficlrl~.
Jla~nlophilws E r ~ f l ~ r ~ norganisms zne wen as small, plcr>nlorpl~ic gi-an~-nrgdti~t= h;tciIIi o n Gram r1;lin or splitu In: nontypable H . i?~fluenzae isolated on sputum culture (tc) grow i l l CIIITII~P. / j . j r t f l ~ ~ ~ ~rrq~lircs l t r t ~ ho~li I ~ C I C ) I X-hematin . allrl fartor \'-nicntina~nitle n ~ ~ c l e o ~ ipresent rle in erythrocytes)

z
0

n rn

Labs

0 C cn
CI
H

LA

lA

rn P
rn

Treatment

Amoxicillin/ampi&llin therapy; TMP-SMX. mithrnn~yciil, and r1;~rit hrornyrin zu-e also excellenl rE~.(~qs li>rrlre trearment of clinically milrl to 1notleixt.e H. 1?7/7jr~nzmr in feet ion5 or the upper ~-r<piratc)ry tract.
It~l'ertinns rat~wcl 1 3 ~ ~lonlypahFe, or ~~nrnrnpaxlaterl, Ilrr~vnt~/~ rn/?ri/wzr~o h ~ l / ~ ~ sti.;til~~ have heen inc.rea~inglv recognized in prrliatl-ic ; ~ r l r ladult pupula tions. Non t!prtblr H , i r l f l ~ t ~ ~ ~ z n r strains arc frequent re~piratory tracr colonizers in pa~ients with .(:OPn i l ~ l d ~o~lmo~ t~ l liy l~trl~ c ll li rr ~ ~ I~rortcliili~ ii~ in l h t ~ p:~ti~tits: nonlvpnble slruins are alco t h e most common caiiw of acute n~itis nlccli:~i r l chilrlren.

Discussion

"

H. INFLUENZAE IN A COPD PATIENT

,2 25-vu;1r-olclinaie prescntecl with s~~dden-onset breathlessness, cough, cyanosis, and hi~h-grade fever.
'The patient failed to iinprorr on 100%)oxygen, became hvpotensivc., and died nf type 2 respiratory Cailure ;I few hours after at1111 isrinn. I-Ie harl hcrrl in per-lkct health a n d hart hreen hiking in several rodent-infested arcaq bcfor-r fall it^^ ill.
PE

On admiwiun he was fuurlcl In I-iavc fever, tachycardia, cyar~osis, ~ ! - ~ V P O ~ U T ~and S ~ O rales II, on auscultation over hoth Iring fields: no rnenir~geal signs o r localizii~g CNS ~igtls could be denlonstrated.
ABGs: rcspit atol-y acidosis with hypoxia and hypercapnia. CBC: 7r~tkc)cytnris: hemoconcenbation: Chrombocytopenia: aivpical hmpliocytn~is. Increased LDH anrl r\LT Icvcls: prc~lnngecl PT irl~lex: spl~tum exam and blc~od rultusc clid not vielrl anv 01-panism;IxM alr ~ihc)dv to han taviruri and irnmuuol~istocl~emical srains thr hantavirus antigen in tissues confirmed infection with
thr v i r ~ ~ s .

Labs

Imaging

CXR: n o n c d o g e n i c pulmonary edema (bat-wing edema


pattern).

Micro Pathology

Hislopathnlogic cxam of lung tissues was s~tggesf ive of acute respiratory distress syndrome (ndult 11ylinr rne~nhrane disease).
Palien1 died dcspitu intensive ventilatory support (Sin Nombre virus mosl freqr~et~ tly c a ~ ~ s hantavirl~c cs pulmanary syndsorn~ in the LTnited Srates) .

Treatment

Discussion

A 1i1-risclnsefy related to thc Hantaan virus (which produces Roreal~ hemorrhaI5ic Fever anrl hemorrhagic fever- with renal y-nrlrornt.) I1a.i hren recovered from mice in varinlrs rcgionc of' the Unitcd Slales: rodents are the natural host Tor chis group nf 1 irliwq. Inferfed rocluilts shetl the virus in saliva, urine, and feres for mall! weeks, and humans ai-r believed to acquire the infection via exposure to rodent excrement or saliva. rirher bv the arro\ol route o r !ly direct inncr~latinn.

@@? v

HANTAVIRVS PULMONARY SYNDROME

ID/CC

male who works its ;I L-.N. liealtli worker prescnts with a high-h~adc fever and marsive hernaternesis.

iZ 35-year-olrl

HP I

I-le rrcently r c t ~ ~ r n efsmn cl Zaire, where he w,vot.Iierl in a


tick-infested forest.
M

Z 7 m
r-l

Labs

CBC: let~kopenia; severe thrombocytopenia. LlTs: eleval ed AST. Crimean-Congo virus isolated.
Treatmcnl invrhvrs a 1(&daycnlirse of ribaviri; platelet bansfi~isions: avoid salicylates; harrier nursing alld containment ir-tfecrerl s ~ r s ~ t i n n since s, airhome infectio~l may occur in hnspital envil.oninen t.

8
C
vl

Treatment

Discussion

Tlle iigenl responsibIe fbr C:riinean-Coilgo l~cmorrl~agic fever is a bunyavirus; reservoirs inrhide wild ancl tlorrles~icatedsheep, cattle, gnat.;, and hares. The cliseasc is trausnritlerl I>y a tick vector, usr:dy an ixodid of the genuc: Hynlo?nmrx; endemic area? include the Middle Fact atid wesrerti China. The diseasc targets

HEMORRHAGIC FEVER-CRIMEAN-CONGO
--. --

ID/CC

A I0-veat.-r,ld i~ b r o u g h t to t h r ER in a staw clr shock accompanied Ijv massive h~rnatemesis.


T ! I ~filrnilv l~;itl,jirsl ~ - r ~ t u r r ~froln e c l ;I var:~licln in ThaiIand. Ilis p;rrellt\ .;;I$ th;l~ lir h , ~ ;I l l~iqh-gtarlprevel- for *5 tn t i dnw, for which he ~ v ; l q rrcri~ing prcwmptivr treatr~ici31 f i r malaria.

HPI

PE

1's: ll\pr>tcnsion: tL~chvtarrli;t. f'S: cool, c l ; ~ ~ n m euil-en-fitir'i; ~, petechial skin raqh over err[]-c-rnities, axillar, rrumk, a n d fncc: hle~rlit~ frnrn g vpnipl~t~ctul-e <ices.

Labs

CRC:: thrornhocytopenia;hernatocrit increa~ed hp > 20%. 12T)~~orn clottillg ~ i ~ l prnfile 51iq~es1 ive 01-disseminated intravasa~llar coagulation (DIC); ])air-erl Fern reveal ~ignilicant riw in titer r d h c ~ n a ~ ~ l u i i n a tinliil~itic>r~ io~i an~ibot1ic.s against Dengue virus seratypes 1 and 2.

Imaging

US:I~ilal~i-a1 [ d r t ~ r di'~1qio11 ~! ;IIICI awitm.


S~mipramatic: mauilgv s l ~ o c lwith ; fluid% a ~ ~ hen~orlvnamic cl r r i o i~ I r~ ~I-~IIR li-r~h : hlnnrI /~>Ialeler-l-iclz placlnn: moicl salicvIates. l)el~yle hcmnl-r-hagic Fcvcr- ic causcd by ;I mosquito-borne 0lrrZ1~ mp/)/i) flavivirtw : ~ i i ( I i~ clli11.:1ct~l-i7c-d bv I'ot~r C ~ ~ S L ~ ~C ~ T C I L~ I I C F C ~ O I T T J C S (ITTC : !i.; cousiclrrcd thc nlort cl;tr~ger.o~r~). /I. ~t~my)l,li 1l;ls ;I tiorllcstic. 11;1F,ilal (st;ign;21\1 warer- in flowel- pots. olcl,j;~r.s. ti11 calls, a~1(1 old lire\) and I j i t ~ ~ I~~ w i n ~g Ile clay. Dcnguc Fwer ha< <tiowlt atz irlcreaw in i~~cirlence i11 Southeast Asia, Central and South America. anrl the Caribbean. Sirlce r l r l specifir tllerapv t.xi-,t.;. I N P V P ~ I lw a v o i r l i ~ ~ rtttltacl g with inrcc~ed rl. riqq/)ti.
F

Treatment

Discussion

Atlas Link

1 1-MI-09.6

. I,i4%vc-;u-ol(l rtian M 110 was hitch liiLi1ix ~1ii.c )ugh central and
southern Africa w,js i ~ d i n i ~ z r 10 t l il Iio\pit:ai i l l 2nil.e ill a s t x e 01 rliock ~[IIIoM i ~ i massive g hemorrhage from the G1 tract (hcniaternesis a11d ~ n e l ~ t l n 11r ) : rlierl wilhin t i 11o~i1.s of ;~rlmir;sioii. IPn rlavr lait-r. a nialcb doctor who had attended this patient nnrl had ; ~ t ~ e n ~ p t r cc rs l ~ ~ station ci F~c~irn illcwith a similar disease svlirlro~nr.

At ;lrlrn iscioii, hc jiavc. a11 X-d;lv h i.;rorv or pl-o~i-es\ivt. fever, sevcrc headachcs, myalg-iaq,and watery diarrhea. 1-lc also t - ~ l m r t e ra l n er\'tti~l31alc,1is. measles-like skin rash tllat Ililil t>eg~~ rr-,nrlerq~ialnate.
PE

VS: f r ~ c rPE: . .;plci~orncg;~lv: hcp:~tr)rneg;~lv.


CBC: Icukol~cni;~; Pclgcl--I IuCt ant>inaly0 1 nt-ulr opl~ils wirh ;lnpic;~lr n r ~ n o i ~ i ~ c l ccll'i: c a ~ -thromhocytopenia with abnormal plateIet a.gregation, hfnrkrrllv rlthva~rrtASI' arid XL1': hloorl w;ls inocul~~terl in ~l-apc=ritonc:llly into young guinea p i p ;ii ltF i 11I ~ I varir ruq tissue c ~ ~ l t ~ uvll irc lints, ant1 Ebola Yirus waq detected hy indirect immunoflnorescent staininc l r c 1111 iq11t.r.
At itillopn: Iymph nodes, liver, and spleen L'mlud tr) I>cmoqt ct)nspic~lc~~~slv iilvol~ccl( r t p l i c a ton ~ I lf F.I,ol;i virAil\cmn nrrllt- it1 vi1 tt~nll! ;tll r>i-~;ir~s); .s~nin;ir-11 anct i r l i ~ s t i t ~ fillcd r ~ with hlood:

Labs

Gross Pathology

prterhii~r feet1 over I>t>~tteI In ilco\.!.

Micro Pathology

Scvcrc cungcstion and stasis OF splccll; widespread necrosis of' liver r ell\: electron micmscopy ot' li~mrlre~ealerl plcomarphic jll-ppal-atioi~s ;IF long, virus particles ii1q)ea1-rti~ i c(lrr t I.;~FI filamentous forms, U-shaped farms, and some cirailar forms resembling a doi~dlnut.

Treatment

S1rppn1.ti1.rcare. .;inr.r 110 ~pecific 1rrntnicnt exists: ;I prior n111 I,rc;lk w;ic h1.1~1gh L u~ ~ r lc co rn ~ r n Iw l istl1:itin~p:llicnts ,111d instituring ctricr h l r ~ i t iriir~-cir~q. 11

Discussion

henlorrl~rigic~ Ccht.iIe infectinn o f humi~ll\ cl~ic. ir, iiiti.ction \villi Elmla anrl hlai.hurg iirllscs. hoth nf wli icli ,lrc Iili>\ ir.i~<ec t Ii;i~ arc s~r-~~c~ul-nIIv inclistingt~isl~;~!~lc hiit ; ~ i ~ ~ i g c i i i r tli\ziilc~. . n l l ~ Tliiu disci~w is a roor~osiq 1>111 I he r.c-sri.\.oii.is 1111 k ~ ~ o t \ . 1nrlivirl~l:~l~ n. c;ln Iwcnmc iuCect~cl rl~rolrghpel-<r>ll-tc+prx-qoti 01n~'ii)~(~ii rot1 ~ i iIl; l LC~.
t lie

+-

H E M O R R H A G I C FEVER-EBOLA

VIRUS

ID/CC

A 25-vcas-old male ~voodcutter who Iives in South Korea is arlmittcd to ihe ER in a state of shock and massive epistaxis.
Tlic p a l i ~ n had t heen complaining of fever, malaise, hradache. rnyl4+as, hack pain, abdorni~lal pain, nausea, and vomiting for ihe paqr week; he also complainctl of extremely reduced urine output. Cartf~zI histor,y revealrd that heforc IIC fell ill, 11eand his friend were cutting MTC)UCI in the foresr wlien they accide~~rallv disturbed a rodent-infested area.

HPI

PE

VS: l~ypntrnsiun. PE: epistaxis: facial fluqhing; pctecljiae a118 suhconjunctival hemorrl-tap.

Labs

Deranged RT;Ts suggestive of acute renal failure. CBC: thrombocytopenia. Surology and culture identify hantavirus, Hantaan serotype.
Supportive mauagemcnt in the form or dialysis (for rend failure); management of shock and he~norrhagc; IV ribavirin (must start within first 4 days of manifertativn uf disease).

Treatment

hetnnrrhagic fever with renal syndrnme is caused hv the D ~ S C U S S ~ Korean O~ Hantaan serotype of bantawrus. 119 reservoirs are v a i i o u ~ r-odcntc rllai are Found distributed over Enrope and Asia: h~~tnans acquire t h e disease mainIv by in ha1 ing aerosols of rodenr virus.

H E M O R R H A G I C FEVER-RENAL

SYNDROME

ID/CC

.%7-year-oldnlalc complains a f a high Fever arzd a very sore


throat.

HPI

T h e paill is st I severe t t i ; ~ lt l ~ CEI r ilrl I - ~ ~ L ~ S10 P Snvnllr>\v.HL'is arlcql~atcIyimmt~nized and achicvcrl norm:tl drwlr~pmen~al

milcctone~.
PE

YS: rr.1x-r. PE: characteristic grayksh-white vesicular lesions,same of rvhich h a w lilccrated. nutccl ovri- soft palate anrl tonsils.

Labs
Treatment

Corcsackievjrus A isulalvd h-nm m ~ ~ c o v a le\icrnx. l


Srlf-limiling cc)nrlilion.

Discussion

In Rand, foot, nnd mouth diseaqe (HFMD), patien I.\ co~rlplain ol' lever. weakness, and clecl-eased appet itr alniig wit11 ?iin~iE~ir le.iion5 rioted i l l lhr or-a1 c;n'ip, palm\, uoleq, irnrl I~s~ttncks. H~rpatlgina may 1 1 ~ rausrrl bv coxsackievir-us XI-A1 0, -416, A22. and B I-B5. Outhrcaks of I-IFMD are usu;~llycauqrrl by co.;wckievirtis X I fi.

HERPANGINA

ID/CC

A 25-vviar-old hoinoscxl1;11 malr visits ;I l ~ ~ i ~ clinic l t l complaining ~ of hcaclacl~c,low-~radttFever, ant1 n painful skin raqh in the perianal area. I-Ic 112s 110 1listor.y trJ penile uFcel-atinnsand aclrnit? to unprotected anal sex with mulZipIe partners.
Pcrinnal vesicular rish in rlustcrs on erythemataw base; iio p~tiile 111cer.arion: pniilf~~l irlgltinal Iynphitdenopathy.

HPI

PE

Labs

M~lltinucleatedgiant ceIIs with intramclear inclusions surroundcrl 1 3 : clear I ~ a l a on Pap-st;~inerl scrtioi~ or Tzanck prt~p;tratiuuol'scrapiragsfrom hasr nl'veqirler;.
Clcar liqnid i r ~ veqicler: sccoudi~ry hi~c~erial infection mxv result; pain till ~~lcet+ations when ~rcsiclus rtiptlirr.
J~~tlarnrni~ror! infiltrnte witli abt~ndant Iynphncyt~s.

Gmss Pathology

Micro Pathology

I I

Treatment Acyclovir
Discussion

An r n v r loprd. dr>l~hlc-s~r;~rldetl IINA vit-11% t rar-rsn~itted b v sexual cotllact. HSV 2 has a tendency to renir anrl can I>etransmitted to the fetus through the birth canal. C:c)n.rlo~m n s p appear? to I>e o n e n l r I I rnmt ~ r li'ective means oI' pre~enting ti an~rnissiot~.

Atlas Links

n M-MI-100

IM2-019A,IM2-019B

HERPES GENITALIS

ID/CC

A ,L5-yr;tr-olrl HlV-positive male i s seen by his family doctor fclllowiilg the appearancr of a painful, burning skin rash nn the left sidc of his chest that i s acr~m~paniecl b y a headache and Io~~~-grarie fevur.
Thc wtien t had r h i c k e n ~ o x a.; a rhijd. He l~acl bee11 well IIIE~1 ~ Iw a r ago, when he was rliagnoscd with non-Hodgkiu's lymphoma, for ~cllichh e i s rl~rrendy urtdcrgoing chemotherapy.

HPI

z n
c
Cn
LA

PE

Vesicl~lar rash on erythematol~s 'base; in dermatomal diwibution (lcft TtiTX) : rxquisitely tcnclei- to I otlrh.
Acan~holv~ic cell.: on Tzanck smear frorn base of vesicles.

z ! 0

Labs Micro Pathology

Intranudear eosinophilic indusions surrol~nded by clear halo

Treatment

Acyclovir.
Shir~gles 1.csprPsent5n reactivation Or3 I n t e ~ infection ~t \rrith varicella-zosterv i m ; the rash typicallv follows the distrihu~iota of a nerve rcjor. I t i? comnlanly cren in immuaosuppres~ed patient% and is alro associ:tlcd with u-numa, ultr.a~iolet radiation, hypnd~ermia. and emotional stress. Portherpetic neuralgia is a common cotnplication in thc uldrrlv.

Discussion

* I

HERPES ZOSTER (SHINGLES)

Das könnte Ihnen auch gefallen